06.08.2013 Views

Problems and Solutions in - Mathematics - University of Hawaii

Problems and Solutions in - Mathematics - University of Hawaii

Problems and Solutions in - Mathematics - University of Hawaii

SHOW MORE
SHOW LESS

Create successful ePaper yourself

Turn your PDF publications into a flip-book with our unique Google optimized e-Paper software.

<strong>Problems</strong> <strong>and</strong> <strong>Solutions</strong> <strong>in</strong><br />

REAL AND COMPLEX ANALYSIS<br />

William J. DeMeo<br />

July 9, 2010<br />

c○ William J. DeMeo. All rights reserved. This document may be copied for personal use. Permission to reproduce<br />

this document for other purposes may be obta<strong>in</strong>ed by email<strong>in</strong>g the author at williamdemeo@gmail.com.<br />

Abstract<br />

The pages that follow conta<strong>in</strong> “un<strong>of</strong>ficial” solutions to problems appear<strong>in</strong>g on the comprehensive exams <strong>in</strong><br />

analysis given by the <strong>Mathematics</strong> Department at the <strong>University</strong> <strong>of</strong> <strong>Hawaii</strong> over the period from 1991 to 2007. I have<br />

done my best to ensure that the solutions are clear <strong>and</strong> correct, <strong>and</strong> that the level <strong>of</strong> rigor is at least as high as that<br />

expected <strong>of</strong> students tak<strong>in</strong>g the ph.d. exams. In solv<strong>in</strong>g many <strong>of</strong> these problems, I benefited enormously from the<br />

wisdom <strong>and</strong> guidance <strong>of</strong> pr<strong>of</strong>essors Tom Ramsey <strong>and</strong> Wayne Smith. Of course, some typos <strong>and</strong> mathematical errors<br />

surely rema<strong>in</strong>, for which I am solely responsible. Nonetheless, I hope this document will be <strong>of</strong> some use to you as<br />

you prepare to take the comprehensive exams. Please email comments, suggestions, <strong>and</strong> corrections to<br />

williamdemeo@gmail.com.<br />

Contents<br />

1 Real Analysis 3<br />

1.1 1991 November 21 . . . . . . . . . . . . . . . . . . . . . . . . . . . . . . . . . . . . . . . . . . . . 3<br />

1.2 1994 November 16 . . . . . . . . . . . . . . . . . . . . . . . . . . . . . . . . . . . . . . . . . . . . 10<br />

1.3 1998 April 3 . . . . . . . . . . . . . . . . . . . . . . . . . . . . . . . . . . . . . . . . . . . . . . . . 12<br />

1.4 2000 November 17 . . . . . . . . . . . . . . . . . . . . . . . . . . . . . . . . . . . . . . . . . . . . 18<br />

1.5 2001 November 26 . . . . . . . . . . . . . . . . . . . . . . . . . . . . . . . . . . . . . . . . . . . . 20<br />

1.6 2004 April 19 . . . . . . . . . . . . . . . . . . . . . . . . . . . . . . . . . . . . . . . . . . . . . . . 27<br />

1.7 2007 November 16 . . . . . . . . . . . . . . . . . . . . . . . . . . . . . . . . . . . . . . . . . . . . 32<br />

2 Complex Analysis 38<br />

2.1 1989 April . . . . . . . . . . . . . . . . . . . . . . . . . . . . . . . . . . . . . . . . . . . . . . . . . 39<br />

2.2 1991 November 21 . . . . . . . . . . . . . . . . . . . . . . . . . . . . . . . . . . . . . . . . . . . . 43<br />

2.3 1995 April 10 . . . . . . . . . . . . . . . . . . . . . . . . . . . . . . . . . . . . . . . . . . . . . . . 48<br />

2.4 2001 November 26 . . . . . . . . . . . . . . . . . . . . . . . . . . . . . . . . . . . . . . . . . . . . 52<br />

2.5 2004 April 19 . . . . . . . . . . . . . . . . . . . . . . . . . . . . . . . . . . . . . . . . . . . . . . . 59<br />

2.6 2006 November 13 . . . . . . . . . . . . . . . . . . . . . . . . . . . . . . . . . . . . . . . . . . . . 60<br />

2.7 2007 April 16 . . . . . . . . . . . . . . . . . . . . . . . . . . . . . . . . . . . . . . . . . . . . . . . 64<br />

2.8 2007 November 16 . . . . . . . . . . . . . . . . . . . . . . . . . . . . . . . . . . . . . . . . . . . . 68<br />

2.9 Some problems <strong>of</strong> a certa<strong>in</strong> type . . . . . . . . . . . . . . . . . . . . . . . . . . . . . . . . . . . . . 69<br />

1


A Miscellaneous Def<strong>in</strong>itions <strong>and</strong> Theorems 71<br />

A.1 Real Analysis . . . . . . . . . . . . . . . . . . . . . . . . . . . . . . . . . . . . . . . . . . . . . . . 71<br />

A.1.1 Metric Spaces . . . . . . . . . . . . . . . . . . . . . . . . . . . . . . . . . . . . . . . . . . . 71<br />

A.1.2 Measurable Functions . . . . . . . . . . . . . . . . . . . . . . . . . . . . . . . . . . . . . . 71<br />

A.1.3 Integration . . . . . . . . . . . . . . . . . . . . . . . . . . . . . . . . . . . . . . . . . . . . 72<br />

A.1.4 Approximat<strong>in</strong>g Integrable Functions 1 . . . . . . . . . . . . . . . . . . . . . . . . . . . . . . 73<br />

A.1.5 Absolute Cont<strong>in</strong>uity <strong>of</strong> Measures . . . . . . . . . . . . . . . . . . . . . . . . . . . . . . . . 74<br />

A.1.6 Absolute Cont<strong>in</strong>uity <strong>of</strong> Functions . . . . . . . . . . . . . . . . . . . . . . . . . . . . . . . . 75<br />

A.1.7 Product Measures <strong>and</strong> the Fub<strong>in</strong>i-Tonelli Theorem . . . . . . . . . . . . . . . . . . . . . . . 75<br />

A.2 Complex Analysis . . . . . . . . . . . . . . . . . . . . . . . . . . . . . . . . . . . . . . . . . . . . . 77<br />

A.2.1 Cauchy’s Theorem 2 . . . . . . . . . . . . . . . . . . . . . . . . . . . . . . . . . . . . . . . . 77<br />

A.2.2 Maximum Modulus Theorems . . . . . . . . . . . . . . . . . . . . . . . . . . . . . . . . . . 78<br />

B List <strong>of</strong> Symbols 79<br />

2


1 Real Analysis<br />

1.1 1991 November 21<br />

1 REAL ANALYSIS<br />

1. (a) Let fn be a sequence <strong>of</strong> cont<strong>in</strong>uous, real valued functions on [0, 1] which converges uniformly to f. Prove that<br />

limn→∞ fn(xn) = f(1/2) for any sequence {xn} which converges to 1/2.<br />

(b) Must the conclusion still hold if the convergence is only po<strong>in</strong>t-wise? Expla<strong>in</strong>.<br />

Solution: (a) Let {xn} be a sequence <strong>in</strong> [0, 1] with xn → 1/2 as n → ∞. Fix ɛ > 0 <strong>and</strong> let N0 ∈ N be such<br />

that n ≥ N0 implies |fn(x) − f(x)| < ɛ/2, for all x ∈ [0, 1]. Let δ > 0 be such that |f(x) − f(y)| < ɛ/2,<br />

for all x, y ∈ [0, 1] with |x − y| < δ. F<strong>in</strong>ally, let N1 ∈ N be such that n ≥ N1 implies |xn − 1/2| < δ. Then<br />

n ≥ max{N0, N1} implies |fn(xn) − f(1/2)| ≤ |fn(xn) − f(xn)| + |f(xn) − f(1/2)| < ɛ/2 + ɛ/2 = ɛ. ⊓⊔<br />

(b) Suppose the convergence is only po<strong>in</strong>t-wise. Then the conclusion is false, as the follow<strong>in</strong>g counterexample<br />

demonstrates:<br />

Def<strong>in</strong>e fn(x) to be the function<br />

⎧<br />

⎪⎨ 0, if 0 ≤ x <<br />

f(x) =<br />

⎪⎩<br />

1<br />

2<br />

2nx − (n − 1), if 1 1<br />

2 − 2n<br />

≤ x ≤ 1.<br />

1, if 1<br />

2<br />

1 − 2n ,<br />

1 ≤ x < 2 ,<br />

That is, fn(x) is constantly zero for x less than 1 1<br />

2 − 2n , then it <strong>in</strong>creases l<strong>in</strong>early until it reaches one at x = 1/2,<br />

<strong>and</strong> then it rema<strong>in</strong>s constantly one for x bigger than 1/2. Now def<strong>in</strong>e the sequence xn = 1 1<br />

2 − n . Then fn(xn) = 0<br />

for all n ∈ N <strong>and</strong> xn → 1/2, while the sequence fn approaches the characteristic function f χ [ 1<br />

2 ,1] which is one<br />

on [ 1<br />

2 , 1] <strong>and</strong> zero elsewhere. Therefore, f(1/2) = 1 = 0 = limn fn(xn). ⊓⊔<br />

2. Let f : R → R be differentiable <strong>and</strong> assume there is no x ∈ R such that f(x) = f ′ (x) = 0. Show that<br />

S = {x | 0 ≤ x ≤ 1, f(x) = 0} is f<strong>in</strong>ite.<br />

Solution: Consider f −1 ({0}). S<strong>in</strong>ce {0} is closed <strong>and</strong> f cont<strong>in</strong>uous, f −1 ({0}) is closed. Therefore S = [0, 1] ∩<br />

f −1 ({0}) is a closed <strong>and</strong> bounded subset <strong>of</strong> R. Hence, S is compact. Assume, by way <strong>of</strong> contradiction, that S is<br />

<strong>in</strong>f<strong>in</strong>ite. Then (by theorem A.1) there is a limit po<strong>in</strong>t x ∈ S; i.e., there is a sequence {xn} <strong>of</strong> dist<strong>in</strong>ct po<strong>in</strong>ts <strong>in</strong> S<br />

which converges to x. Also, as all po<strong>in</strong>ts are <strong>in</strong> S, f(xn) = f(x) = 0 for all n ∈ N.<br />

We now show that f ′ (x) = 0, which will give us our desired contradiction. S<strong>in</strong>ce |xn − x| → 0, we can write the<br />

derivative <strong>of</strong> f as follows:<br />

f ′ f(x + (xn − x)) − f(x) f(xn) − f(x)<br />

(x) = lim<br />

= lim<br />

= 0.<br />

n→∞ xn − x<br />

n→∞ xn − x<br />

The last equality holds s<strong>in</strong>ce f(x) = f(xn) = 0 holds for all n ∈ N. ⊓⊔<br />

3. If (X, Σ, µ) is a measure space <strong>and</strong> if f is µ <strong>in</strong>tegrable, show that for every ɛ > 0 there is E ∈ Σ such that<br />

µ(E) < ∞ <strong>and</strong> <br />

X\E<br />

|f| dµ < ɛ.<br />

3<br />

(1)


1.1 1991 November 21 1 REAL ANALYSIS<br />

Solution: For n = 1, 2, . . ., def<strong>in</strong>e<br />

Clearly,<br />

<strong>and</strong> each An is measurable (why?). 3 Next, def<strong>in</strong>e<br />

An = {x ∈ X : 1/n ≤ |f(x)| < n}.<br />

A1 ⊆ A2 ⊆ · · · ↑ A <br />

∞<br />

n=1<br />

A0 = {x ∈ X : f(x) = 0} <strong>and</strong> A∞ = {x ∈ X : |f(x)| = ∞}.<br />

Then X = A0 ∪ A ∪ A∞ is a disjo<strong>in</strong>t union, <strong>and</strong><br />

<br />

|f| = |f| +<br />

X<br />

A0<br />

A<br />

<br />

|f| +<br />

A∞<br />

An<br />

<br />

|f| =<br />

A<br />

|f|. (2)<br />

The first term <strong>in</strong> the middle expression is zero s<strong>in</strong>ce f is zero on A0, <strong>and</strong> the third term is zero s<strong>in</strong>ce f ∈ L1 (µ)<br />

implies µ(A∞) = 0. To prove the result, then, we must f<strong>in</strong>d a measurable set E such that <br />

|f| < ɛ, <strong>and</strong><br />

A\E<br />

µ(E) < ∞.<br />

Def<strong>in</strong>e fn = |f|χAn . Then {fn} is a sequence <strong>of</strong> non-negative measurable functions <strong>and</strong>, for each x ∈ X,<br />

limn→∞ fn(x) = |f(x)|χA(x). S<strong>in</strong>ce An ⊆ An+1, we have 0 ≤ f1(x) ≤ f2(x) ≤ · · · , so the monotone<br />

convergence theorem4 implies <br />

X fn → <br />

|f|, <strong>and</strong>, by (2),<br />

A<br />

<br />

<br />

lim<br />

n→∞<br />

An<br />

Therefore, there is some N > 0 for which <br />

F<strong>in</strong>ally, note that 1/N ≤ |f| < N on AN, so<br />

<br />

µ(AN) ≤ N<br />

|f| dµ =<br />

X\AN<br />

AN<br />

A<br />

|f| dµ =<br />

|f| dµ < ɛ.<br />

<br />

|f| dµ ≤ N<br />

X<br />

X<br />

|f| dµ.<br />

|f| dµ < ∞.<br />

Therefore, the set E = AN meets the given criteria. ⊓⊔<br />

4. 5 If (X, Σ, µ) is a measure space, f is a non-negative measurable function, <strong>and</strong> ν(E) = <br />

measure.<br />

E<br />

f dµ, show that ν is a<br />

Solution: Clearly µ(E) = 0 ⇒ ν(E) = 0. Therefore, ν(∅) = µ(∅) = 0. In particular ν is not identically <strong>in</strong>f<strong>in</strong>ity,<br />

so we need only check countable additivity. Let {E1, E2, . . .} be a countable collection <strong>of</strong> disjo<strong>in</strong>t measurable sets.<br />

3 Answer: f is measurable <strong>and</strong> x → |x| is cont<strong>in</strong>uous, so g = |f| is measurable. Therefore, An = g −1 ([1/n, n)) is measurable (theorem A.2).<br />

4 Alternatively, we could have cited the dom<strong>in</strong>ated convergence theorem here s<strong>in</strong>ce fn(x) ≤ |f(x)| (x ∈ X; n = 1, 2, . . .).<br />

5 See also: Rud<strong>in</strong> [8], chapter 1. Thanks to Matt Chasse for po<strong>in</strong>t<strong>in</strong>g out a mistake <strong>in</strong> my orig<strong>in</strong>al solution to this problem. I believe the solution<br />

given here is correct, but the skeptical reader is encouraged to consult Rud<strong>in</strong>.<br />

4


1.1 1991 November 21 1 REAL ANALYSIS<br />

Then,<br />

<br />

ν(∪nEn) =<br />

<br />

n En<br />

<br />

f dµ =<br />

fχ n<br />

En dµ<br />

<br />

∞<br />

= fχEn dµ (∵ the En are disjo<strong>in</strong>t)<br />

n=1<br />

n=1<br />

∞<br />

<br />

= fχEn dµ (∵ fχEn ≥ 0, n = 1, 2, . . .)<br />

=<br />

∞<br />

ν(En).<br />

n=1<br />

The penultimate equality follows from the monotone convergence theorem applied to the sequence <strong>of</strong> non-negative<br />

measurable functions gm = m fχEn n=1 (m = 1, 2, . . .). (See also: April ’98, problem A.3.) ⊓⊔<br />

5. Suppose f is a bounded, real valued function on [0, 1]. Show that f is Lebesgue measurable if <strong>and</strong> only if<br />

<br />

<br />

sup ψ dm = <strong>in</strong>f φ dm<br />

where m is Lebesgue measure on [0, 1], <strong>and</strong> ψ <strong>and</strong> φ range over all simple functions, ψ ≤ f ≤ φ.<br />

Solution: This is proposition 4.3 <strong>of</strong> Royden, 3ed. [6].<br />

6. 6 If f is Lebesgue <strong>in</strong>tegrable on [0, 1] <strong>and</strong> ɛ > 0, show that there is δ > 0 such that for all measurable sets E ⊂ [0, 1]<br />

with m(E) < δ, <br />

<br />

<br />

f dm<br />

< ɛ.<br />

E<br />

Solution: This problem appears so <strong>of</strong>ten, I th<strong>in</strong>k it’s worth giv<strong>in</strong>g two different pro<strong>of</strong>s. The first relies on the<br />

frequently useful technique, employed <strong>in</strong> problem 3, <strong>in</strong> which the doma<strong>in</strong> is written as a union <strong>of</strong> the nested sets<br />

An = {x ∈ X : 1/n ≤ |f(x)| < n}. The second is a shorter pro<strong>of</strong>, but it relies on a result about absolute<br />

cont<strong>in</strong>uity <strong>of</strong> measures, which is almost equivalent to the orig<strong>in</strong>al problem statement. I recommend that you learn<br />

the first pro<strong>of</strong>. The second pro<strong>of</strong> is also worth study<strong>in</strong>g, however, as it connects this result to the analogous result<br />

about absolutely cont<strong>in</strong>uous measures.<br />

Pro<strong>of</strong> 1: Let An, n = 1, 2, . . . be the sequence <strong>of</strong> measurable sets def<strong>in</strong>ed <strong>in</strong> problem 3. That is,<br />

Here, X = [0, 1]. As we saw <strong>in</strong> problem 3,<br />

<br />

lim<br />

n→∞<br />

An = {x ∈ X : 1/n ≤ |f(x)| < n}.<br />

An<br />

6 See also: April ’92 (4), November ’97 (6), April ’03 (4).<br />

<br />

|f| dm =<br />

5<br />

A<br />

<br />

|f| dm =<br />

X<br />

|f| dm.


1.1 1991 November 21 1 REAL ANALYSIS<br />

Let n > 0 be such that <br />

X\An<br />

|f| dm < ɛ/2.<br />

Def<strong>in</strong>e δ = (2n) −1 ɛ, <strong>and</strong> suppose E ⊂ [0, 1] is a measurable set with mE < δ. We must show | <br />

<br />

<br />

<br />

<br />

E<br />

<br />

<br />

f dm<br />

≤<br />

<br />

<br />

=<br />

<br />

≤<br />

E<br />

|f| dm<br />

(X\An)∩E<br />

X\An<br />

<br />

|f| dm +<br />

<br />

|f| dm +<br />

< ɛ/2 + n m(An ∩ E)<br />

< ɛ/2 + n ɛ<br />

= ɛ.<br />

2n<br />

An∩E<br />

An∩E<br />

|f| dm<br />

|f| dm<br />

E<br />

f dm| < ɛ.<br />

The penultimate <strong>in</strong>equality holds because |f| < n on An. ⊓⊔<br />

Pro<strong>of</strong> 2: 7 This pro<strong>of</strong> relies on the follow<strong>in</strong>g lemma about absolute cont<strong>in</strong>uity <strong>of</strong> measures:<br />

Lemma 1.1 Let ν be a f<strong>in</strong>ite signed measure <strong>and</strong> µ a positive measure on a measurable space (X, M). Then ν ≪ µ<br />

if <strong>and</strong> only if for every ɛ > 0 there is a δ > 0 such that |ν(E)| < ɛ whenever µ(E) < δ.<br />

The signed measure def<strong>in</strong>ed by<br />

<br />

ν(E) =<br />

is f<strong>in</strong>ite iff 8 f ∈ L 1 (µ). It is also clearly absolutely cont<strong>in</strong>uous with respect to µ. Therefore, lemma 1.1 can be<br />

applied to the real <strong>and</strong> imag<strong>in</strong>ary parts <strong>of</strong> any complex-valued f ∈ L1 (µ). It follows that, for every ɛ > 0, there is<br />

a δ > 0 such that<br />

<br />

<br />

|ν(E)| = <br />

<br />

<br />

<br />

f dµ <br />

< ɛ, whenever µ(E) < δ.<br />

E<br />

7. 9 Suppose f is a bounded, real valued, measurable function on [0, 1] such that x n f dm = 0 for n = 0, 1, 2, . . .,<br />

with m Lebesgue measure. Show that f(x) = 0 a.e.<br />

Solution: Fix an arbitrary cont<strong>in</strong>uous function on [0, 1], say, φ ∈ C[0, 1]. By the Stone-Weierstrass theorem, there<br />

is a sequence {pn} <strong>of</strong> polynomials such that φ − pn∞ → 0 as n → ∞. Then, s<strong>in</strong>ce all functions <strong>in</strong>volved are<br />

7 See also Foll<strong>and</strong> [4], page 89.<br />

8 For what follows we only need that ν is f<strong>in</strong>ite if f is <strong>in</strong>tegrable, but the converse is also true.<br />

9 This question appears very <strong>of</strong>ten <strong>in</strong> vary<strong>in</strong>g forms <strong>of</strong> difficulty. cf. November ’92 (7b, very easy version), November ’96 (B2, very easy),<br />

November ’91 (this question, easy), April ’92 (6, moderate), November ’95 (6, hard–impossible?). I have yet to solve the November ’95 version.<br />

One attempted solution (which I th<strong>in</strong>k is the one given <strong>in</strong> the black notebook), seems to assume f ∈ L 1 , but that assumption makes the problem<br />

even easier than the others.<br />

6<br />

E<br />

f dµ<br />

⊓⊔


1.1 1991 November 21 1 REAL ANALYSIS<br />

<strong>in</strong>tegrable,<br />

<br />

<br />

<br />

<br />

<br />

<br />

fφ<br />

=<br />

<br />

<br />

<br />

<br />

<br />

<br />

f(φ − pn + pn) <br />

<br />

<br />

≤<br />

<br />

<br />

|f||φ − pn| + <br />

<br />

<br />

<br />

fpn<br />

<br />

<br />

<br />

≤ f1φ − pn∞ + <br />

<br />

<br />

<br />

fpn<br />

<br />

= f1φ − pn∞. (3)<br />

The last equality holds s<strong>in</strong>ce x n f = 0 for all n = 0, 1, 2, . . ., which implies that fpn = 0 for all polynomials<br />

pn. F<strong>in</strong>ally, note that f1 < ∞, s<strong>in</strong>ce f is bounded <strong>and</strong> Lebesgue measurable on the bounded <strong>in</strong>terval [0, 1].<br />

Therefore, the right-h<strong>and</strong> side <strong>of</strong> (3) tends to zero as n tends to <strong>in</strong>f<strong>in</strong>ity. S<strong>in</strong>ce the left-h<strong>and</strong> side <strong>of</strong> (3) is <strong>in</strong>dependent<br />

<strong>of</strong> n, we have thus shown that fφ = 0 for any φ ∈ C[0, 1].<br />

Now, s<strong>in</strong>ce C[0, 1] is dense <strong>in</strong> L1 [0, 1], let {φn} ⊂ C[0, 1] satisfy φn − f1 → 0 as n → ∞. Then<br />

<br />

0 ≤ f 2 <br />

<br />

= <br />

<br />

<br />

<br />

f(f − φn + φn) <br />

≤<br />

<br />

<br />

<br />

|f||f − φn| + <br />

<br />

<br />

<br />

fφn<br />

.<br />

The second term on the right is zero by what we proved above. Therefore, if M is the bound on |f|, we have<br />

0 ≤ f 2 ≤ Mf − φn1 → 0. As f 2 is <strong>in</strong>dependent <strong>of</strong> n, we have f 2 = 0. S<strong>in</strong>ce f 2 ≥ 0, this implies<br />

f 2 = 0 a.e., hence f = 0 a.e. ⊓⊔<br />

Alternative Solution: Qu<strong>in</strong>n Culver suggests shorten<strong>in</strong>g the pro<strong>of</strong> by us<strong>in</strong>g the fact that polynomials are dense <strong>in</strong><br />

L 1 [0, 1]. Simply start from the l<strong>in</strong>e, “Now, s<strong>in</strong>ce C[0, 1] is dense <strong>in</strong> L 1 [0, 1], let {φn} ⊂ C[0, 1] satisfy...” but<br />

<strong>in</strong>stead write, “S<strong>in</strong>ce Pol[0, 1] is dense <strong>in</strong> L 1 [0, 1], let {φn} ⊂ Pol[0, 1] satisfy...” This is a nice observation <strong>and</strong><br />

disposes <strong>of</strong> the problem quickly <strong>and</strong> efficiently. However, I have left the orig<strong>in</strong>al, somewhat clumsy pro<strong>of</strong> <strong>in</strong>tact<br />

because it provides a nice demonstration <strong>of</strong> the Stone-Weierstrass theorem (which appears on the exam syllabus),<br />

<strong>and</strong> because everyone should know how to apply this fundamental theorem to problems <strong>of</strong> this sort.<br />

8. 10 If µ <strong>and</strong> ν are f<strong>in</strong>ite measures on the measurable space (X, Σ), show that there is a nonnegative measurable<br />

function f on X such that for all E <strong>in</strong> Σ,<br />

<br />

<br />

(1 − f) dµ = f dν. (4)<br />

E<br />

Solution: There’s an assumption miss<strong>in</strong>g here: µ <strong>and</strong> ν must be positive measures. 11 In fact, one can prove the<br />

result is false without this assumption. So assume µ <strong>and</strong> ν are f<strong>in</strong>ite positive measures on the measurable space<br />

(X, Σ). By the l<strong>in</strong>earity property <strong>of</strong> the <strong>in</strong>tegral, <strong>and</strong> s<strong>in</strong>ce µ(E) = <br />

dµ, we have<br />

E<br />

<br />

<br />

<br />

(1 − f) dµ = dµ − f dµ = µ(E) − f dµ.<br />

E<br />

E E<br />

E<br />

Therefore, (4) is equivalent to<br />

<br />

µ(E) =<br />

E<br />

E<br />

<br />

f dµ + f dν = f d(µ + ν) (∀ E ∈ Σ) (5)<br />

E<br />

E<br />

10 See also: November ’97 (7).<br />

11 Note that a measure µ is called “positive” when it is, <strong>in</strong> fact, nonnegative; that is, µE ≥ 0 for all E ∈ Σ.<br />

7


1.1 1991 November 21 1 REAL ANALYSIS<br />

so this is what we will prove. The Radon-Nikodym theorem (A.12) says, if λ ≪ m are σ-f<strong>in</strong>ite positive measures<br />

on a σ-algebra Σ, then there is a unique g ∈ L1 (dm) such that<br />

<br />

λ(E) = g dm, ∀E ∈ Σ.<br />

E<br />

In the present case, µ ≪ µ + ν (s<strong>in</strong>ce the measures are positive), so the theorem provides an f ∈ L1 (µ + ν) such<br />

that<br />

<br />

µ(E) = f d(µ + ν) ∀ E ∈ Σ,<br />

E<br />

which proves (5). ⊓⊔<br />

9. 12 If f <strong>and</strong> g are <strong>in</strong>tegrable functions on (X, S, µ) <strong>and</strong> (Y, T , ν), respectively, <strong>and</strong> F (x, y) = f(x) g(y), show that<br />

F is <strong>in</strong>tegrable on X × Y <strong>and</strong> <br />

<br />

F d(µ × ν) = f dµ g dν.<br />

Solution: 13 To show F (x, y) = f(x)g(y) is <strong>in</strong>tegrable, an important (but <strong>of</strong>ten overlooked) first step is to prove<br />

that F (x, y) = f(x)g(y) is (S ⊗ T )-measurable. Def<strong>in</strong>e Ψ : X × Y → R × R by Ψ(x, y) = (f(x), g(y)), <strong>and</strong> let<br />

Φ : R × R → R be the cont<strong>in</strong>uous function Φ(s, t) = st. Then,<br />

F (x, y) = f(x)g(y) = (Φ ◦ Ψ)(x, y).<br />

Theorem A.2 states that a cont<strong>in</strong>uous function <strong>of</strong> a measurable function is measurable. Therefore, if we can show<br />

that Ψ(x, y) is an (S ⊗T )-measurable function from X ×Y <strong>in</strong>to R×R, then it will follow that F (x, y) is (S ⊗T )measurable.<br />

To show Ψ is measurable, let R be an open rectangle <strong>in</strong> R × R. Then R = A × B for some open sets<br />

A <strong>and</strong> B <strong>in</strong> R, <strong>and</strong><br />

Ψ −1 (R) = Ψ −1 (A × B)<br />

= {(x, y) : f(x) ∈ A, g(y) ∈ B}<br />

= {(x, y) : f(x) ∈ A} ∩ {(x, y) : g(y) ∈ B}<br />

= (f −1 (A) × Y ) ∩ (X × g −1 (B))<br />

= f −1 (A) × g −1 (B).<br />

Now, f −1 (A) ∈ S <strong>and</strong> g −1 (B) ∈ T , s<strong>in</strong>ce f <strong>and</strong> g are S- <strong>and</strong> T -measurable, resp. Therefore, Ψ −1 (R) ∈ S ⊗ T ,<br />

which proves the claim.<br />

Now that we know F (x, y) = f(x)g(y) is (S ⊗T )-measurable, we can apply part (b) <strong>of</strong> the Fub<strong>in</strong>i-Tonelli theorem<br />

(A.13) to prove that F (x, y) = f(x)g(y) is <strong>in</strong>tegrable if one <strong>of</strong> the iterated <strong>in</strong>tegrals <strong>of</strong> |F (x, y)| is f<strong>in</strong>ite. Indeed,<br />

<br />

<br />

|f(x)g(y)| dν(y) dµ(x) = |f(x)||g(y)| dν(y) dµ(x)<br />

X Y<br />

X<br />

<br />

Y<br />

<br />

<br />

= |f(x)| |g(y)| dν(y) dµ(x)<br />

X<br />

<br />

Y<br />

<br />

= |f(x)| dµ(x) |g(y)| dν(y) < ∞,<br />

12 See also: November ’97 (2), <strong>and</strong> others.<br />

13 I’m not sure if the claim is true unless the measure spaces are σ-f<strong>in</strong>ite, so I’ll assume all measure spaces σ-f<strong>in</strong>ite.<br />

In my op<strong>in</strong>ion, the most useful version <strong>of</strong> the Fub<strong>in</strong>i-Tonelli theorem is the one <strong>in</strong> Rud<strong>in</strong> [8], which assumes σ-f<strong>in</strong>ite measure spaces. There is a<br />

version appear<strong>in</strong>g <strong>in</strong> Royden [6] that does not require σ-f<strong>in</strong>iteness. Instead it beg<strong>in</strong>s with the assumption that f is <strong>in</strong>tegrable. To me, the theorem<br />

<strong>in</strong> Rud<strong>in</strong> is much easier to apply. All you need is a function that is measurable with respect to the product σ-algebra S ⊗ T , <strong>and</strong> from there, <strong>in</strong> a<br />

s<strong>in</strong>gle theorem, you get everyth<strong>in</strong>g you need to answer any <strong>of</strong> the st<strong>and</strong>ard questions about <strong>in</strong>tegration with respect to a product measure.<br />

8<br />

X<br />

Y


1.1 1991 November 21 1 REAL ANALYSIS<br />

which holds s<strong>in</strong>ce f ∈ L 1 (µ) <strong>and</strong> g ∈ L 1 (ν). The Fub<strong>in</strong>i-Tonelli theorem then implies that F (x, y) ∈ L 1 (µ × ν).<br />

F<strong>in</strong>ally, we must prove that F d(µ×ν) = f dµ g dν. S<strong>in</strong>ce F (x, y) ∈ L1 (µ×ν), part (c) <strong>of</strong> the Fub<strong>in</strong>i-Tonelli<br />

theorem asserts that φ(x) = <br />

Y F (x, y) dν(y) is def<strong>in</strong>ed almost everywhere, belongs to L1 (µ), <strong>and</strong>, moreover,<br />

<br />

<br />

F d(µ × ν) = F (x, y) dν(y) dµ(x).<br />

Therefore,<br />

<br />

X×Y<br />

X×Y<br />

<br />

F d(µ × ν) =<br />

<br />

=<br />

<br />

=<br />

X<br />

X<br />

X<br />

X<br />

9<br />

Y<br />

<br />

f(x)g(y) dν(y) dµ(x)<br />

Y<br />

<br />

f(x) g(y) dν(y) dµ(x)<br />

Y <br />

f(x) dµ(x) g(y) dν(y).<br />

Y<br />

⊓⊔


1.2 1994 November 16 1 REAL ANALYSIS<br />

1.2 1994 November 16<br />

Masters students: Do any 5 problems.<br />

Ph.D. students: Do any 6 problems.<br />

1. Let E be a normed l<strong>in</strong>ear space. Show that E is complete if <strong>and</strong> only if, whenever ∞ 1 xn < ∞, then ∞ converges to an s ∈ E.<br />

Solution: Suppose E is complete. Let {xn} ⊂ E be absolutely convergent; i.e., xn < ∞. We must<br />

∞<br />

xn := lim<br />

n=1<br />

Let SN = N<br />

n=1 xn. Then, for any j ∈ N,<br />

SN+j − SN =<br />

<br />

<br />

<br />

<br />

<br />

N→∞<br />

n=1<br />

N+j <br />

n=N+1<br />

1 xn<br />

N<br />

xn = s ∈ E. (6)<br />

xn<br />

<br />

<br />

<br />

<br />

≤<br />

N+j <br />

n=N+1<br />

xn → 0<br />

as N → ∞, s<strong>in</strong>ce xn < ∞. Therefore, {SN } is a Cauchy sequence. S<strong>in</strong>ce E is complete, there is an s ∈ E<br />

such that ∞ n=1 xn = lim<br />

N→∞ SN = s.<br />

Conversely, suppose whenever ∞ 1 xn < ∞, then ∞ 1 xn converges to an s ∈ E. Let {yn} ⊂ E be a Cauchy<br />

sequence. That is, yn − ym → 0 as n, m → ∞. Let n1 < n2 < · · · be a subsequence such that<br />

Next observe, for k > 1,<br />

<strong>and</strong><br />

ynk<br />

n, m ≥ nj ⇒ yn − ym < 2 −j .<br />

k−1 <br />

= yn1 + (yn2 − yn1 ) + (yn3 − yn2 ) + · · · + (ynk − ynk−1 ) = yn1 +<br />

By hypothesis, this implies that<br />

ynk<br />

∞<br />

j=1<br />

ynj+1<br />

− yn1 =<br />

− ynj <<br />

k−1 <br />

j=1<br />

(ynj+1<br />

∞<br />

2 −j = 1<br />

j=1<br />

− ynj ) → s ∈ E,<br />

j=1<br />

(ynj+1<br />

− ynj ),<br />

as k → ∞. We have thus found a subsequence {ynk } ⊆ {yn} hav<strong>in</strong>g a limit <strong>in</strong> E. F<strong>in</strong>ally, s<strong>in</strong>ce {yn} is Cauchy,<br />

it is quite easy to verify that {yn} must converge to the same limit. This proves that every Cauchy sequence <strong>in</strong> E<br />

converges to a po<strong>in</strong>t <strong>in</strong> E. ⊓⊔<br />

2. Let fn be a sequence <strong>of</strong> real cont<strong>in</strong>uous functions on a compact Hausdorff space X. Show that if f1 ≥ f2 ≥ f3 ≥<br />

· · · , <strong>and</strong> fn(x) → 0 for all x ∈ X, then fn → 0 uniformly.<br />

10


1.2 1994 November 16 1 REAL ANALYSIS<br />

3. Let f be <strong>in</strong>tegrable on the real l<strong>in</strong>e with respect to Lebesgue measure. Evaluate lim<br />

Justify all steps.<br />

n→∞<br />

<br />

∞<br />

x<br />

f(x − n)<br />

−∞ 1+|x| dx.<br />

Solution: Fix n > 0. Consider the change <strong>of</strong> variables, y = x − n. Then dy = dx <strong>and</strong> x = y + n, so<br />

∞<br />

−∞<br />

Note that, when y ≥ −n,<br />

x<br />

f(x − n) dx =<br />

1 + |x|<br />

for all y ≥ −n. Def<strong>in</strong>e the function 14<br />

=<br />

∞<br />

−∞<br />

∞<br />

−n<br />

y + n<br />

f(y)<br />

1 + |y + n| dy<br />

y + n<br />

f(y) dy +<br />

1 + y + n<br />

−n<br />

−∞<br />

y+n<br />

1+y+n ∈ [0, 1), <strong>and</strong> <strong>in</strong>creases to 1 as n tends to <strong>in</strong>f<strong>in</strong>ity. Thus,<br />

0 ≤ |f(y)|<br />

gn(y) = f(y)<br />

y + n<br />

≤ |f(y)|,<br />

1 + y + n<br />

y + n<br />

1 + y + n 1 [−n,∞)(y).<br />

Then |gn| ≤ |f| <strong>and</strong> lim<br />

n→∞ gn = f. Therefore, by the dom<strong>in</strong>ated convergence theorem,<br />

∞<br />

lim<br />

n→∞<br />

−n<br />

y + n<br />

f(y)<br />

1 + y + n<br />

Next, consider the second term <strong>in</strong> (7). Def<strong>in</strong>e the function<br />

It is not hard to check that<br />

hn(y) = f(y)<br />

∞<br />

∞<br />

dy = lim<br />

n→∞<br />

gn(y) dy =<br />

−∞<br />

f(y) dy.<br />

−∞<br />

y + n<br />

1 − (y + n) 1 (−∞,−n](y).<br />

|y + n|<br />

|1 − (y + n)| 1 (−∞,−n](y) ∈ [0, 1),<br />

from which it follows that |hn| ≤ |f|. Also, it is clear that, for all y,<br />

lim<br />

n→∞ hn(y)<br />

y + n<br />

= f(y) lim<br />

n→∞ 1 − (y + n) 1 (−∞,−n](y) = 0.<br />

Therefore, the dom<strong>in</strong>ated convergence theorem implies that<br />

−n<br />

lim<br />

n→∞<br />

−∞<br />

y + n<br />

f(y)<br />

dy = 0.<br />

1 − (y + n)<br />

y + n<br />

f(y)<br />

dy. (7)<br />

1 − (y + n)<br />

<br />

∞<br />

x<br />

Comb<strong>in</strong><strong>in</strong>g the two results above, we see that lim f(x − n)<br />

n→∞ −∞ 1+|x| dx = ∞<br />

f(x) dx. ⊓⊔<br />

−∞<br />

Remark. Intuitively, this is the result we expect because the translation f(x − n) = Tnf(x) is merely shift<strong>in</strong>g the<br />

support <strong>of</strong> f to the right tail <strong>of</strong> the measure dµ := x<br />

1+|x| dx, <strong>and</strong> <strong>in</strong> the tail this measure looks like dx.<br />

14 Here 1A(x) denotes the <strong>in</strong>dicator function <strong>of</strong> the set A, which is 1 if x ∈ A <strong>and</strong> 0 if x /∈ A.<br />

11


1.3 1998 April 3 1 REAL ANALYSIS<br />

1.3 1998 April 3<br />

Instructions Do at least four problems <strong>in</strong> Part A, <strong>and</strong> at least two problems <strong>in</strong> Part B.<br />

PART A<br />

1. Let {xn} ∞ n=1 be a bounded sequence <strong>of</strong> real numbers, <strong>and</strong> for each positive n def<strong>in</strong>e<br />

(a) Expla<strong>in</strong> why the limit ℓ = limn→∞ ˆxn exists.<br />

ˆxn = sup xk<br />

k≥n<br />

(b) Prove that, for any ɛ > 0 <strong>and</strong> positive <strong>in</strong>teger N, there exists an <strong>in</strong>teger k such that k ≥ N <strong>and</strong> |xk − ℓ| < ɛ.<br />

2. Let C be a collection <strong>of</strong> subsets <strong>of</strong> the real l<strong>in</strong>e R, <strong>and</strong> def<strong>in</strong>e<br />

Aσ(C) = {A : C ⊂ A <strong>and</strong> A is a σ-algebra <strong>of</strong> subsets <strong>of</strong> R}.<br />

(a) Prove that Aσ(C) is a σ-algebra, that C ⊂ Aσ(C), <strong>and</strong> that Aσ(C) ⊂ A for any other σ-algebra A conta<strong>in</strong><strong>in</strong>g<br />

all the sets <strong>of</strong> C.<br />

(b) Let O be the collection <strong>of</strong> all f<strong>in</strong>ite open <strong>in</strong>tervals <strong>in</strong> R, <strong>and</strong> F the collection <strong>of</strong> all f<strong>in</strong>ite closed <strong>in</strong>tervals <strong>in</strong> R.<br />

Show that<br />

Aσ(O) = Aσ(F ).<br />

3. Let (X, A, µ) be a measure space, <strong>and</strong> suppose X = ∪nXn, where {Xn} ∞ n=1 is a pairwise disjo<strong>in</strong>t collection <strong>of</strong><br />

measurable subsets <strong>of</strong> X. Use the monotone convergence theorem <strong>and</strong> l<strong>in</strong>earity <strong>of</strong> the <strong>in</strong>tegral to prove that, if f is<br />

a non-negative measurable real-valued function on X,<br />

<br />

f dµ = <br />

<br />

f dµ.<br />

X<br />

n<br />

Solution: 15 Def<strong>in</strong>e fn = n k=1 fχXk = fχ∪n 1<br />

Xk . Then it is clear that the hypotheses <strong>of</strong> the monotone convergence<br />

theorem are satisfied. That is, for all x ∈ X,<br />

(i) 0 ≤ f1(x) ≤ f2(x) ≤ · · · ≤ f(x), <strong>and</strong><br />

(ii) limn→∞ fn(x) = f(x)χX(x) = f(x).<br />

<br />

15Note that the hypotheses imply ν(E) = E f dµ is a measure (problem 4, Nov. ’91), from which the desired conclusion immediately follows.<br />

Of course, this does not answer the question as stated, s<strong>in</strong>ce the exam<strong>in</strong>ers specifically require the use <strong>of</strong> the MCT <strong>and</strong> l<strong>in</strong>earity <strong>of</strong> the <strong>in</strong>tegral.<br />

12<br />

Xn


1.3 1998 April 3 1 REAL ANALYSIS<br />

Therefore,<br />

∞<br />

<br />

k=1<br />

Xk<br />

f dµ = lim<br />

n<br />

<br />

n→∞<br />

k=1<br />

X<br />

fχXk dµ<br />

n<br />

= lim fχXk dµ<br />

n→∞<br />

X<br />

k=1<br />

<br />

(by l<strong>in</strong>earity <strong>of</strong> the <strong>in</strong>tegral)<br />

= lim<br />

n→∞<br />

<br />

fn dµ<br />

X<br />

(by def<strong>in</strong>ition <strong>of</strong> fn)<br />

= lim<br />

X<br />

n→∞ fn dµ<br />

<br />

(by the monotone convergence theorem)<br />

= f dµ.<br />

X<br />

4. Us<strong>in</strong>g the Fub<strong>in</strong>i/Tonelli theorems to justify all steps, evaluate the <strong>in</strong>tegral<br />

1 1<br />

0<br />

y<br />

x −3/2 cos<br />

<br />

πy<br />

<br />

dx dy.<br />

2x<br />

Solution: By Tonelli’s theorem, if f(x, y) ≥ 0 is measurable <strong>and</strong> one <strong>of</strong> the iterated <strong>in</strong>tegrals f(x, y) dx dy or<br />

f(x, y) dy dx exists, then they both exist <strong>and</strong> are equal. Moreover, if one <strong>of</strong> the iterated <strong>in</strong>tegrals is f<strong>in</strong>ite, then<br />

f(x, y) ∈ L 1 (dx, dy). Fub<strong>in</strong>i’s theorem states: if f(x, y) ∈ L 1 (dx, dy), then the iterated <strong>in</strong>tegrals exist <strong>and</strong> are<br />

equal.<br />

Now let g(x, y) = x −3/2 cos(πy/2x), <strong>and</strong> apply the Tonelli theorem to the non-negative measurable function<br />

|g(x, y)| as follows:<br />

1 x<br />

0<br />

0<br />

|g(x, y)| dy dx =<br />

1 x<br />

0<br />

0<br />

|x| −3/2<br />

<br />

πy<br />

<br />

<br />

cos dy dx ≤<br />

2x<br />

1 x<br />

0<br />

0<br />

x −3/2 · 1 dy dx =<br />

1<br />

0<br />

x −1/2 dx = 2.<br />

Thus one <strong>of</strong> the iterated <strong>in</strong>tegrals <strong>of</strong> |g(x, y)| is f<strong>in</strong>ite which, by the Tonelli theorem, implies g(x, y) ∈ L 1 (dx, dy).<br />

Therefore, the Fub<strong>in</strong>i theorem applies to g(x, y), <strong>and</strong> gives the first <strong>of</strong> the follow<strong>in</strong>g equalities:<br />

1 1<br />

0<br />

y<br />

x −3/2 cos<br />

<br />

πy<br />

<br />

dx dy =<br />

2x<br />

=<br />

=<br />

1 x<br />

0<br />

1<br />

0<br />

1<br />

0<br />

= 2<br />

π<br />

= 4<br />

π .<br />

13<br />

0<br />

x −3/2 cos<br />

x −3/2 · 2x<br />

π<br />

2<br />

π x−1/2 dx<br />

<br />

2x 1/2 x=1<br />

x=0<br />

<br />

πy<br />

<br />

dy dx<br />

2x<br />

<br />

πy<br />

y=x s<strong>in</strong><br />

dx<br />

2x y=0<br />

⊓⊔<br />

⊓⊔


1.3 1998 April 3 1 REAL ANALYSIS<br />

5. Let I be the <strong>in</strong>terval [0, 1], <strong>and</strong> let C(I), C(I × I) denote the spaces <strong>of</strong> real valued cont<strong>in</strong>uous functions on I <strong>and</strong><br />

I × I, respectively, with the usual supremum norm on these spaces. Show that the collection <strong>of</strong> f<strong>in</strong>ite sums <strong>of</strong> the<br />

form<br />

f(x, y) = <br />

φi(x)ψi(y),<br />

where φi, ψi ∈ C(I) for each i, is dense <strong>in</strong> C(I × I).<br />

6. Let m be Lebesgue measure on the real l<strong>in</strong>e R, <strong>and</strong> for each Lebesgue measurable subset E <strong>of</strong> R def<strong>in</strong>e<br />

<br />

1<br />

µ(E) = dm(x).<br />

1 + x2 E<br />

Show that m is absolutely cont<strong>in</strong>uous with respect to µ, <strong>and</strong> compute the Radon-Nikodym derivative dm/dµ.<br />

i<br />

Solution: Obviously both measures are non-negative. We must first prove m ≪ µ. To this end, suppose m(E) ><br />

0, where E ∈ M, the σ-algebra <strong>of</strong> Lebesgue measurable sets. Then, if we can show µ(E) > 0, this will establish<br />

that the implication µ(E) = 0 ⇒ m(E) = 0 holds for all E ∈ M; i.e., m ≪ µ.<br />

For n = 1, 2, . . ., def<strong>in</strong>e<br />

An =<br />

<br />

x ∈ R :<br />

1<br />

n + 1 <<br />

Then Ai ∩ Aj = ∅ for all i = j <strong>in</strong> N, <strong>and</strong>, for all n = 1, 2, . . .,<br />

µ(An) ≥ 1<br />

n + 1 m(An).<br />

1<br />

<br />

1<br />

≤ .<br />

1 + x2 n<br />

Also, R = ∪An, s<strong>in</strong>ce 0 < 1<br />

1+x2 ≤ 1 holds for all x ∈ R. Therefore,<br />

µ(E) = µ(E ∩ (∪nAn)) = µ(∪n(An ∩ E)) = <br />

µ(An ∩ E).<br />

The last equality might need a bit <strong>of</strong> justification: S<strong>in</strong>ce f(x) = 1<br />

1+x 2 is cont<strong>in</strong>uous, hence measurable, the sets<br />

{An} are measurable. Therefore, the last equality holds by countable additivity <strong>of</strong> disjo<strong>in</strong>t measurable sets.<br />

Now note that m(E) = m(An∩E) > 0 implies the existence <strong>of</strong> an n ∈ N such that m(An∩E) > 0. Therefore,<br />

µ(E) ≥ µ(An ∩ E) ≥ 1<br />

n + 1 m(An ∩ E) > 0,<br />

which proves that m ≪ µ. By the Radon-Nikodym theorem (A.1), there is a unique h ∈ L1 (µ) such that<br />

<br />

<br />

m(E) = h dµ, <strong>and</strong> f dm = fh dµ ∀ f ∈ L 1 (m).<br />

In particular, if E ∈ M <strong>and</strong> f(x) = 1<br />

1+x2 χE, then<br />

<br />

<br />

1<br />

h(x)<br />

µ(E) = dm(x) = dµ(x).<br />

E 1 + x2 E 1 + x2 That is, h(x)<br />

dµ = E E 1+x2 dµ(x) holds for all measurable sets E, which implies16 that, h(x)<br />

1+x2 = 1 holds for<br />

µ-almost every x ∈ R. Therefore,<br />

dm<br />

dµ (x) = h(x) = 1 + x2 .<br />

One f<strong>in</strong>al note: h is uniquely def<strong>in</strong>ed only up to an equivalence class <strong>of</strong> functions that are equal to 1+x 2 , µ-a.e. ⊓⊔<br />

<br />

16Recall the st<strong>and</strong>ard result: if f <strong>and</strong> g are <strong>in</strong>tegrable functions such that E f = E g holds for all measurable sets E, then f = g, µ-a.e.<br />

This is an exam problem, but I can’t remember on which exam it appears. When I come across it aga<strong>in</strong> I’ll put a cross reference here.<br />

14<br />

n


1.3 1998 April 3 1 REAL ANALYSIS<br />

PART B<br />

7. Let φ(x, y) = x 2 y be def<strong>in</strong>ed on the square S = [0, 1] × [0, 1] <strong>in</strong> the plane, <strong>and</strong> let m be two-dimensional Lebesgue<br />

measure on S. Given a Borel subset E <strong>of</strong> the real l<strong>in</strong>e R, def<strong>in</strong>e<br />

(a) Show that µ is a Borel measure on R.<br />

µ(E) = m(φ −1 (E)).<br />

(b) Let χE denote the characteristic function <strong>of</strong> the set E. Show that<br />

<br />

χE ◦ φ dm.<br />

χE dµ =<br />

R<br />

(c) Evaluate the <strong>in</strong>tegral ∞<br />

S<br />

t<br />

−∞<br />

2 dµ(t).<br />

8. Let f be a real valued <strong>and</strong> <strong>in</strong>creas<strong>in</strong>g function on the real l<strong>in</strong>e R, such that f(−∞) = 0 <strong>and</strong> f(∞) = 1. Prove that<br />

f is absolutely cont<strong>in</strong>uous on every closed f<strong>in</strong>ite <strong>in</strong>terval if <strong>and</strong> only if<br />

<br />

f ′ dm = 1.<br />

R<br />

Solution: 17 First note that f is <strong>in</strong>creas<strong>in</strong>g, so f ′ exists for a.e. x ∈ R, <strong>and</strong> f ′ (x) ≥ 0 wherever f ′ exists. Also, f ′<br />

is measurable. To see this, def<strong>in</strong>e<br />

g(x) = lim sup [f(x + 1/n) − f(x)] n.<br />

n→∞<br />

As a lim sup <strong>of</strong> a sequence <strong>of</strong> measurable functions, g is measurable (Rud<strong>in</strong> [8], theorem 1.14?). Let E be the set<br />

on which f ′ exists. Then m(R \ E) = 0, <strong>and</strong> f ′ = g on E (by the def<strong>in</strong>ition <strong>of</strong> derivative), so f ′ is measurable.<br />

(⇐) Suppose <br />

R f ′ dm = 1. We must show f ∈ AC[a, b] for all −∞ < a < b < ∞. First, check that<br />

f ′ ∈ L1 (R), s<strong>in</strong>ce<br />

<br />

1 =<br />

R<br />

f ′ <br />

dm =<br />

R\E<br />

<strong>and</strong>, s<strong>in</strong>ce f is <strong>in</strong>creas<strong>in</strong>g, f ′ ≥ 0 on E, so<br />

<br />

|f ′ <br />

|dm = |f ′ <br />

|dm +<br />

R<br />

R\E<br />

f ′ <br />

dm +<br />

E<br />

E<br />

|f ′ <br />

|dm =<br />

f ′ <br />

dm =<br />

E<br />

E<br />

|f ′ <br />

|dm =<br />

f ′ dm,<br />

E<br />

f ′ dm = 1.<br />

Thus, f ′ ∈ L 1 (R) as claimed. A couple <strong>of</strong> lemmas will be needed to complete the ⇐ direction <strong>of</strong> the pro<strong>of</strong>. The<br />

first is proved <strong>in</strong> the appendix (sec. A), while the second can be found <strong>in</strong> Royden [6] on page 100.<br />

Lemma 1.2 Let f : R → R be a function. If f is differentiable on [a, b], f ′ ∈ L 1 ([a, b]), <strong>and</strong> x<br />

a f ′ (t)dt =<br />

f(x) − f(a) for a ≤ x ≤ b, then f ∈ AC[a, b].<br />

17 I have worked this problem a number <strong>of</strong> times, <strong>and</strong> what follows is the clearest <strong>and</strong> most <strong>in</strong>structive pro<strong>of</strong> I’ve come up with. It’s by no means<br />

the shortest, most elegant solution, <strong>and</strong> probably not the type <strong>of</strong> detailed answer one should give on an actual exam. However, some <strong>of</strong> the facts that<br />

I prove <strong>in</strong> detail have appeared as separate questions on other exams, so the pro<strong>of</strong>s are worth know<strong>in</strong>g.<br />

15


1.3 1998 April 3 1 REAL ANALYSIS<br />

The converse <strong>of</strong> this lemma is also true. 18<br />

Lemma 1.3 If f : R → R is <strong>in</strong>creas<strong>in</strong>g <strong>and</strong> f ′ ∈ L 1 ([a, b]), then x<br />

a f ′ (t)dt ≤ f(x) − f(a).<br />

To f<strong>in</strong>ish the ⇐ direction <strong>of</strong> the pro<strong>of</strong>, by lemma 1.2, it suffices to show that <br />

R f ′ dm = 1 implies b<br />

a f ′ (t)dt =<br />

f(b) − f(a) holds for all −∞ < a < b < ∞. By lemma 1.3, we have b<br />

a f ′ (t)dt ≤ f(b) − f(a), so we need only<br />

show that strict <strong>in</strong>equality cannot hold. Suppose, by way <strong>of</strong> contradiction, that b<br />

a f ′ (t)dt < f(b) − f(a) holds for<br />

some −∞ < a < b < ∞. Then,<br />

<br />

1 =<br />

R<br />

f ′ dm =<br />

a<br />

−∞<br />

f ′ dm +<br />

b<br />

a<br />

f ′ dm +<br />

∞<br />

b<br />

f ′ dm<br />

< [f(a) − f(−∞)] + [f(b) − f(a)] + [f(∞) − f(b)]<br />

= f(∞) − f(−∞) = 1.<br />

This contradiction proves that <br />

R f ′ dm = 1 implies b<br />

a f ′ (t)dt = f(b) − f(a) holds for all −∞ < a < b < ∞, as<br />

desired.<br />

(⇒) Now assume f ∈ AC[a, b] for all −∞ < a < b < ∞. We must show <br />

f(∞) − f(−∞) = 1, so this is equivalent to show<strong>in</strong>g<br />

x<br />

lim f<br />

x→∞<br />

−x<br />

′ (t)dm(t) = lim [f(x) − f(−x)].<br />

x→∞<br />

R f ′ dm = 1. By assumption<br />

Let x ∈ R, x > 0, <strong>and</strong> f ∈ AC[−x, x]. Then we claim f(x) − f(−x) = x<br />

−x f ′ dm. Assum<strong>in</strong>g the claim is true<br />

(see Royden [6], p. 110 for the pro<strong>of</strong>), we have<br />

x<br />

1 = lim [f(x) − f(−x)] = lim<br />

x→∞ x→∞<br />

f ′ <br />

(t)dm(t) = f ′ dm.<br />

9. Let F be a cont<strong>in</strong>uous l<strong>in</strong>ear functional on the space L 1 [−1, 1], with the property that F (f) = 0 for all odd<br />

functions f <strong>in</strong> L 1 [−1, 1]. Show that there exists an even function φ such that<br />

F (f) =<br />

1<br />

−1<br />

−x<br />

f(x)φ(x) dx, for all f ∈ L 1 [−1, 1].<br />

[H<strong>in</strong>t: One possible approach is to use the fact that any function <strong>in</strong> L p [−1, 1] is the sum <strong>of</strong> an odd function <strong>and</strong> an<br />

even function.]<br />

Solution: S<strong>in</strong>ce F ∈ L 1 [−1, 1] ∗ , then by the Riesz representation theorem 19 there is a unique h ∈ L ∞ [−1, 1]<br />

such that<br />

F (f) =<br />

1<br />

−1<br />

f(x)h(x) dx (∀f ∈ L 1 [−1, 1])<br />

18 See Foll<strong>and</strong> [4] for a nice, concise treatment <strong>of</strong> the fundamental theorem <strong>of</strong> calculus for Lebesgue <strong>in</strong>tegration.<br />

19 See problem 3 <strong>of</strong> section 1.5.<br />

16<br />

R<br />

⊓⊔


1.3 1998 April 3 1 REAL ANALYSIS<br />

Now (us<strong>in</strong>g the h<strong>in</strong>t) write h = φ + ψ, where φ <strong>and</strong> ψ are the even <strong>and</strong> odd functions<br />

φ(x) =<br />

h(x) + h(−x)<br />

2<br />

<strong>and</strong> ψ(x) =<br />

h(x) − h(−x)<br />

.<br />

2<br />

Similarly, let f = fe + fo be the decomposition <strong>of</strong> f <strong>in</strong>to a sum <strong>of</strong> even <strong>and</strong> odd functions. Then, by l<strong>in</strong>earity <strong>of</strong><br />

F , <strong>and</strong> s<strong>in</strong>ce F (fo) = 0 by hypothesis,<br />

F (f) = F (fe) + F (fo) = F (fe) =<br />

1<br />

−1<br />

feh =<br />

1<br />

−1<br />

1<br />

feφ + feψ.<br />

−1<br />

Now note that feψ is an odd function (s<strong>in</strong>ce it’s an even times an odd), so 1<br />

−1 feψ = 0, s<strong>in</strong>ce [−1, 1] is symmetric.<br />

Similarly, 1<br />

−1 foφ = 0. Therefore,<br />

F (f) = F (fe) =<br />

1<br />

−1<br />

feφ =<br />

1<br />

−1<br />

1<br />

feφ + foφ =<br />

−1<br />

17<br />

1<br />

(fe + fo)φ =<br />

−1<br />

1<br />

−1<br />

fφ.<br />

⊓⊔


1.4 2000 November 17 1 REAL ANALYSIS<br />

1.4 2000 November 17<br />

Do as many problems as you can. Complete solutions to five problems would be considered a good performance.<br />

1. (a) 20 State the <strong>in</strong>verse function theorem.<br />

(b) Suppose L : R 3 → R 3 is an <strong>in</strong>vertible l<strong>in</strong>ear map <strong>and</strong> that g : R 3 → R 3 has cont<strong>in</strong>uous first order partial<br />

derivatives <strong>and</strong> satisfies g(x) ≤ Cx 2 for some constant C <strong>and</strong> all x ∈ R 3 . Here x denotes the usual<br />

Euclidean norm on R 3 . Prove that f(x) = L(x) + g(x) is locally <strong>in</strong>vertible near 0.<br />

Solution:<br />

(a) (Inverse function theorem (IFT) <strong>of</strong> calculus) 21<br />

Let f : E → R n be a C 1 -mapp<strong>in</strong>g <strong>of</strong> an open set E ⊂ R n . Suppose that f ′ (a) is <strong>in</strong>vertible for some a ∈ E <strong>and</strong><br />

that f(a) = b. Then,<br />

(i) there exist open sets U <strong>and</strong> V <strong>in</strong> R n such that a ∈ U, b ∈ V , <strong>and</strong> f maps U bijectively onto V , <strong>and</strong><br />

(ii) if g is the <strong>in</strong>verse <strong>of</strong> f (which exists by (i)), def<strong>in</strong>ed on V by g(f(x)) = x, for x ∈ U, then g ∈ C 1 (V ).<br />

(b) First note that L <strong>and</strong> g both have cont<strong>in</strong>uous first order partial derivatives; i.e., L, g ∈ C 1 (R 3 ). Therefore, the<br />

derivative <strong>of</strong> f = L + g,<br />

f ′ <br />

∂fi<br />

(x) Jf (x) <br />

∂xj<br />

exists. Furthermore, Jf (x) is cont<strong>in</strong>uous <strong>in</strong> a neighborhood <strong>of</strong> the zero vector, because this is true <strong>of</strong> the partials <strong>of</strong><br />

g(x), <strong>and</strong> the partials <strong>of</strong> L(x) are the constant matrix L. Therefore, f ∈ C 1 (R 3 ). By the IFT, then, we need only<br />

show that f ′ (0) is <strong>in</strong>vertible. S<strong>in</strong>ce f ′ (x) = L + g ′ (x), we must show f ′ (0) = L + g ′ (0) is <strong>in</strong>vertible. Consider<br />

the matrix g ′ (0) = Jg(0). We claim, Jg(0) = 0. Indeed, if x1, x2, x3 are the elementary unit vectors (also known<br />

as i, j, k), then the elements <strong>of</strong> Jg(0) are<br />

∂gi<br />

∂xj<br />

3<br />

i,j=1<br />

gi(0 + hxj) − gi(0)<br />

(0) = lim<br />

= lim<br />

h→0 h<br />

h→0<br />

gi(hxj)<br />

. (8)<br />

h<br />

The second equality follows by the hypothesis that g is cont<strong>in</strong>uous <strong>and</strong> satisfies g(x) ≤ Cx 2 , which implies<br />

that g(0) = 0. F<strong>in</strong>ally, to show that (8) is zero, consider<br />

which implies<br />

|gi(hxj)| ≤ g(hxj) ≤ Chxj 2 = C|h| 2 ,<br />

|gi(hxj)|<br />

|h|<br />

2 |hxj|<br />

≤ C = C|h| → 0, as h → 0.<br />

|h|<br />

This proves that f ′ (0) = L, which is <strong>in</strong>vertible by assumption, so the IFT implies that f(x) is locally <strong>in</strong>vertible<br />

near 0. ⊓⊔<br />

2. Let f be a differentiable real valued function on the <strong>in</strong>terval (0, 1), <strong>and</strong> suppose the derivative <strong>of</strong> f is bounded on<br />

this <strong>in</strong>terval. Prove the existence <strong>of</strong> the limit L = lim x→0 + f(x).<br />

20 The <strong>in</strong>verse function theorem does not appear on the syllabus <strong>and</strong>, as far as I know, this is the only exam problem <strong>in</strong> which it has appeared. The<br />

implicit function theorem does appear on the syllabus, but I have never encountered an exam problem that required it.<br />

21 See Rud<strong>in</strong> [7].<br />

18


1.4 2000 November 17 1 REAL ANALYSIS<br />

3. Let f <strong>and</strong> g be Lebesgue <strong>in</strong>tegrable functions on [0, 1], <strong>and</strong> let F <strong>and</strong> G be the <strong>in</strong>tegrals<br />

F (x) =<br />

x<br />

Use Fub<strong>in</strong>i’s <strong>and</strong>/or Tonelli’s theorem to prove that<br />

1<br />

0<br />

0<br />

f(t) dt, G(x) =<br />

F (x)g(x) dx = F (1)G(1) −<br />

1<br />

0<br />

x<br />

0<br />

g(t) dt.<br />

f(x)G(x) dx.<br />

Other approaches to this problem are possible, but credit will be given only to solutions based on these theorems.<br />

4. Let (X, A, µ) be a f<strong>in</strong>ite measure space <strong>and</strong> suppose ν is a f<strong>in</strong>ite measure on (X, A) that is absolutely cont<strong>in</strong>uous<br />

with respect to µ. Prove that the norm <strong>of</strong> the Radon-Nikodym derivative f =<br />

L ∞ (ν).<br />

dν<br />

dµ<br />

<br />

is the same <strong>in</strong> L ∞ (µ) as it is <strong>in</strong><br />

1<br />

5. Suppose that {fn} is a sequence <strong>of</strong> Lebesgue measurable functions on [0, 1] such that limn→∞ 0 |fn| dx = 0 <strong>and</strong><br />

there is an <strong>in</strong>tegrable function g on [0, 1] such that |fn| 2 1<br />

≤ g, for each n. Prove that limn→∞ 0 |fn| 2 dx = 0.<br />

6. Denote by Pe the family <strong>of</strong> all even polynomials. Thus a polynomial p belongs to Pe if <strong>and</strong> only if p(x) =<br />

p(x)+p(−x)<br />

2 for all x. Determ<strong>in</strong>e, with pro<strong>of</strong>, the closure <strong>of</strong> Pe <strong>in</strong> L1 [−1, 1]. You may use without pro<strong>of</strong> the fact<br />

that cont<strong>in</strong>uous functions on [−1, 1] are dense <strong>in</strong> L1 [−1, 1].<br />

7. Suppose that f is real valued <strong>and</strong> <strong>in</strong>tegrable with respect to Lebesgue measure m on R <strong>and</strong> that there are real<br />

numbers a < b such that<br />

<br />

a · m(U) ≤ f dm ≤ b · m(U),<br />

for all open sets U <strong>in</strong> R. Prove that a ≤ f(x) ≤ b a.e.<br />

U<br />

19


1.5 2001 November 26 1 REAL ANALYSIS<br />

1.5 2001 November 26<br />

Instructions Masters students do any 4 problems Ph.D. students do any 5 problems. Use a separate sheet <strong>of</strong> paper for<br />

each new problem.<br />

1. Let {fn} be a sequence <strong>of</strong> Lebesgue measurable functions on a set E ⊂ R, where E is <strong>of</strong> f<strong>in</strong>ite Lebesgue measure.<br />

Suppose that there is M > 0 such that |fn(x)| ≤ M for n ≥ 1 <strong>and</strong> for all x ∈ E, <strong>and</strong> suppose that<br />

limn→∞ fn(x) = f(x) for each x ∈ E. Use Egor<strong>of</strong>f’s theorem to prove that<br />

<br />

<br />

f(x) dx = lim fn(x) dx.<br />

n→∞<br />

E<br />

Solution: First note that |f(x)| ≤ M for all x ∈ E. To see this, suppose it’s false for some x0 ∈ E, so that<br />

|f(x0)| > M. Then there is some ɛ > 0 such that |f(x0)| = M + ɛ. By the triangle <strong>in</strong>equality, then, for all n ∈ N,<br />

|f(x0) − fn(x0)| ≥ ||f(x0)| − |fn(x0)|| = |M + ɛ − |fn(x0)|| ≥ ɛ,<br />

which contradicts fn(x0) → f(x0). Thus, |f(x)| ≤ M for all x ∈ E.<br />

Next, fix ɛ > 0. By Egor<strong>of</strong>f’s theorem (A.8), there is a G ⊂ E such that µ(E \ G) < ɛ <strong>and</strong> fn → f uniformly<br />

on G. Furthermore, s<strong>in</strong>ce |fn| ≤ M <strong>and</strong> |f| ≤ M <strong>and</strong> µ(E) < ∞, it’s clear that {fn} ⊂ L 1 <strong>and</strong> f ∈ L 1 , so the<br />

follow<strong>in</strong>g <strong>in</strong>equalities make sense (here we’re us<strong>in</strong>g the notation fG = sup{|f(x)| : x ∈ G}):<br />

<br />

<br />

<br />

<br />

<br />

f dµ − fn dµ <br />

<br />

E<br />

E<br />

≤<br />

<br />

<br />

<br />

|f − fn| dµ = |f − fn| dµ + |f − fn| dµ<br />

E<br />

E\G<br />

G<br />

<br />

<br />

≤ |f| dµ + |fn| dµ + f(x) − fn(x)Gµ(G)<br />

E\G<br />

E\G<br />

≤ 2Mµ(E \ G) + f(x) − fn(x)Gµ(G)<br />

< ɛ + f(x) − fn(x)Gµ(G).<br />

F<strong>in</strong>ally, µ(G) ≤ µ(E) < ∞ <strong>and</strong> f(x) − fn(x)G → 0, which proves that <br />

E fn dµ → <br />

2. Let f(x) be a real-valued Lebesgue <strong>in</strong>tegrable function on [0, 1].<br />

(a) Prove that if f > 0 on a set F ⊂ [0, 1] <strong>of</strong> positive measure, then<br />

<br />

f(x) dx > 0.<br />

(b) Prove that if x<br />

f(x) dx = 0, for each x ∈ [0, 1],<br />

then f(x) = 0 for almost all x ∈ [0, 1].<br />

Solution: (a) Def<strong>in</strong>e Fn = {x ∈ F : f(x) > 1/n}. Then<br />

<strong>and</strong> m(F ) > 0 implies<br />

0<br />

F<br />

F1 ⊆ F2 ⊆ · · · ↑ <br />

Fn = F,<br />

0 < m(F ) = m(∪nFn) ≤ <br />

m(Fn).<br />

20<br />

n<br />

E<br />

n<br />

E<br />

f dµ. ⊓⊔


1.5 2001 November 26 1 REAL ANALYSIS<br />

Therefore, m(Fk) > 0 for some k ∈ N, <strong>and</strong> then it follows from the def<strong>in</strong>ition <strong>of</strong> Fk that<br />

0 < 1<br />

k m(Fk)<br />

<br />

≤<br />

<br />

f dm ≤ f dm.<br />

Fk<br />

(b)<br />

Suppose there is a subset E ⊂ [0, 1] <strong>of</strong> positive measure such that f > 0 on E. Then part (a) implies<br />

f dm > 0. Let F ⊂ E be a closed subset <strong>of</strong> positive measure. (That such a closed subset exists follows from<br />

E<br />

Prop. 3.15 <strong>of</strong> Royden [6].) Then, aga<strong>in</strong> by (a), <br />

f dm > 0. Now consider the set G = [0, 1] \ F , which is open<br />

F<br />

<strong>in</strong> [0, 1], <strong>and</strong> hence22 is a countable union <strong>of</strong> disjo<strong>in</strong>t open <strong>in</strong>tervals; i.e., G = ·∪n(an, bn). Therefore,<br />

<br />

0 = f dm = <br />

<br />

<br />

f dm + f dm,<br />

so <br />

f dm > 0 implies<br />

F<br />

[0,1]<br />

<br />

<br />

n<br />

n<br />

(an,bn)<br />

(an,bn)<br />

f dm < 0.<br />

Thus, <br />

(ak,bk) f dm < 0 for some (ak, bk) ⊂ [0, 1]. On the other h<strong>and</strong>,<br />

<br />

(ak,bk)<br />

f dm =<br />

bk<br />

0<br />

ak<br />

f(x) dm(x) − f(x) dm(x).<br />

0<br />

By the <strong>in</strong>itial hypothesis, both terms on the right are zero, which gives the desired contradiction. ⊓⊔<br />

3. State each <strong>of</strong> the follow<strong>in</strong>g:<br />

(a) The Stone-Weierstrass theorem<br />

(b) The Lebesgue (dom<strong>in</strong>ated) convergence theorem<br />

(c) Hölder’s <strong>in</strong>equality<br />

(d) The Riesz representation theorem for L p<br />

(e) The Hahn-Banach theorem.<br />

Solution: 23<br />

(a) (Stone-Weierstrass theorem)<br />

Let X be a compact Hausdorff space <strong>and</strong> let A be a closed subalgebra <strong>of</strong> functions <strong>in</strong> C(X, R) which separates<br />

po<strong>in</strong>ts. Then either A = C(X, R), or A = {f ∈ C(X, R) : f(x0) = 0} for some x0 ∈ X. The first case occurs iff<br />

A conta<strong>in</strong>s the constant functions.<br />

(b) (Lebesgue dom<strong>in</strong>ated convergence theorem) 24 Let {fn} be a sequence <strong>of</strong> measurable functions on (X, M, µ)<br />

such that fn → f a.e.. If there exists g ∈ L1 (X, M, µ) such that |fn(x)| ≤ g(x) holds for all x ∈ X <strong>and</strong><br />

n = 1, 2, . . .. Then {fn} ⊂ L1 , f ∈ L1 , lim <br />

X fn dµ = <br />

X f dµ, <strong>and</strong> fn − f1 → 0.<br />

(c) (Hölder’s <strong>in</strong>equality)<br />

Let f <strong>and</strong> g be measurable functions.<br />

22Every open set <strong>of</strong> real numbers is the union <strong>of</strong> a countable collection <strong>of</strong> disjo<strong>in</strong>t open <strong>in</strong>tervals (Royden [6], Prop. 8, page 42).<br />

23The presentations <strong>of</strong> (a) <strong>and</strong> (c) <strong>in</strong> Foll<strong>and</strong> [4] are especially nice. For (b) <strong>and</strong> (e), as well as (c), I like Rud<strong>in</strong> [8]. A version <strong>of</strong> (d) appears <strong>in</strong><br />

Royden [6].<br />

24See theorem A.7 for a more general version.<br />

21<br />

F<br />

F<br />

⊓⊔


1.5 2001 November 26 1 REAL ANALYSIS<br />

(i) If 1 < p < ∞ <strong>and</strong> 1 1<br />

p + q = 1, then fg1 = fpgq. Thus, if f ∈ Lp <strong>and</strong> g ∈ Lq , then fg ∈ L1 (ii) If p = ∞ <strong>and</strong> if f ∈ L ∞ <strong>and</strong> g ∈ L 1 , then |fg| ≤ f∞|g|, so fg1 ≤ f∞g1.<br />

(d) (Riesz representation theorem for L p ) 25<br />

Suppose 1 < p < ∞ <strong>and</strong> 1<br />

p<br />

+ 1<br />

q = 1. If Λ is a l<strong>in</strong>ear functional on Lp , then there is a unique g ∈ L q such that<br />

<br />

Λf = fg dµ (∀f ∈ L p ).<br />

(e) (Hahn-Banach theorem)<br />

Suppose X is a normed l<strong>in</strong>ear space, Y ⊆ X is a subspace, <strong>and</strong> T : Y → R is a bounded l<strong>in</strong>ear functional.<br />

Then there exists a bounded l<strong>in</strong>ear functional ¯ T : X → R such that ¯ T (y) = T (y) for all y ∈ Y , <strong>and</strong> such that<br />

¯ T X = T Y , where ¯ T X <strong>and</strong> T Y are the usual operator norms,<br />

¯ T X = sup{| ¯ T x| : x ∈ X, x ≤ 1} <strong>and</strong> T Y = sup{|T x| : x ∈ Y, x ≤ 1}.<br />

4. (a) State the Baire category theorem.<br />

(b) Prove the follow<strong>in</strong>g special case <strong>of</strong> the uniform boundedness theorem: Let X be a (nonempty) complete metric<br />

space <strong>and</strong> let F ⊆ C(X). Suppose that for each x ∈ X there is a nonnegative constant Mx such that<br />

|f(x)| ≤ Mx for all f ∈ F.<br />

Prove that there is a nonempty open set G ⊆ X <strong>and</strong> a constant M > 0 such that<br />

|f(x)| ≤ M holds for all x ∈ G <strong>and</strong> for all f ∈ F .<br />

Solution: 26<br />

(a) (Baire category theorem)<br />

If X is a complete metric space <strong>and</strong> {An} is a collection <strong>of</strong> open dense subsets, then ∞<br />

n=1 An is dense <strong>in</strong> X.<br />

Corollary 1. If X is a complete metric space <strong>and</strong> G ⊆ X is a non-empty open subset <strong>and</strong> G = ∞ o<br />

= ∅ for at least one n ∈ N.<br />

¯<br />

Gn<br />

Corollary 2. A nonempty complete metric space is not a countable union <strong>of</strong> nowhere dense sets.<br />

n=1 Gn then<br />

(b) Def<strong>in</strong>e Am = {x ∈ X : |f(x)| ≤ m, ∀f ∈ F }. Then X = ∞ m=1 Am, s<strong>in</strong>ce for every x there is a f<strong>in</strong>ite<br />

number Mx such that |f(x)| ≤ Mx for all f ∈ F . Now note that Am = <br />

f∈F {x ∈ X : |f(x)| ≤ m}, <strong>and</strong>, s<strong>in</strong>ce f<br />

<strong>and</strong> a ↦→ |a| are cont<strong>in</strong>uous functions, each {x ∈ X : |f(x)| ≤ m} is closed, so Am is closed. Therefore, corollary<br />

2 <strong>of</strong> the Baire category theorem implies that there must be some m ∈ N such that A◦ m = ∅, so the set G = A◦ m <strong>and</strong><br />

the number M = m satisfy the given criteria. ⊓⊔<br />

25 Note to self: add case p = ∞<br />

26 See Royden [6], § 7.8, for an excellent treatment <strong>of</strong> this topic. Part (b) <strong>of</strong> this problem appears there as theorem 32, <strong>and</strong> another popular exam<br />

question is part c <strong>of</strong> problem 37.<br />

22


1.5 2001 November 26 1 REAL ANALYSIS<br />

5. Prove or disprove:<br />

(a) L 2 convergence implies po<strong>in</strong>twise convergence.<br />

(b)<br />

∞<br />

s<strong>in</strong>(x<br />

lim<br />

n→∞<br />

0<br />

n )<br />

xn dx = 0.<br />

(c) Let {fn} be a sequence <strong>of</strong> measurable functions def<strong>in</strong>ed on [0, ∞). If fn → 0 uniformly on [0, ∞), as n → ∞,<br />

then<br />

<br />

<br />

lim fn(x) dx = lim fn(x) dx.<br />

Solution:<br />

[0,∞)<br />

[0,∞)<br />

(a) This is false, as the follow<strong>in</strong>g example demonstrates: For each k ∈ N, def<strong>in</strong>e fk,j = χ [ j−1<br />

k<br />

<strong>and</strong> let {gn} be the sequence def<strong>in</strong>ed by<br />

g1 = f1,1,<br />

g2 = f2,1, g3 = f2,2,<br />

g4 = f3,1, g5 = f3,2, g6 = f3,3,<br />

g7 = f4,1, . . .<br />

j<br />

, k ) for j = 1, . . . , k,<br />

Then |fk,j| 2 dµ = 1/k for each j = 1, . . . , k, so fk,j2 = 1/ √ k → 0, as k → ∞. Therefore gn2 → 0 as<br />

n → ∞. However, {gn} does not converge po<strong>in</strong>twise s<strong>in</strong>ce, for every x ∈ [0, 1] <strong>and</strong> every N ∈ N, we can always<br />

f<strong>in</strong>d some k ∈ N <strong>and</strong> j ∈ {1, . . . , k} such that gn(x) = fk,j(x) = 1 with n ≥ N, <strong>and</strong> we can also f<strong>in</strong>d a k ′ ∈ N<br />

<strong>and</strong> j ′ ∈ {1, . . . , k ′ } such that gn ′(x) = fk ′ ,j ′(x) = 0 with n′ ≥ N. ⊓⊔<br />

s<strong>in</strong> t<br />

t<br />

(b) For any fixed 0 < x < 1, limn→∞ xn = 0. Also,<br />

Together, these two facts yield<br />

s<strong>in</strong> x<br />

lim<br />

n→∞<br />

n<br />

xn = 1.<br />

→ 1, as t → 0, which can be proved by L’Hopital’s rule.<br />

Now, recall that | s<strong>in</strong> θ| ≤ |θ| for all real θ. Indeed, s<strong>in</strong>ce s<strong>in</strong> θ = θ<br />

cos x dx, we have, for θ ≥ 0,<br />

0<br />

<strong>and</strong>, for θ < 0,<br />

In particular, for any 0 < x < 1, | s<strong>in</strong> xn |<br />

|xn |<br />

function<br />

s<strong>in</strong> xn<br />

x n , to obta<strong>in</strong><br />

| s<strong>in</strong> θ| ≤<br />

θ<br />

0<br />

| cos x| dx ≤<br />

θ<br />

0<br />

1 dx = θ,<br />

| s<strong>in</strong> θ| = | s<strong>in</strong>(−θ)| ≤ | − θ| = |θ|.<br />

≤ 1. Therefore, we can apply the dom<strong>in</strong>ated convergence theorem to the<br />

1<br />

lim<br />

n→∞<br />

0<br />

s<strong>in</strong> xn dx =<br />

xn 1<br />

0<br />

1 dx = 1. (9)<br />

Next consider the part <strong>of</strong> the <strong>in</strong>tegral over 1 ≤ x < N, for any real N > 1. Fix n ≥ 2. The change <strong>of</strong> variables<br />

u = xn results <strong>in</strong> du = nxn−1dx, <strong>and</strong>, s<strong>in</strong>ce u1/n = x, we have xn−1 1 1− = u n . Therefore,<br />

N<br />

1<br />

s<strong>in</strong> xn n<br />

N<br />

s<strong>in</strong> u<br />

dx =<br />

xn 1 u<br />

23<br />

du<br />

1<br />

nu1− n<br />

= 1<br />

n<br />

N<br />

n 1<br />

s<strong>in</strong> u<br />

1<br />

u2− n<br />

du.


1.5 2001 November 26 1 REAL ANALYSIS<br />

Now,<br />

1<br />

lim<br />

N→∞ n<br />

<br />

<br />

<br />

<br />

<br />

N n<br />

1<br />

s<strong>in</strong> u<br />

1<br />

u2− n<br />

<br />

<br />

<br />

du<br />

<br />

Therefore, ∞<br />

<strong>and</strong> so,<br />

Comb<strong>in</strong><strong>in</strong>g results (9) <strong>and</strong> (10) yields<br />

n<br />

N<br />

1<br />

≤ lim u<br />

N→∞ n 1<br />

1<br />

n −2 1<br />

du = lim<br />

N→∞ n<br />

1<br />

<br />

<br />

dx<br />

n <br />

s<strong>in</strong> x n<br />

x<br />

∞<br />

lim<br />

n→∞<br />

1<br />

∞<br />

lim<br />

n→∞<br />

0<br />

≤ 1<br />

n − 1 ,<br />

u 1<br />

n −1<br />

<br />

<br />

<br />

<br />

− 1<br />

1<br />

n<br />

N n<br />

1<br />

= 1<br />

n − 1 .<br />

s<strong>in</strong> xn dx = 0. (10)<br />

xn s<strong>in</strong> xn dx = 1.<br />

xn (c) This is false, as the follow<strong>in</strong>g example demonstrates: Let fn = 1<br />

n χ [0,n). Then fn → 0 uniformly <strong>and</strong> so<br />

lim fn = 0. On the other h<strong>and</strong>, fn = 1 for all n ∈ N. Therefore, lim fn = 1 = 0 = lim fn. ⊓⊔<br />

6. Let f : H → H be a bounded l<strong>in</strong>ear functional on a separable Hilbert space H (with <strong>in</strong>ner product denoted by<br />

〈·, ·〉). Prove that there is a unique element y ∈ H such that<br />

f(x) = 〈x, y〉 for all x ∈ H <strong>and</strong> f = y.<br />

H<strong>in</strong>t. You may use the follow<strong>in</strong>g facts: A separable Hilbert space, H, conta<strong>in</strong>s a complete orthonormal sequence,<br />

{φk} ∞ k=1 , satisfy<strong>in</strong>g the follow<strong>in</strong>g properties: (1) If x, y ∈ H <strong>and</strong> if 〈x, φk〉 = 〈y, φk〉 for all k, then x = y. (2)<br />

Parseval’s equality holds; that is, for all x ∈ H, 〈x, x〉 = ∞<br />

k=1 a2 k , where ak = 〈x, φk〉.<br />

Solution: Def<strong>in</strong>e y = ∞ k=1 f(φk)φk, <strong>and</strong> check that this y ∈ H has the desired properties.<br />

First observe that, by properties (1) <strong>and</strong> (2) given the h<strong>in</strong>t, any x ∈ H can be written as x = ∞ k=1 akφk, where<br />

ak = 〈x, φk〉, for each k ∈ N. Therefore, by l<strong>in</strong>earity <strong>of</strong> f,<br />

f(x) = f( <br />

akφk) = <br />

akf(φk). (11)<br />

Now<br />

<strong>and</strong>, by def<strong>in</strong>ition <strong>of</strong> y,<br />

k<br />

k<br />

〈x, y〉 = 〈 <br />

akφk, y〉 = <br />

ak〈φk, y〉, (12)<br />

k<br />

k<br />

〈φk, y〉 = 〈φk, <br />

f(φj)φj〉 = <br />

f(φj)〈φk, φj〉 = f(φk). (13)<br />

j<br />

The last equality holds by orthonormality; i.e., 〈φk, φj〉 is 1 when j = k <strong>and</strong> 0 otherwise. Putt<strong>in</strong>g it all together,<br />

we see that, for every x ∈ H,<br />

f(x) = <br />

akf(φk) ∵ (11)<br />

k<br />

= <br />

ak〈φk, y〉 ∵ (13)<br />

k<br />

= 〈x, y〉 ∵ (12)<br />

24<br />

j<br />

⊓⊔


1.5 2001 November 26 1 REAL ANALYSIS<br />

Moreover, this y is unique. For, suppose there is another y ′ ∈ H such that f(x) = 〈x, y ′ 〉 for all x ∈ X. Then<br />

〈x, y〉 = f(x) = 〈x, y ′ 〉 for all x ∈ X. In particular, 〈φk, y〉 = f(φk) = 〈φk, y ′ 〉 for each k ∈ N, which, by<br />

property (1) <strong>of</strong> the h<strong>in</strong>t, proves that y = y ′ .<br />

F<strong>in</strong>ally, we must show f = y. Observe,<br />

|f(x)| |(x, y)|<br />

f = sup{|f(x)|<br />

: x ≤ 1} = sup = sup<br />

x∈X<br />

x∈X x x∈X x ,<br />

<strong>and</strong> recall that |(x, y)| ≤ xy holds for all x, y ∈ X. Whence,<br />

On the other h<strong>and</strong>,<br />

|(x, y)|<br />

f = sup ≤ y. (14)<br />

x∈X x<br />

|(x, y)| |(y, y)| y2<br />

f = sup ≥ = = y. (15)<br />

x∈X x y y<br />

Together, (14) <strong>and</strong> (15) give f = y, as desired. ⊓⊔<br />

7. Let X be a normed l<strong>in</strong>ear space <strong>and</strong> let Y be a Banach space. Let<br />

B(X, Y ) = {A | A : X → Y is a bounded l<strong>in</strong>ear operator}.<br />

Then with the norm A = sup x≤1 Ax, B(X, Y ) is a normed l<strong>in</strong>ear space (you need not show this). Prove<br />

that B(X, Y ) is a Banach space; that is, prove that B(X, Y ) is complete.<br />

Solution: Let {Tn} ⊂ B(X, Y ) be a Cauchy sequence; i.e., Tn − Tm → 0 as m, n → ∞. Fix x ∈ X. Then,<br />

Tnx − TmxY ≤ Tn − TmxX → 0, as n, m → ∞.<br />

Therefore, the sequence {Tnx} ⊂ Y is a Cauchy sequence <strong>in</strong> (Y, · Y ). S<strong>in</strong>ce the latter is complete, the limit<br />

limn→∞ Tnx = y ∈ Y exists. Def<strong>in</strong>e T : X → Y by T x = limn→∞ Tnx, for each x ∈ X. To complete the pro<strong>of</strong>,<br />

we must check that T is l<strong>in</strong>ear, bounded, <strong>and</strong> satisfies limn→∞ Tn − T = 0.<br />

• T is l<strong>in</strong>ear:<br />

For x1, x2 ∈ X,<br />

T (x1 + x2) = lim<br />

n→∞ Tn(x1 + x2)<br />

= lim<br />

n→∞ (Tnx1 + Tnx2) (∵ Tn is l<strong>in</strong>ear)<br />

= lim<br />

n→∞ Tnx1 + lim<br />

n→∞ Tnx2<br />

= T x1 + T x2.<br />

(∵ both limits exist)<br />

• T is bounded:<br />

First, note that {Tn} is a Cauchy sequence <strong>of</strong> real numbers, s<strong>in</strong>ce | Tn − Tm | ≤ Tn − Tm → 0,<br />

as n, m → ∞. Therefore, there is a c ∈ R such that Tn → c, as n → ∞. For some N ∈ N, then,<br />

Tn ≤ c + 1 for all n ≥ N. Thus,<br />

TnxY ≤ TnxX ≤ (c + 1)xX (∀x ∈ X). (16)<br />

25


1.5 2001 November 26 1 REAL ANALYSIS<br />

Now, by def<strong>in</strong>ition, Tnx → T x, for all x ∈ X <strong>and</strong>, s<strong>in</strong>ce the norm · Y is uniformly cont<strong>in</strong>uous, 27<br />

TnxY → T xY (∀x ∈ X). (17)<br />

Taken together, (16) <strong>and</strong> (17) imply T xY ≤ (c + 1)xX, for all x ∈ X. Therefore, T is bounded.<br />

• limn→∞ Tn − T = 0:<br />

Fix ɛ > 0 <strong>and</strong> choose N ∈ N such that n, m ≥ N implies Tn − Tm < ɛ. Then,<br />

Tnx − Tmx ≤ Tn − TmxX < ɛxX<br />

holds for all n, m ≥ N, <strong>and</strong> x ∈ X. Lett<strong>in</strong>g m go to <strong>in</strong>f<strong>in</strong>ity, then,<br />

Tnx − T x = lim<br />

m→∞ Tnx − Tmx ≤ ɛxX.<br />

That is, Tnx − T x ≤ ɛxX, for all n ≥ N <strong>and</strong> x ∈ X. Whence, Tn − T ≤ ɛ for all n ≥ N.<br />

27 Pro<strong>of</strong>: | aY − bY | ≤ a − bY (∀a, b ∈ Y ).<br />

26<br />

⊓⊔


1.6 2004 April 19 1 REAL ANALYSIS<br />

1.6 2004 April 19<br />

Instructions. Use a separate sheet <strong>of</strong> paper for each new problem. Do as many problems as you can. Complete<br />

solutions to five problems will be considered as an excellent performance. Be advised that a few complete <strong>and</strong> well<br />

written solutions will count more than several partial solutions.<br />

Notation: f ∈ C(X) means that f is a real-valued, cont<strong>in</strong>uous function def<strong>in</strong>ed on X.<br />

1. (a) Let S be a (Lebesgue) measurable subset <strong>of</strong> R <strong>and</strong> let f, g : S → R be measurable functions. Prove that (i)<br />

f + g is measurable <strong>and</strong> (ii) if φ ∈ C(R), then φ(f) is measurable.<br />

(b) Let f : [a, b] → [−∞, ∞] be a measurable function. Suppose that f takes the value ±∞ only on a set <strong>of</strong><br />

(Lebesgue) measure zero. Prove that for any ɛ > 0 there is a positive number M such that |f| ≤ M, except on a<br />

set <strong>of</strong> measure less than ɛ.<br />

Solution:<br />

(a) Pro<strong>of</strong> 1: S<strong>in</strong>ce f <strong>and</strong> g are real measurable functions <strong>of</strong> S, <strong>and</strong> s<strong>in</strong>ce the mapp<strong>in</strong>g Φ : R × R → R def<strong>in</strong>ed by<br />

Φ(x, y) = x + y is cont<strong>in</strong>uous, theorem A.3 implies that the function f + g = Φ(f, g) is measurable. If φ ∈ C(R),<br />

then φ(f) is measurable by part (b) <strong>of</strong> theorem A.2.<br />

Pro<strong>of</strong> 2: Let {qi} ∞ i=1 be an enumeration <strong>of</strong> the rationals. Then, for any α ∈ R,<br />

{x ∈ S : f(x) + g(x) < α} =<br />

∞<br />

{x ∈ S : f(x) < α − qi} ∩ {x ∈ S : g(x) < qi}.<br />

i=1<br />

S<strong>in</strong>ce each set on the right is measurable, <strong>and</strong> s<strong>in</strong>ce σ-algebras are closed under countable unions <strong>and</strong> <strong>in</strong>tersections,<br />

{x ∈ S : f(x) + g(x) < α} is measurable. S<strong>in</strong>ce α was arbitrary, f + g is measurable.<br />

The function φf is measurable if <strong>and</strong> only if, for any open subset U <strong>of</strong> R, the set (φf) −1 (U) is measurable. Let U<br />

be open <strong>in</strong> R. Then φ −1 (U) is open, s<strong>in</strong>ce φ ∈ C(R), <strong>and</strong> so (φf) −1 (U) = f −1 (φ −1 (U)) is measurable, s<strong>in</strong>ce f<br />

is measurable. Therefore, φf is measurable.<br />

(b) Fix ɛ > 0. For n ∈ N, def<strong>in</strong>e An = {x ∈ [a, b] : |f(x)| ≤ n}. Then<br />

[a, b] =<br />

∞<br />

An ∪ A∞, (18)<br />

n=1<br />

where 28 A∞ = {x : f(x) = ±∞}. Also, A1 ⊆ A2 ⊆ · · · <strong>and</strong>, s<strong>in</strong>ce f is measurable, each An is measurable.<br />

Therefore, µ(An) ↑ µ(∪nAn), as n → ∞. Note that all sets are conta<strong>in</strong>ed <strong>in</strong> [a, b] <strong>and</strong> thus have f<strong>in</strong>ite measure.<br />

Let M ∈ N be such that µ(∪nAn) − µ(AM ) < ɛ. Then |f| ≤ M except on [a, b] \ AM , <strong>and</strong> by (18),<br />

µ([a, b] \ AM ) = µ(∪nAn ∪ A∞ \ AM )<br />

≤ µ(∪nAn \ AM ) + µ(A∞)<br />

= µ(∪nAn \ AM ) < ɛ.<br />

The second equality holds s<strong>in</strong>ce we assumed f(x) = ±∞ only on a set <strong>of</strong> measure zero; i.e., µ(A∞) = 0. ⊓⊔<br />

28 S<strong>in</strong>ce f is an extended real valued function, we must not forget to <strong>in</strong>clude A∞, without which the union <strong>in</strong> (18) would not be all <strong>of</strong> [a, b].<br />

27


1.6 2004 April 19 1 REAL ANALYSIS<br />

2. (a) State Egorov’s theorem.<br />

(b) State Fatou’s lemma.<br />

(c) Let {fn} ⊂ L p [0, 1], where 1 ≤ p < ∞. Suppose that fn → f a.e., where f ∈ L p [0, 1]. Prove that<br />

fn − fp → 0 if <strong>and</strong> only if fnp → fp.<br />

Solution:<br />

(a) See theorem A.8.<br />

(b) See theorem A.6.<br />

(c) (⇒) By the M<strong>in</strong>kowsky <strong>in</strong>equality, fnp = fn − f + fp ≤ fn − fp + fp. Similarly, fp ≤<br />

fn − fp + fnp. Together, the two <strong>in</strong>equalities yield |fnp − fp| ≤ fn − fp. Therefore, fn − fp → 0<br />

implies |fnp − fp| → 0. This proves necessity.<br />

(⇐) I know <strong>of</strong> three pro<strong>of</strong>s <strong>of</strong> sufficiency. The second is similar to the first, only much shorter as it exploits the full<br />

power <strong>of</strong> the general version <strong>of</strong> Lebesgue’s dom<strong>in</strong>ated convergence theorem, whereas the first pro<strong>of</strong> merely relies<br />

on Fatou’s lemma. 29 The third pro<strong>of</strong> uses both Fatou’s lemma <strong>and</strong> Egor<strong>of</strong>f’s theorem, so, judg<strong>in</strong>g from parts (a)<br />

<strong>and</strong> (b), this may be closer to what the exam<strong>in</strong>ers had <strong>in</strong> m<strong>in</strong>d. Note that none <strong>of</strong> the pro<strong>of</strong>s use the assumption that<br />

the measure space is f<strong>in</strong>ite, so we may as well work <strong>in</strong> the more general space L p (X, M, µ).<br />

Both pro<strong>of</strong>s 1 <strong>and</strong> 2 make use <strong>of</strong> the follow<strong>in</strong>g:<br />

Lemma 1.4 If α, β ∈ [0, ∞) <strong>and</strong> 1 ≤ p < ∞, then (α + β) p ≤ 2 p−1 (α p + β p ).<br />

Pro<strong>of</strong>: When p ≥ 1, φ(x) = x p is convex on [0, ∞). Thus, for all α, β ∈ [0, ∞),<br />

p <br />

α + β α + β<br />

= φ ≤<br />

2<br />

2<br />

1<br />

1<br />

[φ(α) + φ(β)] =<br />

2 2 (αp + β p ).<br />

When α, β ∈ R, the triangle <strong>in</strong>equality followed by the lemma yields<br />

Pro<strong>of</strong> 1: By (19),<br />

|α − β| p ≤ ||α| + |β|| p ≤ 2 p−1 (|α| p + |β| p ). (19)<br />

|fn − f| p ≤ 2 p−1 (|fn| p + |f| p ).<br />

In particular, fn − f ∈ L p , for each n ∈ N. Moreover, the functions<br />

gn = 2 p−1 (|fn| p + |f| p ) − |fn − f| p . (20)<br />

are non-negative. Now notice that lim gn = 2p |f| p . Apply<strong>in</strong>g Fatou’s lemma to (20), then,<br />

<br />

2 p |f| p <br />

=<br />

<br />

lim gn ≤ lim<br />

<br />

2p−1 gn = lim (|fn| p + |f| p ) − |fn − f| p .<br />

S<strong>in</strong>ce fnp → fp, this implies<br />

2 p<br />

<br />

|f| p ≤ 2 p<br />

<br />

|f| p <br />

− lim<br />

|fn − f| p .<br />

Equivalently 0 ≤ − lim |fn − f| p . This proves fn − f p → 0. ⊓⊔<br />

29 Disclaimer: I made up the first pro<strong>of</strong>, so you should check it carefully for yourself <strong>and</strong> decide whether you believe me.<br />

28


1.6 2004 April 19 1 REAL ANALYSIS<br />

Pro<strong>of</strong> 2: By (19),<br />

In particular, fn − f ∈ L p , for each n ∈ N. Def<strong>in</strong>e the functions<br />

|fn − f| p ≤ 2 p−1 (|fn| p + |f| p ). (21)<br />

gn = 2 p−1 (|fn| p + |f| p ) <strong>and</strong> g = 2 p |f| p .<br />

Then gn → g a.e., <strong>and</strong> fnp → fp implies gn → g. Also, gn ≥ |fn − f| p → 0 a.e., by (21). Therefore,<br />

the dom<strong>in</strong>ated convergence theorem (theorem A.7) implies |fn − f| p → 0. ⊓⊔<br />

Pro<strong>of</strong> 3: S<strong>in</strong>ce f ∈ Lp , for all ɛ > 0, there is a number δ > 0 <strong>and</strong> a set B ∈ M <strong>of</strong> f<strong>in</strong>ite measure such that f is<br />

bounded on B, <br />

X\B |f|p dµ < ɛ/2, <strong>and</strong> <br />

E |f|p dµ < ɛ/2, for all E ∈ M with µE < δ. By Egor<strong>of</strong>f’s theorem,<br />

there is a set A ⊆ B such that µ(B \ A) < δ <strong>and</strong> fn → f uniformly on A. Therefore,<br />

<br />

|f| p <br />

= |f| p <br />

+ |f| p <br />

+ |f| p<br />

X<br />

X\B<br />

s<strong>in</strong>ce, by Fatou’s lemma, <br />

A |f|p = <br />

A lim |fn| p ≤ lim <br />

<br />

lim |fn|<br />

A<br />

p = lim<br />

By (22), then, <br />

Therefore, lim <br />

Therefore,<br />

X<br />

B\A<br />

A<br />

<br />

< ɛ/2 + ɛ/2 + |f|<br />

A<br />

p<br />

<br />

≤ ɛ + lim |fn| p , (22)<br />

X<br />

A<br />

|fn| p <br />

− |fn|<br />

X\A<br />

p<br />

<br />

A |fn| p . By hypothesis, fnp → fp. Therefore<br />

<br />

=<br />

|f| p <br />

< ɛ + |f|<br />

X<br />

p <br />

− lim |fn|<br />

X\A<br />

p .<br />

X\A |fn| p < ɛ (s<strong>in</strong>ce f ∈ L p ). F<strong>in</strong>ally, note that<br />

fn − f p = (fn − f)χA + (fn − f)χ X\Ap<br />

≤ (fn − f)χAp + (fn − f)χ X\Ap<br />

≤ (fn − f)χAp + fnχ X\Ap + fχ X\Ap.<br />

lim fn − f p ≤ lim{|fn(x) − f(x) : x ∈ A}µ(A) 1/p + lim<br />

X<br />

<br />

|f| p <br />

− lim |fn|<br />

X\A<br />

p .<br />

|fn|<br />

X\A<br />

p<br />

(M<strong>in</strong>kowsky)<br />

1/p<br />

+<br />

<br />

|f|<br />

X\A<br />

p<br />

The first term on the right goes to zero s<strong>in</strong>ce fn → f uniformly on A. The other terms are bounded by 2ɛ 1/p . ⊓⊔<br />

29<br />

1/p<br />

.


1.6 2004 April 19 1 REAL ANALYSIS<br />

3. (a) Let S = [0, 1] <strong>and</strong> let {fn} ⊂ Lp (S), where 1 < p < ∞. Suppose that fn → f a.e. on S, where f ∈ Lp (S). If<br />

there is a constant M such that fnp ≤ M for all n, prove that for each g ∈ Lq (S), 1 1<br />

p + q = 1, we have<br />

<br />

lim<br />

n→∞<br />

fng =<br />

S<br />

fg.<br />

S<br />

(b) Show by means <strong>of</strong> an example that this result is false for p = 1.<br />

Solution:<br />

(a) S<strong>in</strong>ce g ∈ Lq (S), for all ɛ > 0 there exists δ > 0 such that if A is a measurable set with µA < δ then<br />

<br />

|g| q dµ < ɛ.<br />

A<br />

Let B0 ⊂ S denote the set on which fn does not converge to f. Let B ⊂ S be such that fn → f uniformly <strong>in</strong><br />

S \ B, <strong>and</strong> such that µB < δ. (Such a set exists by Egor<strong>of</strong>f’s theorem s<strong>in</strong>ce µS < ∞.) Now throw the set B0 <strong>in</strong><br />

with B (i.e. redef<strong>in</strong>e B to be B ∪ B0). Then,<br />

<br />

<br />

Dn := |fng − fg| dµ = |fn − f||g| dµ<br />

S <br />

S<br />

=<br />

|fn − f||g| dµ +<br />

S\B<br />

|fn − f||g| dµ<br />

B<br />

≤ (fn − f)χ S\Bpgq + fn − fpgχBq.<br />

The f<strong>in</strong>al <strong>in</strong>equality holds because fn, f ∈ L p implies |fn − f| ∈ L p <strong>and</strong> by Hölder’s <strong>in</strong>equality. Now, by<br />

M<strong>in</strong>kowski’s <strong>in</strong>equality, fn − fp ≤ fnp + fp, so<br />

Dn ≤ sup{|fn(x) − f(x)| : x /∈ B}µ(S \ B) 1/p <br />

gq + (fnp + fp) |g|<br />

B<br />

q 1/q dµ .<br />

Now we are free to choose the δ > 0 above so that<br />

<br />

|g|<br />

B<br />

q dµ<br />

1/q<br />

<<br />

ɛ<br />

2(M + fp)<br />

holds whenever µB < δ. Also, fn → f uniformly on S \ B, so let N be such that<br />

ɛ<br />

sup{|fN(x) − f(x)| : x /∈ B} <<br />

2µ(S \ B) 1/p .<br />

gq<br />

Then DN < ɛ/2 + ɛ/2. ⊓⊔<br />

(b) The result is not true for p = 1 as the follow<strong>in</strong>g example shows: Let fn = nχ [0,1/n], n = 1, 2, . . . . First note<br />

that fn → 0 a.e. For if x ∈ (0, 1], there exists N > 0 such that 1/N < x <strong>and</strong> thus fn(x) = 0 for all n ≥ N.<br />

Therefore, {x ∈ [0, 1] : fn(x) 0} = {0}. Now, if g is the constant function g ≡ 1, then fng dµ = 1 for all<br />

n = 1, 2, . . . . Therefore, fng dµ → 1, while fg dµ = 0g dµ = 0. F<strong>in</strong>ally, note that fn1 = nµ([0, 1<br />

n ]) = 1<br />

for n = 1, 2, . . . , so {fn} satisfies the hypothesis fn1 ≤ some constant M. ⊓⊔<br />

30


1.6 2004 April 19 1 REAL ANALYSIS<br />

4. State <strong>and</strong> prove the closed graph theorem.<br />

5. Prove or disprove:<br />

(a) For 1 ≤ p < ∞, let ℓ p := {x = {xk} | xp = ( ∞<br />

k=1 |xk| p ) 1/p < ∞}. Then for p = 2, ℓ p is a Hilbert<br />

space.<br />

(b) Let X = (C[0, 1], · 1), where the l<strong>in</strong>ear space C[0, 1] is endowed with the L1-norm: f1 = 1<br />

|f(x)| dx.<br />

0<br />

Then X is a Banach space.<br />

(c) Every real, separable Hilbert space is isometrically isomorphic to ℓ 2 .<br />

6. (a) Give a precise statement <strong>of</strong> some version <strong>of</strong> Fub<strong>in</strong>i’s theorem that is valid for non-negative functions.<br />

(b) Let f, g ∈ L1 (R). (i) Prove that the <strong>in</strong>tegral<br />

<br />

h(x) =<br />

R<br />

f(x − t)g(t) dt<br />

exists for almost all x ∈ R <strong>and</strong> that h ∈ L 1 (R). (ii) Show that h1 ≤ f1g1.<br />

7. (a) State the Radon-Nikodym theorem.<br />

(b) Let (X, B, µ) be a complete measure space, where µ is a positive measure def<strong>in</strong>ed on the σ-algebra, B, <strong>of</strong><br />

subsets <strong>of</strong> X. Suppose µ(X) < ∞. Let S be a closed subset <strong>of</strong> R <strong>and</strong> let f ∈ L1 (µ), where f is an extended<br />

real-valued function def<strong>in</strong>ed on X. If<br />

AE(f) = 1<br />

<br />

f dµ ∈ S<br />

µ(E)<br />

for every E ∈ B with µ(E) > 0, prove that f(x) ∈ S for almost all x ∈ X.<br />

31<br />

E


1.7 2007 November 16 1 REAL ANALYSIS<br />

1.7 2007 November 16<br />

Notation: R is the set <strong>of</strong> real numbers <strong>and</strong> R n is n-dimensional Euclidean space. Denote by m Lebesgue measure on<br />

R <strong>and</strong> mn n-dimensional Lebesgue measure. Be sure to give a complete statement <strong>of</strong> any theorems from analysis that<br />

you use <strong>in</strong> your pro<strong>of</strong>s below.<br />

1. Let µ be a positive measure on a measure space X. Assume that E1, E2, . . . are measurable subsets <strong>of</strong> X with the<br />

property that for n = m, µ(En ∩ Em) = 0. Let E be the union <strong>of</strong> these sets. Prove that<br />

µ(E) =<br />

∞<br />

µ(En)<br />

Solution: Def<strong>in</strong>e F1 = E1, F2 = E2 \ E1, F3 = E3 \ (E1 ∪ E2), . . . , <strong>and</strong>, <strong>in</strong> general,<br />

n=1<br />

n−1 <br />

Fn = En \ Ei (n = 2, 3, . . . ).<br />

i=1<br />

If M is the σ-algebra <strong>of</strong> µ-measurable subsets <strong>of</strong> X, then Fn ∈ M for each n ∈ N, s<strong>in</strong>ce M is a σ-algebra. Also,<br />

Fi ∩ Fj = ∅ for i = j, <strong>and</strong> F1 ∪ F2 ∪ · · · ∪ Fn = E1 ∪ E2 ∪ · · · ∪ En for all n ∈ N. Thus,<br />

<strong>and</strong>, by σ-additivity <strong>of</strong> µ,<br />

∞<br />

n=1<br />

Fn =<br />

n=1<br />

∞<br />

En E,<br />

n=1<br />

∞<br />

∞<br />

µ(E) = µ( Fn) = µ(Fn).<br />

Therefore, if we can show µ(En) = µ(Fn) holds for all n ∈ N, the pro<strong>of</strong> will be complete.<br />

Now, for each n = 2, 3, . . . ,<br />

<strong>and</strong><br />

n=1<br />

n−1 <br />

Fn = En ∩ ( Ei) c<br />

i=1<br />

n−1 <br />

µ(En) = µ(En ∩ ( Ei) c n−1 <br />

) + µ(En ∩ ( Ei)). (24)<br />

Equation (24) holds because n−1<br />

i=1 Ei is a measurable set for each n = 2, 3, . . . . F<strong>in</strong>ally, note that<br />

which implies<br />

i=1<br />

n−1 <br />

En ∩ ( Ei) =<br />

i=1<br />

n−1 <br />

µ(En ∩ ( Ei)) ≤<br />

i=1<br />

<br />

(En ∩ Ei),<br />

n−1<br />

i=1<br />

i=1<br />

n−1 <br />

µ(En ∩ Ei),<br />

by σ-subadditivity. By assumption, each term <strong>in</strong> the last sum is zero, <strong>and</strong> therefore, by (23) <strong>and</strong> (24),<br />

n−1 <br />

µ(En) = µ(En ∩ ( Ei) c ) = µ(Fn) holds for each n = 2, 3, . . . .<br />

i=1<br />

For n = 1, we have F1 = E1, by def<strong>in</strong>ition. This completes the pro<strong>of</strong>. ⊓⊔<br />

32<br />

i=1<br />

(23)


1.7 2007 November 16 1 REAL ANALYSIS<br />

2. (a) State a theorem that illustrates Littlewood’s Pr<strong>in</strong>ciple for po<strong>in</strong>twise a.e. convergence <strong>of</strong> a sequence <strong>of</strong> functions<br />

on R.<br />

(b) Suppose that fn ∈ L 1 (m) for n = 1, 2, . . . . Assum<strong>in</strong>g that fn − f1 → 0 <strong>and</strong> fn → g a.e. as n → ∞, what<br />

relation exists between f <strong>and</strong> g? Make a conjecture <strong>and</strong> then prove it us<strong>in</strong>g the statement <strong>in</strong> Part (a).<br />

Solution:<br />

(a) I th<strong>in</strong>k it’s generally accepted that the Littlewood pr<strong>in</strong>ciple deal<strong>in</strong>g with a.e. convergence <strong>of</strong> a sequence <strong>of</strong><br />

functions on R is Egor<strong>of</strong>f’s theorem, which is stated below <strong>in</strong> section A.8.<br />

(b) Conjecture: f = g a.e.<br />

Pro<strong>of</strong> 1: 30 First recall that L1 convergence implies convergence <strong>in</strong> measure. That is, if {fn} ⊂ L1 (m) <strong>and</strong><br />

fn − f1 → 0, then fn → f <strong>in</strong> measure. (Pro<strong>of</strong>: m({x : |fn(x) − f(x)| > ɛ}) ≤ 1<br />

ɛ fn − f1 → 0.) Next recall<br />

another important theorem31 which states that if fn → f <strong>in</strong> measure then there is a subsequence {fnj } ⊆ {fn}<br />

which converges a.e. to f as j → ∞. Comb<strong>in</strong><strong>in</strong>g these two results <strong>in</strong> the present context (Lebesgue measure on<br />

the real l<strong>in</strong>e), we can say the follow<strong>in</strong>g: 32 If {fn} ⊂ L 1 (m) <strong>and</strong> fn − f1 → 0 then there is a subsequence<br />

{fnj } ⊆ {fn} with the property fnj (x) → f(x) for almost all x ∈ R.<br />

Now, if fn(x) → g(x) for almost all x ∈ R, <strong>and</strong> if B1 be the set <strong>of</strong> measure zero where fn(x) g(x), then <strong>of</strong>f <strong>of</strong><br />

B1 the sequence fn, as well as every subsequence <strong>of</strong> fn, converges to g. Let {fnj } be the subsequence mentioned<br />

above which converges to f almost everywhere. Then<br />

|f(x) − g(x)| ≤ |f(x) − fnj (x)| + |fnj (x) − g(x)|. (25)<br />

Def<strong>in</strong>e B2 = {x ∈ R : fnj (x) f(x)}. Then the set B = B1 ∪ B2 has measure zero <strong>and</strong>, for all x ∈ R \ B,<br />

fnj (x) → f(x) <strong>and</strong> fnj (x) → g(x) . Therefore, by (25), |f(x) − g(x)| = 0 for all x ∈ R \ B. It follows that the<br />

set {x ∈ R : f (x) = g(x)} ⊂ B, as a subset <strong>of</strong> a null set, must itself be a null set (s<strong>in</strong>ce m is complete). That is,<br />

f = g a.e. <strong>and</strong> the conjecture is proved. ⊓⊔<br />

Pro<strong>of</strong> 2: First, we claim that if f = g a.e. on [−n, n] for every n ∈ N, then f = g a.e. <strong>in</strong> R. To see this, let<br />

Bn = {x ∈ [−n, n] : f(x) = g(x)}. Then mBn = 0 for all n ∈ N, so that if B = {x ∈ R : f(x) = g(x)}, then<br />

B = ∪Bn <strong>and</strong> mB ≤ mBn = 0, as claimed. Thus, to prove the conjecture, it is enough to show that f = g for<br />

almost every −n ≤ x ≤ n, for an arbitrary fixed n ∈ N.<br />

Fix n ∈ N, <strong>and</strong> suppose we know that f − g ∈ L1 ([−n, n], m). (This will follow from the fact that f, g ∈<br />

L1 ([−n, n], m), which we prove below.) Then, for all ɛ > 0 there is a δ > 0 such that <br />

|f − g| dm < ɛ for<br />

E<br />

all measurable E ⊆ [−n, n] with mE < δ. Now apply Egor<strong>of</strong>f’s theorem to f<strong>in</strong>d a set A ⊆ [−n, n] such that<br />

m([−n, n] \ A) < δ <strong>and</strong> fn → g uniformly on A. Then<br />

n<br />

<br />

<br />

|f − g| dm = |f − g| dm + |f − g| dm<br />

−n<br />

[−n,n]\A<br />

<br />

<br />

≤ ɛ + |f − fn| dm + |fn − g| dm<br />

A<br />

A<br />

≤ ɛ + f − fn1 + mA sup |fn(x) − g(x)|,<br />

x∈A<br />

30 Note that Pro<strong>of</strong> 1, which seems to me the more natural one, doesn’t use Egor<strong>of</strong>f’s theorem, so either the exam<strong>in</strong>ers were look<strong>in</strong>g for a different<br />

pro<strong>of</strong>, or a different conjecture, or perhaps Egor<strong>of</strong>f’s theorem was not the Littlewood pr<strong>in</strong>ciple they had <strong>in</strong> m<strong>in</strong>d. In any event, I have found a way<br />

to prove the conjecture which does make use <strong>of</strong> Egor<strong>of</strong>f’s theorem, <strong>and</strong> this appears here as Pro<strong>of</strong> 2.<br />

31 Foll<strong>and</strong> [4], theorem 2.30 <strong>and</strong> its corollary.<br />

32 Perhaps this statement is the version <strong>of</strong> the Littlewood pr<strong>in</strong>ciple deal<strong>in</strong>g with a.e. convergence that we were meant to cite <strong>in</strong> part (a).<br />

33<br />

A


1.7 2007 November 16 1 REAL ANALYSIS<br />

where f − fn1 → 0, by assumption, <strong>and</strong> supx∈A |fn(x) − g(x)| → 0 s<strong>in</strong>ce fn → g uniformly on A. S<strong>in</strong>ce ɛ > 0<br />

was arbitrary, it follows that n<br />

−n |f − g| dm = 0 <strong>and</strong>, for functions f, g ∈ L1 ([−n, n], m), this implies that f = g<br />

a.e. on [−n, n].<br />

It rema<strong>in</strong>s to show that f, g ∈ L1 ([−n, n], m). It’s clear that f ∈ L1 s<strong>in</strong>ce f1 ≤ f − fn1 + fn1 < ∞. To<br />

a.e.<br />

prove g ∈ L1 ([−n, n], m) note that fn −−→ g implies limn |fn(x)| = |g(x)| for almost all x, so by Fatou’s lemma,<br />

<br />

<br />

g1 = |g| dm = lim |fn| dm ≤ lim |fn| dm = lim fn1 = f1 < ∞.<br />

The last equality holds because, by the triangle <strong>in</strong>equality, |fn1 − f1| ≤ fn − f1 → 0. ⊓⊔<br />

3. Let K be a compact subset <strong>in</strong> R 3 <strong>and</strong> let f(x) = dist(x, K).<br />

(a) Prove that f is a cont<strong>in</strong>uous function <strong>and</strong> that f(x) = 0 if <strong>and</strong> only if x ∈ K.<br />

<br />

n (b) Let g = max(1 − f, 0) <strong>and</strong> prove that limn→∞ g exists <strong>and</strong> is equal to m3(K).<br />

Solution: (a) Def<strong>in</strong>e dist(x, K) = f(x) = <strong>in</strong>fk∈K |x − k|. Clearly, for all k ∈ K, f(x) ≤ |x − k|. Therefore, by<br />

the triangle <strong>in</strong>equality, for any x, y ∈ R 3 ,<br />

<strong>and</strong> so, tak<strong>in</strong>g the <strong>in</strong>fimum over k ∈ K on the right,<br />

Similarly,<br />

f(x) ≤ |x − y| + |y − k|, ∀k ∈ K,<br />

f(x) ≤ |x − y| + f(y). (26)<br />

f(y) ≤ |x − y| + f(x). (27)<br />

Obviously, for any given x ∈ R 3 , f(x) is f<strong>in</strong>ite. Therefore, (26) <strong>and</strong> (27) together imply that<br />

Whence f is (Lipschitz) cont<strong>in</strong>uous.<br />

|f(x) − f(y)| ≤ |x − y|, ∀ x, y ∈ R 3 .<br />

Now, if x ∈ K, then it’s clear that f(x) = 0. Suppose x /∈ K; that is, x is <strong>in</strong> the complement K c <strong>of</strong> K. S<strong>in</strong>ce K is<br />

closed, K c is open <strong>and</strong> we can f<strong>in</strong>d an ɛ-neighborhood about x fully conta<strong>in</strong>ed <strong>in</strong> K c , <strong>in</strong> which case f(x) > ɛ. We<br />

have thus proved that f(x) = 0 if <strong>and</strong> only if x ∈ K. ⊓⊔<br />

(b) First observe that f(x) = 0 for all x ∈ K, <strong>and</strong> f(x) > 0 for all x /∈ K. Def<strong>in</strong>e K1 to be a closed <strong>and</strong> bounded<br />

set conta<strong>in</strong><strong>in</strong>g K on which f(x) ≤ 1. That is, K1 is the set <strong>of</strong> po<strong>in</strong>ts that are a distance <strong>of</strong> not more than 1 unit<br />

from the set K. In particular K ⊂ K1. Notice that g = max(1 − f, 0) = (1 − f)χK1 . Also, if x ∈ K1 \ K,<br />

then 0 ≤ 1 − f(x) < 1, so g n → 0 on the set K1 \ K, while on the set K, g n = 1 for all n ∈ N. Therefore,<br />

g n → χK. F<strong>in</strong>ally, note that g n ≤ χK1 ∈ L 1 (R 3 ) so the dom<strong>in</strong>ated convergence theorem can be applied to yield<br />

limn→∞<br />

g n = χK = m3(K). ⊓⊔<br />

4. Let E be a Borel subset <strong>of</strong> R 2 .<br />

(a) Expla<strong>in</strong> what this means.<br />

(b) Suppose that for every real number t the set Et = {(x, y) ∈ E | x = t} is f<strong>in</strong>ite. Prove that E is a Lebesgue<br />

null set.<br />

34


1.7 2007 November 16 1 REAL ANALYSIS<br />

Solution:<br />

(a) The Borel σ-algebra <strong>of</strong> R 2 , which we denote by B(R 2 ), is the smallest σ-algebra that conta<strong>in</strong>s the open subsets<br />

<strong>of</strong> R 2 . The sets belong<strong>in</strong>g to B(R 2 ) are called Borel subsets <strong>of</strong> R 2 .<br />

(b) First observe that if G is a f<strong>in</strong>ite subset <strong>of</strong> R, then G is a Lebesgue null set. That is, mG = 0. In fact, it is easy<br />

to prove that if G is any countable subset, then mG = 0. (Just fix ɛ > 0 <strong>and</strong> cover each po<strong>in</strong>t xn ∈ G with a set<br />

En <strong>of</strong> measure less than ɛ2 −n . Then mG ≤ mEn < ɛ.)<br />

In problems <strong>in</strong>volv<strong>in</strong>g 2-dimensional Lebesgue measure, dist<strong>in</strong>guish<strong>in</strong>g x <strong>and</strong> y coord<strong>in</strong>ates sometimes clarifies<br />

th<strong>in</strong>gs. To wit, let (X, B(X), µ) = (Y, B(Y ), ν) be two identical copies <strong>of</strong> the measure space (R, B(R), m), <strong>and</strong><br />

represent Lebesgue measure on R 2 by 33 (X × Y, B(X) ⊗ B(Y ), µ × ν) = (R 2 , B(R 2 ), m2).<br />

Our goal is to prove that m2E = 0. First note that<br />

<br />

m2E = (µ × ν)(E) =<br />

X×Y<br />

χE d(µ × ν).<br />

The <strong>in</strong>tegr<strong>and</strong> χE is non-negative <strong>and</strong> measurable (s<strong>in</strong>ce E is Borel). Therefore, by Tonelli’s theorem (A.13),<br />

<br />

<br />

m2E = χE(x, y) dµ(x) dν(y) = χE(x, y) dν(y) dµ(x). (28)<br />

Now, let<br />

Y<br />

X<br />

Gt = {y ∈ R : (x, y) ∈ E <strong>and</strong> x = t}.<br />

This is the so called “x-section” <strong>of</strong> E at the po<strong>in</strong>t x = t. It is a subset <strong>of</strong> R, but we can view it as a subset <strong>of</strong> R2 by<br />

simply identify<strong>in</strong>g each po<strong>in</strong>t y ∈ Gt with the po<strong>in</strong>t (t, y) ∈ Et = {(x, y) ∈ E : x = t}. It follows that, for each<br />

t ∈ R, Gt is a f<strong>in</strong>ite subset <strong>of</strong> R. Therefore, mGt = 0. F<strong>in</strong>ally, by (28),<br />

<br />

<br />

m2E = χGt (y) dν(y) dµ(t) = νGt dµ(t) = 0.<br />

X<br />

Y<br />

s<strong>in</strong>ce νGt mGt = 0. ⊓⊔<br />

5. Let µ <strong>and</strong> ν be f<strong>in</strong>ite positive measures on the measurable space (X, A) such that ν ≪ µ ≪ ν, <strong>and</strong> let dν<br />

d(µ+ν)<br />

represent the Radon-Nikodym derivative <strong>of</strong> ν with respect to µ + ν. Show that<br />

0 <<br />

X<br />

Y<br />

X<br />

dν<br />

< 1 a.e. [µ].<br />

d(µ + ν)<br />

Solution: First note that ν ≪ µ implies ν ≪ µ + ν, so, by the Radon-Nikodym theorem (A.12), there is a unique<br />

f ∈ L1 (µ + ν) such that<br />

<br />

ν(E) = f d(µ + ν) ∀E ∈ A.<br />

E<br />

Indeed, f is the Radon-Nikodym derivative; i.e., f = dν<br />

d(µ+ν) . We want to show 0 < f(x) < 1 holds for µ-almost<br />

every x ∈ X. S<strong>in</strong>ce we’re deal<strong>in</strong>g with positive measures, we can assume f(x) ≥ 0 for all x ∈ X.<br />

If B0 = {x ∈ X : f(x) = 0}, then<br />

<br />

ν(B0) =<br />

B0<br />

f d(µ + ν) = 0.<br />

33 The notation B(X) ⊗ B(Y ) denotes the σ-algebra generated by all sets A × B ⊆ X × Y with A ∈ B(X) <strong>and</strong> B ∈ B(Y ). See A.1.7. In<br />

this case, B(X) ⊗ B(Y ) is the same as B(R 2 ).<br />

35


1.7 2007 November 16 1 REAL ANALYSIS<br />

Therefore, µ(B0) = 0, s<strong>in</strong>ce µ ≪ ν, which proves that f(x) > 0, [µ]-a.e.<br />

If B1 = {x ∈ X : f(x) ≥ 1}, then<br />

<br />

ν(B1) =<br />

B1<br />

f d(µ + ν) ≥ (µ + ν)(B1) = µ(B1) + ν(B1).<br />

S<strong>in</strong>ce ν is f<strong>in</strong>ite by assumption, we can subtract ν(B0) from both sides to obta<strong>in</strong> µ(B1) = 0. This proves f(x) < 1,<br />

[µ]-a.e. ⊓⊔<br />

6. 34 Suppose that 1 < p < ∞ <strong>and</strong> that q = p/(p − 1).<br />

(a) Let a1, a2, . . . be a sequence <strong>of</strong> real numbers for which the series anbn converges for all real sequences {bn}<br />

satisfy<strong>in</strong>g the condition |bn| q < ∞. Prove that |an| p < ∞.<br />

(b) Discuss the cases <strong>of</strong> p = 1 <strong>and</strong> p = ∞. Prove your assertions.<br />

Solution: (a) For each k ∈ N, def<strong>in</strong>e Tk : ℓq(N) → R by Tk(b) = k<br />

n=1 anbn, for b ∈ ℓq(N). Then {Tk} is a<br />

family <strong>of</strong> po<strong>in</strong>twise bounded l<strong>in</strong>ear functionals. That is, each Tk is a l<strong>in</strong>ear functional, <strong>and</strong>, for each b ∈ ℓq(N),<br />

there is an Mb ≥ 0 such that |Tk(b)| ≤ Mb holds for all k ∈ N. To see this, simply note that a convergent sequence<br />

<strong>of</strong> real numbers is bounded, <strong>and</strong>, <strong>in</strong> the present case, we have<br />

Sk <br />

k<br />

∞<br />

anbn → anbn = x ∈ R.<br />

n=1<br />

Thus, {Tk(b)} = {Sk} is a convergent sequence <strong>of</strong> reals, so, if N ∈ N is such that k ≥ N implies |Sk − x| < 1,<br />

<strong>and</strong> if M ′ b is def<strong>in</strong>ed to be max{|Sk| : 1 ≤ k ≤ N}, then, for any k ∈ N,<br />

n=1<br />

|Tk(b)| ≤ Mb max{M ′ b, x + 1}.<br />

Next note that ℓq is a Banach space, so the (Banach-Ste<strong>in</strong>hauss) pr<strong>in</strong>ciple <strong>of</strong> uniform boundedness implies that<br />

there is a s<strong>in</strong>gle M > 0 such that Tk ≤ M for all k ∈ N. In other words,<br />

(∃ M > 0) (∀b ∈ ℓq) (∀k ∈ N) |Tk(b)| ≤ Mb.<br />

Def<strong>in</strong>e let T (b) ∞<br />

n=1 anbn = limk→∞ Tk(b), which exists by assumption. S<strong>in</strong>ce |·| is cont<strong>in</strong>uous, we conclude<br />

that limk→∞ |Tk(b)| = |T (b)|. F<strong>in</strong>ally, s<strong>in</strong>ce |Tk(b)| ≤ Mb for all k ∈ N, we have |T (b)| ≤ Mb. That is T is<br />

a bounded l<strong>in</strong>ear functional on ℓq(N).<br />

Now, by the Riesz representation theorem, if 1 ≤ q < ∞, then any bounded l<strong>in</strong>ear functional T ∈ ℓ ∗ q is uniquely<br />

representable by some α = (α1, α2, . . . ) ∈ ℓp as<br />

T (b) =<br />

∞<br />

αnbn. (29)<br />

n=1<br />

On the other h<strong>and</strong>, by def<strong>in</strong>ition, T (b) = ∞<br />

n=1 anbn, for all b ∈ ℓq. S<strong>in</strong>ce the representation <strong>in</strong> (29) is unique,<br />

a = α ∈ ℓp. That is, <br />

n |an| p < ∞. ⊓⊔<br />

34 On the orig<strong>in</strong>al exam this question asked only about the special case p = q = 2.<br />

36


1.7 2007 November 16 1 REAL ANALYSIS<br />

(b) Consider the case p = 1 <strong>and</strong> q = ∞. First recall that the Riesz representation theorem says that every<br />

T ∈ ℓ ∗ q (1 ≤ q < ∞) is uniquely representable by some α ∈ ℓp (where p = q/(q − 1), so 1 < p ≤ ∞). That is ℓp<br />

is the dual <strong>of</strong> ℓq, when 1 ≤ q < ∞ <strong>and</strong> p = q/(q − 1). However, <strong>in</strong> the present case we have q = ∞ <strong>and</strong> p = 1<br />

<strong>and</strong> ℓ1 is not the dual <strong>of</strong> ℓ∞. (Perhaps the easiest way to see this is to note that ℓ1 is separable but ℓ∞ is not. For<br />

the collection <strong>of</strong> a ∈ ℓ∞ such that an ∈ {0, 1}, n ∈ N, is uncountable <strong>and</strong>, for any two dist<strong>in</strong>ct such sequences<br />

a, b ∈ {0, 1} N , we have a − b∞ = 1, so there cannot be a countable base, so ℓ∞ is not second countable, <strong>and</strong> a<br />

metric space is separable iff it is second countable.)<br />

So we can’t use the same method <strong>of</strong> pro<strong>of</strong> for this case. However, I believe the result still holds by the follow<strong>in</strong>g<br />

simple argument: Def<strong>in</strong>e b = (b1, b2, . . . ) by<br />

<br />

ān/|an|, for an = 0,<br />

bn = sgn(an) =<br />

(n ∈ N).<br />

0, for an = 0,<br />

Then <br />

n |an| = <br />

n anbn converges by the hypothesis, s<strong>in</strong>ce |bn| ∈ {0, 1} implies b ∈ ℓ∞. Therefore, a ∈ ℓ1.<br />

F<strong>in</strong>ally, <strong>in</strong> case p = ∞ <strong>and</strong> q = 1, the Riesz representation theorem can be applied as <strong>in</strong> part (a). ⊓⊔<br />

Please email comments, suggestions, <strong>and</strong> corrections to williamdemeo@gmail.com.<br />

37


2 Complex Analysis<br />

38<br />

2 COMPLEX ANALYSIS


2.1 1989 April 2 COMPLEX ANALYSIS<br />

2.1 1989 April<br />

INSTRUCTIONS: Do at least four problems.<br />

TIME LIMIT: 1.5 hours 35<br />

1. (a) Let U be the unit disk <strong>in</strong> the complex plane C, U = {z ∈ C : |z| < 1}, <strong>and</strong> let f be an analytic function <strong>in</strong> a<br />

neighborhood <strong>of</strong> the closure <strong>of</strong> U. Show that if f is real on all the boundary <strong>of</strong> U, then f must be constant.<br />

(b) Let u be a real harmonic function <strong>in</strong> all the complex plane C. Show that if u(z) ≥ 0 for all z ∈ C, then<br />

u must be constant.<br />

Solution: (a) The hypotheses imply that Imf(eiθ ) = 0 for all θ ∈ R. S<strong>in</strong>ce f is holomorphic <strong>in</strong> a neighborhood<br />

Ω <strong>of</strong> U, the series<br />

∞<br />

f(z) = anz n<br />

n=0<br />

converges uniformly on any compact subset <strong>of</strong> Ω. The unit circle T = {z : |z| = 1} = {eiθ : θ ∈ R} is one such<br />

compact subset, <strong>and</strong> here the series is<br />

f(e iθ ∞<br />

) = ane <strong>in</strong>θ .<br />

If we write the coefficients as an = cn + ibn, where cn, bn ∈ R, then we have<br />

n=0<br />

ane <strong>in</strong>θ = (cn + ibn)[cos(nθ) + i s<strong>in</strong>(nθ)] = [cn cos(nθ) − bn s<strong>in</strong>(nθ)] + i[cn s<strong>in</strong>(nθ) + bn cos(nθ)].<br />

Thus, by the hypothesis, the series<br />

Imf(e iθ ) =<br />

∞<br />

[cn s<strong>in</strong>(nθ) + bn cos(nθ)]<br />

n=0<br />

converges uniformly to zero for all θ ∈ [0, 2π]. Therefore, with the possible exception <strong>of</strong> c0, we have cn = bn = 0,<br />

for all n, so f ≡ c0. ⊓⊔<br />

(b) S<strong>in</strong>ce C is simply connected, there is a real-valued harmonic conjugate v(z) such that the function f(z) =<br />

u(z) + iv(z) is entire. Now, s<strong>in</strong>ce u(z) ≥ 0, f(z) maps the complex plane <strong>in</strong>to the right half-plane, {Ref(z) ≥ 0}.<br />

It follows immediately from Picard’s theorem that f must be constant. 36 However, an elementary argument us<strong>in</strong>g<br />

only Liouville’s theorem is probably preferable, so let f be as above, <strong>and</strong> def<strong>in</strong>e g(z) = f(z) + 1. Then g is<br />

entire <strong>and</strong> maps C <strong>in</strong>to {w ∈ C : Rew ≥ 1}. In particular, g(z) is bounded away from zero, so the function<br />

h(z) = 1/g(z) is a bounded entire function. (In fact, |h(z)| ≤ 1.) Therefore, by Liouville’s theorem, h is constant,<br />

so f is constant, so u = Ref is constant. ⊓⊔<br />

2. Let f be an analytic function <strong>in</strong> the region {z : |z| > 1}, <strong>and</strong> suppose that<br />

lim f(z) = 0.<br />

z→∞<br />

35In 1989 there was a s<strong>in</strong>gle three hour test cover<strong>in</strong>g both real <strong>and</strong> complex analysis. Students were required to do n<strong>in</strong>e problems, with at least<br />

four from each part.<br />

36Picard’s theorem states that a non-constant entire function can omit at most one value <strong>of</strong> C from its range. This is a very powerful theorem, but<br />

if you use it for an easy problem like this one, you might be accused <strong>of</strong> kill<strong>in</strong>g a fly with a sledge hammer!<br />

39


2.1 1989 April 2 COMPLEX ANALYSIS<br />

Show that if |z| > 2, then<br />

Solution: By Cauchy’s formula, if |z| < R, then<br />

f(z) = 1<br />

<br />

2πi |ζ|=R<br />

<br />

1 f(ζ)<br />

dζ = −f(z). (30)<br />

2πi |ζ|=2 ζ − z<br />

<br />

f(ζ) 1<br />

dζ −<br />

ζ − z 2πi |ζ|=2<br />

f(ζ)<br />

dζ. (31)<br />

ζ − z<br />

Note that this holds for all R > |z| > 2. Fix ɛ > 0. Let Rɛ be such that |f(ζ)| < ɛ/2 for all |ζ| = Rɛ <strong>and</strong><br />

Rɛ > 2|z|. Then |ζ − z| > Rɛ/2 for all |ζ| = Rɛ, so<br />

<br />

1 |f(ζ)| ɛ/2 2πRɛ<br />

|dζ| < = ɛ.<br />

2π |ζ|=Rɛ |ζ − z| 2π Rɛ/2<br />

Therefore, by (31), <br />

<br />

1<br />

2πi<br />

|ζ|=2<br />

f(ζ)<br />

ζ − z<br />

<br />

<br />

<br />

dζ + f(z) < ɛ.<br />

<br />

This holds for any ɛ, which proves (30). ⊓⊔<br />

3. 37 Let U be the open unit disk <strong>in</strong> C, <strong>and</strong> let U + be the top half <strong>of</strong> this disk,<br />

U + = {z ∈ C : Imz > 0, |z| < 1}.<br />

Exhibit a one-to-one conformal mapp<strong>in</strong>g from U + onto U.<br />

Solution: Consider ϕ0(z) = 1−z<br />

1+z , a l<strong>in</strong>ear fractional transformation which takes U onto the right half-plane<br />

Ω = {z ∈ C : Rez > 0}. (This property <strong>of</strong> ϕ0 can be seen by consider<strong>in</strong>g ϕ0(0) = 1 <strong>and</strong> ϕ0(∞) = −1 <strong>and</strong><br />

argu<strong>in</strong>g by symmetry.)<br />

Note that ϕ0(1) = 0 <strong>and</strong> ϕ0(x) ∈ R, for all x ∈ R. Also, ϕ0 is conformal, so it preserves the right angle formed<br />

by the <strong>in</strong>tersection <strong>of</strong> the circle <strong>and</strong> the real axis at the po<strong>in</strong>t z = 1. Therefore, ϕ0 takes U + onto either the first<br />

quadrant, Ω + = {z ∈ Ω : Imz > 0}, or the fourth quadrant, Ω − = {z ∈ Ω : Imz < 0}. To see which, consider<br />

z = i/2.<br />

Thus, ϕ0 : U + → Ω − .<br />

ϕ0(i/2) =<br />

1 − i/2<br />

1 + i/2 =<br />

1 − i/2<br />

1 + i/2<br />

<br />

1 − i/2<br />

=<br />

1 − i/2<br />

3 − 4i<br />

∈ Ω<br />

5<br />

− .<br />

Let ϕ1(z) = iz, so that ϕ1 : Ω− → Ω + , let ϕ2(z) = z2 , so that ϕ2 : Ω + → {Imz > 0}, <strong>and</strong> let ϕ3(z) = −iz,<br />

so that ϕ3 : {Imz > 0} → Ω = {Rez > 0}. F<strong>in</strong>ally, note that ϕ −1<br />

0 (z) = ϕ0(z) maps Ω onto U. Putt<strong>in</strong>g it all<br />

together, we see that a map satisfy<strong>in</strong>g the requirements is<br />

37 This problem also appears <strong>in</strong> April ’95 (5) <strong>and</strong> November ’06 (2).<br />

ϕ(z) = (ϕ0 ◦ ϕ3 ◦ ϕ2 ◦ ϕ1 ◦ ϕ0)(z).<br />

40<br />

⊓⊔


2.1 1989 April 2 COMPLEX ANALYSIS<br />

4. Let {fn} be a sequence <strong>of</strong> analytic functions <strong>in</strong> the unit disk U, <strong>and</strong> suppose there exists a constant M such that<br />

<br />

|fn(z)| |dz| ≤ M<br />

C<br />

for each fn <strong>and</strong> for all circles C ly<strong>in</strong>g <strong>in</strong> U. Prove that {fn} has a subsequence converg<strong>in</strong>g uniformly on compact<br />

subsets <strong>of</strong> U.<br />

Solution: See the solution to problem 7 <strong>of</strong> November ’91.<br />

5. Let Q be a complex polynomial with dist<strong>in</strong>ct simple roots at the po<strong>in</strong>ts a1, a2, . . . , an, <strong>and</strong> let P be a complex<br />

polynomial <strong>of</strong> degree less than that <strong>of</strong> Q. Show that<br />

P (z)<br />

Q(z) =<br />

n<br />

k=1<br />

P (ak)<br />

Q ′ (ak)(z − ak) .<br />

6. Use contour <strong>in</strong>tegration <strong>and</strong> the residue method to evaluate the <strong>in</strong>tegral<br />

∞<br />

0<br />

cos x<br />

(1 + x2 dx.<br />

) 2<br />

Solution: Denote the <strong>in</strong>tegral by I. S<strong>in</strong>ce the <strong>in</strong>tegr<strong>and</strong> is even,<br />

2I =<br />

∞<br />

−∞<br />

cos x<br />

(1 + x2 dx.<br />

) 2<br />

Consider the simple closed contour ΓR = γR ∪ [−R, R], where the trace <strong>of</strong> γR is the set {Re iθ : 1 ≤ θ ≤ π},<br />

oriented counter-clockwise. Note that, if R > 1, then i is <strong>in</strong>side the region bounded by ΓR.<br />

The function<br />

f(z) =<br />

cos z<br />

(1 + z2 cos z<br />

=<br />

) 2 (z + i) 2 (z − i) 2<br />

is holomorphic <strong>in</strong>side <strong>and</strong> on ΓR, except for a double pole at z = i, where the residue is computed as follows:<br />

Res(f, i) = lim<br />

z→i<br />

d<br />

dz [(z − i)2 f(z)] = lim<br />

z→i<br />

d<br />

dz<br />

cos z<br />

(z + i) 2<br />

−(z + i)<br />

= lim<br />

z→i<br />

2 s<strong>in</strong> z − 2(z + i) cos z<br />

(z + i) 4<br />

= −(2i)2 s<strong>in</strong>(i) − 4i cos(i)<br />

(2i) 4<br />

= s<strong>in</strong>(i) − i cos(i)<br />

4<br />

= −iei·i<br />

4<br />

= −i<br />

4e .<br />

By the residue theorem, it follows that, for all R > 1,<br />

<br />

f(z) dz = 2πi Res(f, i) = π<br />

2e .<br />

ΓR<br />

41


2.1 1989 April 2 COMPLEX ANALYSIS<br />

It rema<strong>in</strong>s to check that <br />

2I =<br />

∞<br />

−∞<br />

Indeed, <br />

γR<br />

f(z) dz → 0, as R → ∞, which will allow us to conclude that<br />

<br />

f(x) dx = lim<br />

R→∞<br />

ΓR<br />

<br />

f(z) dz −<br />

<br />

<br />

f(z) dz<br />

<br />

γR<br />

=<br />

<br />

<br />

<br />

cos z<br />

<br />

γR (1 + z2 <br />

<br />

dz<br />

) 2 ≤<br />

This <strong>in</strong>equality holds for all R > 1, so, lett<strong>in</strong>g R → ∞, we have <br />

I =<br />

∞<br />

0<br />

γR<br />

<br />

f(z) dz = lim f(z) dz =<br />

R→∞ ΓR<br />

π<br />

. (32)<br />

2e<br />

1<br />

(R2 − 1) 2 ℓ(γR)<br />

πR<br />

=<br />

(R2 .<br />

− 1) 2<br />

γR<br />

cos x<br />

(1 + x2 π<br />

dx =<br />

) 2 4e .<br />

42<br />

f(z) dz → 0. Therefore, by (32),<br />

⊓⊔


2.2 1991 November 21 2 COMPLEX ANALYSIS<br />

2.2 1991 November 21<br />

INSTRUCTIONS: In each <strong>of</strong> sections A, B, <strong>and</strong> C, do all but one problem.<br />

TIME LIMIT: 2 hours<br />

1. Where does the function<br />

SECTION A<br />

(Do 3 <strong>of</strong> the 4 problems.)<br />

f(z) = zRez + ¯zImz + ¯z<br />

have a complex derivative? Compute the derivative wherever it exists.<br />

Solution: Writ<strong>in</strong>g f <strong>in</strong> terms <strong>of</strong> the real <strong>and</strong> imag<strong>in</strong>ary parts <strong>of</strong> z = x + iy, we have<br />

f(x + iy) = (x + iy)x + (x − iy)y + x − iy<br />

= x 2 + xy + x + i(xy − y 2 − y)<br />

= u(x, y) + iv(x, y),<br />

where u(x, y) = x 2 + xy + x <strong>and</strong> v(x, y) = xy − y 2 − y are the real <strong>and</strong> imag<strong>in</strong>ary parts <strong>of</strong> f. Therefore,<br />

ux = 2x + y + 1 vy = x − 2y − 1 (33)<br />

uy = x vx = y. (34)<br />

If f is holomorphic <strong>in</strong> some region, the Cauchy-Riemann equations (ux = vy, uy = −vx) must hold there. By (33)<br />

<strong>and</strong> (34), this requires 2x + y + 1 = x − 2y − 1 <strong>and</strong> x = −y. Substitut<strong>in</strong>g the second equation <strong>in</strong>to the first yields<br />

−y + 1 = −3y − 1, or y = −1. Then, s<strong>in</strong>ce x = −y, we must have x = 1. Therefore, f has a complex derivative<br />

at (x, y) = (1, −1), or z = 1 − i.<br />

For any region Ω ⊆ C, we def<strong>in</strong>e the l<strong>in</strong>ear functional ∂ : H(Ω) → C by ∂ = 1<br />

2<br />

f ∈ H(Ω), then the derivative <strong>of</strong> f is given by f ′ (z) = (∂f)(z), z ∈ Ω. In the present case,<br />

∂f<br />

= 2x + y + 1 + iy,<br />

∂x<br />

∂f<br />

∂y<br />

= x + i(x − 2y − 1).<br />

<br />

∂ ∂<br />

∂x − i ∂y , <strong>and</strong> recall that, if<br />

Therefore, ∂f(x + iy) = 1<br />

1<br />

2 [(2x + y + 1 + iy) − i(x + i(x − 2y − 1))] = 2 [(3x − y) + i(y − x)], <strong>and</strong> f<strong>in</strong>ally,<br />

f ′ (1 − i) = 1<br />

(4 − 2i) = 2 − i.<br />

2<br />

2. (a) Prove that any nonconstant polynomial with complex coefficients has at least one root.<br />

(b) From (a) it follows that every nonconstant polynomial P has the factorization<br />

P (z) = a<br />

N<br />

(z − λn),<br />

n=1<br />

43<br />

⊓⊔


2.2 1991 November 21 2 COMPLEX ANALYSIS<br />

where a <strong>and</strong> each root λn are complex constants. Prove that if P has only real coefficients, then P has a factorization<br />

K<br />

M<br />

P (z) = a (z − rk) (z 2 − bmz + cm),<br />

k=1<br />

where a <strong>and</strong> each rk, bm, cm are real constants.<br />

m=1<br />

3. Use complex residue methods to compute the <strong>in</strong>tegral<br />

π<br />

1<br />

5 + 3 cos θ dθ.<br />

Solution: Let I = π<br />

0<br />

For z = e iθ ,<br />

0<br />

1<br />

5+3 cos θ dθ. Note that cos θ is an even function (i.e., cos(−θ) = cos θ), so<br />

2I =<br />

<strong>and</strong> dz = ieiθdθ, from which it follows that<br />

<br />

2I =<br />

π<br />

−π<br />

cos θ = eiθ + e −iθ<br />

2<br />

= 1<br />

i<br />

= 2<br />

3i<br />

|z|=1<br />

<br />

|z|=1<br />

<br />

|z|=1<br />

1<br />

5 + 3 cos θ dθ.<br />

= 1 1<br />

(z +<br />

2 z ),<br />

1<br />

5 + 3 1<br />

2 (z + z )<br />

dz<br />

iz<br />

dz<br />

5z + 3<br />

2 (z2 + 1)<br />

dz<br />

z2 + 10<br />

3 z + 1.<br />

Let p(z) = z 2 + 10<br />

3 z + 1. Then the roots <strong>of</strong> p(z) are z1 = −1/3 <strong>and</strong> z2 = −3. Only z1 = −1/3 is <strong>in</strong>side the circle<br />

|z| = 1, so the residue theorem implies<br />

Now,<br />

which implies<br />

Therefore,<br />

2I = 2<br />

<br />

1<br />

· 2πi · Res , z1 .<br />

3i p(z)<br />

<br />

1<br />

Res , z1 = lim<br />

p(z) z→z1<br />

1<br />

p(z) =<br />

1<br />

(z − z1)(z − z2) ,<br />

1<br />

=<br />

z − z2<br />

1<br />

− 1<br />

3<br />

=<br />

3 − (−3) 8 .<br />

2I = 2 3 π<br />

· 2πi · =<br />

3i 8 2 ,<br />

so I = π<br />

4 . ⊓⊔<br />

44


2.2 1991 November 21 2 COMPLEX ANALYSIS<br />

4. (a) Expla<strong>in</strong> how to map an <strong>in</strong>f<strong>in</strong>ite strip (i.e., the region strictly between two parallel l<strong>in</strong>es) onto the unit disk by a<br />

one-to-one conformal mapp<strong>in</strong>g.<br />

(b) Two circles lie outside one another except for common po<strong>in</strong>t <strong>of</strong> tangency. Expla<strong>in</strong> how to map the region<br />

exterior to both circles (<strong>in</strong>clud<strong>in</strong>g the po<strong>in</strong>t at <strong>in</strong>f<strong>in</strong>ity) onto an <strong>in</strong>f<strong>in</strong>ite strip by a one-to-one conformal mapp<strong>in</strong>g.<br />

SECTION B<br />

(Do 3 <strong>of</strong> the 4 problems.)<br />

5. 38 Suppose that f is analytic <strong>in</strong> the annulus 1 < |z| < 2, <strong>and</strong> that there exists a sequence <strong>of</strong> polynomials converg<strong>in</strong>g<br />

to f uniformly on every compact subset <strong>of</strong> this annulus. Show that f has an analytic extension to all <strong>of</strong> the disk<br />

|z| < 2.<br />

Solution: Note that the function f, be<strong>in</strong>g holomorphic <strong>in</strong> the annulus 1 < |z| < 2, has Laurent series representation<br />

f(z) =<br />

∞<br />

an(z − z0) n ,<br />

n=−∞<br />

converg<strong>in</strong>g locally uniformly for 1 < |z| < 2, where z0 is any po<strong>in</strong>t <strong>in</strong> the disk |z| < 2. I claim that an = 0 for all<br />

negative <strong>in</strong>tegers n. To see this, first recall the formula for the coefficients <strong>in</strong> the Laurent series,<br />

an = 1<br />

<br />

f(z)<br />

dz, (n ∈ Z; 1 < R < 2).<br />

2πi (z − z0) n+1<br />

|z|=R<br />

Let {pm} be the sequence <strong>of</strong> polynomials mentioned <strong>in</strong> the problem statement. Of course, pm ∈ H(C), so Cauchy’s<br />

theorem implies <br />

|z|=R pm(z) dz = 0, <strong>and</strong>, more generally,<br />

<br />

pm(z)(z − z0) −n−1 dz = 0, (n = −1, −2, . . . ).<br />

Therefore,<br />

|z|=R<br />

|an| = 1<br />

<br />

<br />

<br />

<br />

f(z)<br />

pm(z)<br />

<br />

<br />

<br />

dz −<br />

dz<br />

2π |z|=R (z − z0) n+1<br />

|z|=R (z − z0) n+1 <br />

<br />

≤ 1<br />

<br />

|f(z) − pm(z)|<br />

2π |z|=R |z − z0| n+1 |dz|. (35)<br />

F<strong>in</strong>ally, pm → f uniformly on |z| = R, so (35) implies |an| = 0 for n = −1, −2, . . . . This proves that f(z) =<br />

∞<br />

n=0 an(z − z0) n , converg<strong>in</strong>g locally uniformly <strong>in</strong> |z| < 2. Whence f ∈ H(|z| < 2). ⊓⊔<br />

6. Let f be analytic <strong>in</strong> |z| < 2, with the only zeros <strong>of</strong> f be<strong>in</strong>g the dist<strong>in</strong>ct po<strong>in</strong>ts a1, a2, . . . , an, <strong>of</strong> multiplicities<br />

m1, m2, . . . , mn, respectively, <strong>and</strong> with each aj ly<strong>in</strong>g <strong>in</strong> the disk |z| < 1. Given that g is analytic <strong>in</strong> |z| < 2, what<br />

is <br />

(Verify your answer.)<br />

38 See also: April ’96 (8).<br />

|z|=1<br />

f ′ (z)g(z)<br />

f(z)<br />

45<br />

dz ?


2.2 1991 November 21 2 COMPLEX ANALYSIS<br />

7. Let {fn} be a sequence <strong>of</strong> analytic functions <strong>in</strong> the unit disk D, <strong>and</strong> suppose there exists a positive constant M<br />

such that <br />

|fn(z)| |dz| ≤ M<br />

C<br />

for each fn <strong>and</strong> for every circle C ly<strong>in</strong>g <strong>in</strong> D. Prove that {fn} has a subsequence converg<strong>in</strong>g uniformly on compact<br />

subsets <strong>of</strong> D.<br />

Solution: We must show that F = {fn} is a normal family. If we can prove that F is a locally bounded family<br />

<strong>of</strong> holomorphic functions – that is, F ⊂ H(D) <strong>and</strong>, for any compact set K ⊂ D, there is an MK > 0 such that<br />

|fn(z)| ≤ MK for all z ∈ K <strong>and</strong> all n = 1, 2, . . . – then the Montel theorem (corollary 2.2) will give the desired<br />

result.<br />

To show F is locally bounded, it is equivalent to show that, for each po<strong>in</strong>t zα ∈ D, there is a number Mα <strong>and</strong> a<br />

neighborhood B(zα, rα) ⊂ D such that |fn(z)| ≤ Mα for all z ∈ B(zα, rα) <strong>and</strong> all n = 1, 2, . . .. (Why is this<br />

equivalent?) 39 So, fix zα ∈ D. Let Rα > 0 be such that ¯ B(zα, Rα) = {z ∈ C : |z − zα| ≤ Rα} ⊂ D. Then, for<br />

any z ∈ B(zα, Rα/2), Cauchy’s formula gives<br />

|fn(z)| ≤ 1<br />

<br />

|fn(ζ)|<br />

2π |ζ−zα|=Rα |ζ − z| |dζ|<br />

≤ 1 1<br />

2π Rα/2<br />

≤ M<br />

.<br />

πRα<br />

<br />

|ζ−zα|=Rα<br />

|fn(ζ)| |dζ|<br />

The second <strong>in</strong>equality holds s<strong>in</strong>ce |ζ − zα| = Rα <strong>and</strong> |z − zα| < Rα/2 imply |ζ − z| > Rα/2. The last <strong>in</strong>equality<br />

follows from the hypothesis <br />

C |fn(z)| |dz| ≤ M for any circle C <strong>in</strong> D. Lett<strong>in</strong>g Mα = M<br />

πRα , <strong>and</strong> rα = Rα/2, we<br />

have |fn(z)| ≤ Mα for all z ∈ B(zα, rα) <strong>and</strong> all n = 1, 2, . . . , as desired. ⊓⊔<br />

8. State <strong>and</strong> prove:<br />

(a) the mean value property for analytic functions<br />

(b) the maximum pr<strong>in</strong>ciple for analytic functions.<br />

39 Answer: If K ⊂ D is compact, we could select a f<strong>in</strong>ite cover<strong>in</strong>g <strong>of</strong> K by such neighborhoods B(zαj , rα j ) (j = 1, . . . , J) <strong>and</strong> then<br />

|fn(z)| ≤ maxj Mα j MK, for all z ∈ K <strong>and</strong> n = 1, 2, . . . .<br />

46


2.2 1991 November 21 2 COMPLEX ANALYSIS<br />

SECTION C<br />

(Do 2 <strong>of</strong> the 3 problems.)<br />

9. Let X be a Hausdorff topological space, let K be a compact subset <strong>of</strong> X, <strong>and</strong> let x be a po<strong>in</strong>t <strong>of</strong> X not <strong>in</strong> K. Show<br />

that there exist open sets U <strong>and</strong> V such that<br />

K ⊂ U, x ∈ V, U ∩ V = ∅.<br />

10. A topological space X satisfies the second axiom <strong>of</strong> countability. Prove that every open cover <strong>of</strong> X has a countable<br />

subcover.<br />

11. Let X be a topological space, <strong>and</strong> let U be a subset <strong>of</strong> X.<br />

(a) Show that if an open set <strong>in</strong>tersects the closure <strong>of</strong> Y then it <strong>in</strong>tersects Y .<br />

(b) Show that if Y is connected <strong>and</strong> if Y ⊂ Z ⊂ ¯ Y , then Z is connected.<br />

47


2.3 1995 April 10 2 COMPLEX ANALYSIS<br />

2.3 1995 April 10<br />

Instructions. Work as many <strong>of</strong> the problems as you can. Each solution should be clearly written on a separate sheet<br />

<strong>of</strong> paper.<br />

1. Let f(z) = anz n be an entire function.<br />

(a) Suppose that |f(z)| ≤ A|z| N + B for all z ∈ C where A, B are f<strong>in</strong>ite constants. Show that f is a polynomial<br />

<strong>of</strong> degree N or less.<br />

(b) Suppose that f satisfies the condition: |f(zn)| → ∞ whenever |zn| → ∞. Show that f is a polynomial.<br />

Solution: (a) By Cauchy’s formula, we have<br />

for every R > 0. Therefore,<br />

|an| ≤ 1<br />

2π<br />

<br />

|ζ|=R<br />

an = f (n) (0)<br />

n!<br />

<br />

1 f(ζ)<br />

= dζ,<br />

2πi |ζ|=R ζn+1 |f(ζ)| 1 A R<br />

|dζ| ≤<br />

|ζ| n+1 2π<br />

N + B<br />

Rn+1 2πR = A R N−n + B R −n .<br />

Aga<strong>in</strong>, this holds for every R > 0. Thus, for any n > N <strong>and</strong> ɛ > 0, tak<strong>in</strong>g R large enough forces |an| < ɛ<br />

(n = N + 1, N + 2, . . . ). S<strong>in</strong>ce ɛ was arbitrary, we have an = 0 for all n = N + 1, N + 2, . . . . Therefore,<br />

f(z) = N<br />

n=0 anz n . ⊓⊔<br />

(b) We give three different pro<strong>of</strong>s. The first is the shortest, but relies on the heaviest mach<strong>in</strong>ery.<br />

Pro<strong>of</strong> 1: If we take for granted that any transcendental (i.e. non-polynomial) entire function has an essential s<strong>in</strong>gularity<br />

at <strong>in</strong>f<strong>in</strong>ity, then the Casorati-Weierstrass theorem (see 3 <strong>of</strong> Nov. ’01) implies that, for any complex number<br />

w, there is a sequence {zn} with zn → ∞ <strong>and</strong> f(zn) → w as n → ∞. S<strong>in</strong>ce this contradicts the given hypotheses,<br />

f(z) cannot be a transcendental function. That is, f(z) must be a polynomial. ⊓⊔<br />

Pro<strong>of</strong> 2: S<strong>in</strong>ce f ∈ H(C), the series f(z) = anz n converges locally uniformly <strong>in</strong> C. The hypotheses imply that<br />

the function f(1/z) has a pole at z = 0. Let<br />

g(z) = f(1/z) =<br />

∞<br />

n=−∞<br />

be the Laurent series expansion <strong>of</strong> the function g about z = 0. Suppose the pole at z = 0 is <strong>of</strong> order m. Clearly m<br />

is f<strong>in</strong>ite, by the criterion for a pole (i.e., limz→0 f(1/z) = ∞). Therefore, we can write<br />

g(z) = f(1/z) =<br />

∞<br />

n=−m<br />

bnz n<br />

bnz n = b−mz −m + b−m+1z −m+1 + · · · b−1z −1 + b0 + b1z + · · · (36)<br />

Now f is entire, so it has the form f(z) = ∞<br />

n=0 anz n , which implies that f(1/z) = a0 + a1z −1 + a2z −2 + · · · .<br />

Compared with (36),<br />

a0 + a1z −1 + a2z −2 + · · · = f(1/z) = b−mz −m + b−m+1z −m+1 + · · · b−1z −1 + b0 + b1z + · · ·<br />

That is, 0 = am+1 = am+2 = · · · , so<br />

f(z) =<br />

48<br />

m<br />

anz n .<br />

n=0


2.3 1995 April 10 2 COMPLEX ANALYSIS<br />

Pro<strong>of</strong> 3: By the hypotheses, there is an R > 0 such that |f(z)| > 0 for all |z| > R. Therefore, the zeros <strong>of</strong> f are<br />

conf<strong>in</strong>ed to a closed disk DR = {|z| ≤ R}. S<strong>in</strong>ce the zeros <strong>of</strong> f are isolated, there are at most f<strong>in</strong>itely many <strong>of</strong><br />

them <strong>in</strong> any compact subset <strong>of</strong> C. In particular, DR conta<strong>in</strong>s only f<strong>in</strong>itely many zeros <strong>of</strong> f. This proves that f has<br />

only f<strong>in</strong>itely many zeros <strong>in</strong> C.<br />

Let {α1, . . . , αN} be the collections <strong>of</strong> all zeros <strong>of</strong> f (count<strong>in</strong>g multiplicities). Consider the function<br />

g(z) =<br />

f(z)<br />

. (37)<br />

(z − α1) · · · (z − αN)<br />

This is def<strong>in</strong>ed <strong>and</strong> holomorphic <strong>in</strong> C \ {α1, . . . , αN }, but the αi’s are removable s<strong>in</strong>gularities, so g(z) is a nonzero<br />

entire function. In particular, for any R > 0,<br />

m<strong>in</strong> |g(z)| ≥ m<strong>in</strong> |g(z)| = ɛ > 0,<br />

z∈DR<br />

|z|=R<br />

for some ɛ > 0. Therefore, 1/g is a bounded entire function, hence constant, by Liouville’s theorem. What we<br />

have shown is that the left h<strong>and</strong> side <strong>of</strong> (37) is constant, <strong>and</strong> this proves that f(z) is a polynomial. ⊓⊔<br />

Remark: A nice corollary to part (b) is the follow<strong>in</strong>g:<br />

Corollary 2.1 If f is an <strong>in</strong>jective entire function, then f(z) = az + b for some constants a <strong>and</strong> b.<br />

The pro<strong>of</strong> appears below <strong>in</strong> section 2.9.<br />

2. (a) State a form <strong>of</strong> the Cauchy theorem.<br />

(b) State a converse <strong>of</strong> the Cauchy theorem.<br />

Solution: (a) See theorem A.16.<br />

(b) See theorem A.18.<br />

3. Let 40 f(z) = ∞<br />

n=0 anz n be analytic <strong>and</strong> one-to-one on |z| < 1. Suppose that |f(z)| < 1 for all |z| < 1.<br />

(a) Prove that<br />

(b) Is the constant 1 the best possible?<br />

∞<br />

n|an| 2 ≤ 1.<br />

n=1<br />

Solution: (a) This is a special case <strong>of</strong> the follow<strong>in</strong>g area theorem:<br />

Theorem 2.1 Suppose f(z) = ∞<br />

n=0 anz n is a holomorphic function which maps the unit disk D = {|z| < 1}<br />

bijectively onto a doma<strong>in</strong> f(D) = G hav<strong>in</strong>g area A. Then<br />

A = π<br />

∞<br />

n|an| 2 .<br />

n=1<br />

40 On the orig<strong>in</strong>al exam, the power series representation was given as f(z) = ∞<br />

n=1 anzn . However, the problem can be solved without<br />

assum<strong>in</strong>g a0 = 0 a priori.<br />

49


2.3 1995 April 10 2 COMPLEX ANALYSIS<br />

Pro<strong>of</strong>: The area <strong>of</strong> the image <strong>of</strong> D under f is the <strong>in</strong>tegral over D <strong>of</strong> the Jacobian <strong>of</strong> f. That is,<br />

<br />

A = |f ′ (z)| 2 dx dy.<br />

Compute |f ′ (z)| by differentiat<strong>in</strong>g the power series <strong>of</strong> f(z) term by term,<br />

f ′ ∞<br />

(z) = nanz n−1 .<br />

Next, take the squared modulus,<br />

This gives,<br />

Lett<strong>in</strong>g z = re iθ ,<br />

A =<br />

|f ′ (z)| 2 =<br />

<br />

A =<br />

∞<br />

m,n=1<br />

∞<br />

D<br />

m,n=1<br />

∞<br />

D m,n=1<br />

m n aman<br />

n=1<br />

m n amanz m−1 z n−1 .<br />

m n amanz m−1 z n−1 dx dy.<br />

1 2π<br />

0<br />

0<br />

r m+n−1 e i(m−n)θ dθ dr.<br />

Now, for all k = 0, the <strong>in</strong>tegral <strong>of</strong> e ikθ over 0 ≤ θ < 2π vanishes, so the only non-vanish<strong>in</strong>g terms <strong>of</strong> the series are those for<br />

which m = n. That is,<br />

∞<br />

A = 2π n 2 |an| 2<br />

1<br />

r 2n−1 ∞<br />

dr = π n 2 |an| 2 . (38)<br />

n=1<br />

0<br />

To apply this theorem to the problem at h<strong>and</strong>, note that the hypotheses <strong>of</strong> the problem imply that f maps the unit<br />

disk bijectively onto its range f(D), which is conta<strong>in</strong>ed <strong>in</strong>side D <strong>and</strong>, therefore, has area less or equal to π. This<br />

<strong>and</strong> (38) together imply<br />

∞<br />

π ≥ π n 2 |an| 2 ,<br />

n=1<br />

which gives the desired <strong>in</strong>equality. ⊓⊔<br />

(b) The identity function f(z) = z satisfies the given hypotheses <strong>and</strong> its power series expansion has coefficients<br />

a1 = 1 <strong>and</strong> 0 = a0 = a2 = a3 = · · · . This shows that the upper bound <strong>of</strong> 1 is obta<strong>in</strong>ed <strong>and</strong> is therefore the best<br />

possible. ⊓⊔<br />

4. Let u(z) be a nonconstant, real valued, harmonic function on C. Prove there exists a sequence {zn} with |zn| → ∞<br />

for which u(zn) → 0.<br />

Solution: Suppose, by way <strong>of</strong> contradiction, that there is no such sequence. Then u(z) is bounded away from zero<br />

for all z <strong>in</strong> some neighborhood <strong>of</strong> <strong>in</strong>f<strong>in</strong>ity, say, {|z| > R}, for some R > 0. S<strong>in</strong>ce u is cont<strong>in</strong>uous, either u(z) > 0<br />

for all |z| > R, or u(z) < 0 for all |z| > R. Assume without loss <strong>of</strong> generality that u(z) > 0 for all |z| > R.<br />

S<strong>in</strong>ce u is cont<strong>in</strong>uous on the compact set {|z| ≤ R}, it atta<strong>in</strong>s its m<strong>in</strong>imum on that set. Thus, there is an M > 0<br />

such that −M ≤ u(z) for all |z| ≤ R. Consider the function U(z) = u(z) + M. By construction, U(z) ≥ 0 for<br />

all z ∈ C, <strong>and</strong> U is harmonic <strong>in</strong> C. But this implies U(z), hence u(z), must be constant. 41 This contradicts the<br />

hypothesis that u(z) be nonconstant <strong>and</strong> completes the pro<strong>of</strong>. ⊓⊔<br />

41 Recall problem 1(b), April ’89, where we proved that a real valued harmonic function u(z) satisfy<strong>in</strong>g u(z) ≥ 0 for all z ∈ C must be constant.<br />

50<br />

n=1<br />

⊓⊔


2.3 1995 April 10 2 COMPLEX ANALYSIS<br />

5. 42 F<strong>in</strong>d an explicit conformal mapp<strong>in</strong>g <strong>of</strong> the semidisk<br />

onto the unit disk.<br />

H = {z : |z| < 1, Rez > 0}<br />

Solution: See the solution to (3) <strong>of</strong> April ’89, or (2) <strong>of</strong> November ’06.<br />

6. 43 Suppose f(z) is a holomorphic function on the unit disk which satisfies:<br />

(a) State the Schwarz lemma, as applied to f.<br />

(b) If f(0) = 1<br />

2 , how large can |f ′ (0)| be?<br />

Solution: (a) See theorem A.22.<br />

|f(z)| < 1 all |z| < 1.<br />

(b) Assume f satisfies the given hypotheses. In particular, f(0) = 1<br />

2 . Consider the map<br />

ϕ(z) =<br />

1<br />

2<br />

− z<br />

1 − z .<br />

2<br />

This is a holomorphic bijection <strong>of</strong> the unit disk, with φ(1/2) = 0. Therefore, g = ϕ ◦ f satisfies the hypotheses <strong>of</strong><br />

Schwarz’s lemma. In particular, |g ′ (0)| ≤ 1. S<strong>in</strong>ce g ′ (z) = ϕ ′ (f(z))f ′ (z), we have<br />

Now,<br />

1 ≥ |g ′ (0)| = |ϕ ′ (1/2)||f ′ (0)|. (39)<br />

ϕ ′ (z) = − 1 − z<br />

<br />

1 1<br />

2 + 2 2 − z<br />

<br />

z 2 .<br />

1 − 2<br />

Therefore, ϕ ′ (1/2) = −4/3, <strong>and</strong> it follows from (39) that<br />

|f ′ (0)| ≤<br />

42 This problem also appears <strong>in</strong> April ’89 (3) <strong>and</strong> November ’06 (2).<br />

43 A very similar problem appeared <strong>in</strong> November ’06 (3).<br />

1<br />

|ϕ ′ = 3/4.<br />

(1/2)|<br />

51<br />

⊓⊔


2.4 2001 November 26 2 COMPLEX ANALYSIS<br />

2.4 2001 November 26<br />

Instructions. Make a substantial effort on all parts <strong>of</strong> the follow<strong>in</strong>g problems. If you cannot completely answer Part<br />

(a) <strong>of</strong> a problem, it is still possible to do Part (b). Partial credit is given for partial progress. Include as many details as<br />

time permits. Throughout the exam, z denotes a complex variable, <strong>and</strong> C denotes the complex plane.<br />

1. (a) Suppose that f(z) = f(x+iy) = u(x, y)+iv(x, y) where u <strong>and</strong> v are C1 functions def<strong>in</strong>ed on a neighborhood<br />

<strong>of</strong> the closure <strong>of</strong> a bounded region G ⊂ C with boundary which is parametrized by a properly oriented, piecewise<br />

C1 curve γ. If u <strong>and</strong> v obey the Cauchy-Riemann equations, show that Cauchy’s theorem <br />

f(z) dz = 0 follows<br />

γ<br />

from Green’s theorem, namely<br />

<br />

<br />

∂Q ∂P<br />

P dx + Q dy = − dx dy for C<br />

∂x ∂y<br />

1 functions P <strong>and</strong> Q. (40)<br />

γ<br />

G<br />

(b) Suppose that we do not assume that u <strong>and</strong> v are C 1 , but merely that u <strong>and</strong> v are cont<strong>in</strong>uous <strong>in</strong> G <strong>and</strong><br />

f ′ f(z) − f(z0)<br />

(z0) = lim<br />

z→z0 z − z0<br />

exists at some (possibly only one!) po<strong>in</strong>t z0 ∈ G. Show that given any ɛ > 0, we can f<strong>in</strong>d a triangular region ∆<br />

conta<strong>in</strong><strong>in</strong>g z0, such that if T is the boundary curve <strong>of</strong> ∆, then<br />

<br />

<br />

<br />

<br />

f(z) dz<br />

1<br />

=<br />

2 ɛL2 ,<br />

T<br />

where L is the length <strong>of</strong> the perimeter <strong>of</strong> ∆.<br />

H<strong>in</strong>t for (b) Note that part (a) yields <br />

(az + b) dz = 0 for a, b ∈ C, which you can use here <strong>in</strong> (b), even if you<br />

T<br />

could not do Part (a). You may also use the fact that <br />

T g(z) dz ≤ L · sup{|g(z)| : z ∈ T } for g cont<strong>in</strong>uous on T .<br />

Solution: (a) Let P = u <strong>and</strong> Q = −v <strong>in</strong> (40). Then, by the Cauchy-Riemann equations, 44<br />

<br />

<br />

u(x, y) dx − v(x, y) dy = (vx + uy) dx dy = 0. (41)<br />

γ<br />

G<br />

Similarly, if P = v <strong>and</strong> Q = u <strong>in</strong> Green’s theorem, then the Cauchy-Riemann equations imply<br />

<br />

<br />

v(x, y) dx + u(x, y) dy = (ux − vy) dx dy = 0. (42)<br />

γ<br />

G<br />

Next, note that<br />

f(z) dz = [u(x, y) + iv(x, y)] d(x + iy) = u(x, y) dx − v(x, y) dy + i[v(x, y) dx + u(x, y) dy].<br />

Therefore, by (41) <strong>and</strong> (42),<br />

<br />

<br />

f(z) dz =<br />

44 These are ux = vy <strong>and</strong> uy = −vx.<br />

γ<br />

γ<br />

<br />

u(x, y) dx − v(x, y) dy + i v(x, y) dx + u(x, y) dy = 0.<br />

γ<br />

52<br />

⊓⊔


2.4 2001 November 26 2 COMPLEX ANALYSIS<br />

(b) Suppose u <strong>and</strong> v are cont<strong>in</strong>uous <strong>and</strong> f ′ (z) exists at the po<strong>in</strong>t z0 ∈ G. Then, for any ɛ > 0 there is a δ > 0 such<br />

that B(z0, δ) ⊆ G, <strong>and</strong> <br />

<br />

f ′ <br />

f(z) − f(z0) <br />

(z0) − <br />

z − z0<br />

< ɛ, for all |z − z0| < δ.<br />

Pick a triangular region ∆ ⊂ B(z0, δ) with z0 ∈ ∆, <strong>and</strong> let T be the boundary. Def<strong>in</strong>e<br />

R(z) = f(z) − [f(z0) + f ′ (z0)(z − z0)].<br />

Then, by Cauchy’s theorem (part (a)), <br />

T [f(z0) + f ′ (z0)(z − z0)] dz = 0, whence <br />

<br />

R(z) dz = T<br />

F<strong>in</strong>ally, note that <br />

R(z) <br />

<br />

z − z0<br />

=<br />

<br />

<br />

<br />

f ′ <br />

f(z) − f(z0) <br />

(z0) − <br />

z − z0<br />

< ɛ, for all |z − z0| < δ.<br />

Therefore,<br />

<br />

<br />

<br />

<br />

f(z) dz<br />

<br />

T<br />

=<br />

<br />

<br />

<br />

<br />

R(z) dz<br />

<br />

T<br />

≤<br />

<br />

|R(z)| |dz|<br />

T<br />

<br />

<br />

= <br />

R(z) <br />

<br />

<br />

T z − z0<br />

|z − z0| |dz|<br />

<br />

≤ ɛ |z − z0| |dz| ≤ ɛrL.<br />

T<br />

T<br />

f(z) dz.<br />

where L denotes the length <strong>of</strong> the perimeter <strong>of</strong> ∆ (i.e., the length <strong>of</strong> T ), <strong>and</strong> r denotes the length <strong>of</strong> one side <strong>of</strong> T ,<br />

which must, <strong>of</strong> course, be greater than |z − z0| for all z ∈ T . Also, the length <strong>of</strong> one side <strong>of</strong> ∆ is surely less than<br />

half the length <strong>of</strong> the perimeter (i.e., r < L/2). Therefore,<br />

<br />

<br />

<br />

<br />

T<br />

<br />

<br />

f(z) dz<br />

<br />

≤ 1<br />

2 ɛL2 .<br />

2. Give two quite different pro<strong>of</strong>s <strong>of</strong> the fundamental theorem <strong>of</strong> algebra, that if a polynomial with complex coefficients<br />

has no complex zero, then it is constant. You may use <strong>in</strong>dependent, well-known theorems <strong>and</strong> pr<strong>in</strong>ciples<br />

such as Liouville’s theorem, the argument pr<strong>in</strong>ciple, the maximum pr<strong>in</strong>ciple, Rouché’s theorem, <strong>and</strong>/or the open<br />

mapp<strong>in</strong>g theorem.<br />

Solution: In the two pro<strong>of</strong>s below, we beg<strong>in</strong> by suppos<strong>in</strong>g p(z) is not constant <strong>and</strong> thus has the form p(z) =<br />

a0 + a1z + a2z 2 + · · · + anz n with an = 0 for some n ≥ 1. Both pro<strong>of</strong>s also rely on the follow<strong>in</strong>g observation:<br />

If {aj} n j=0 ⊂ C with an = 0, then for all 1 ≤ R ≤ |z| < ∞,<br />

<br />

<br />

<br />

<br />

a0<br />

an<br />

z −n + · · · + an−1<br />

z −1<br />

<br />

<br />

<br />

|a0|<br />

≤<br />

|an| |z|−n + · · · + |an−1|<br />

|z|<br />

|an|<br />

−1<br />

an<br />

|aj|<br />

≤ n max<br />

0≤j


2.4 2001 November 26 2 COMPLEX ANALYSIS<br />

In particular, if we choose45 R = 1 + 2 n max0≤j


2.4 2001 November 26 2 COMPLEX ANALYSIS<br />

3. (a) State <strong>and</strong> prove the Casorati-Weierstrass theorem concern<strong>in</strong>g the image <strong>of</strong> any punctured disk about a certa<strong>in</strong><br />

type <strong>of</strong> isolated s<strong>in</strong>gularity <strong>of</strong> an analytic function. You may use the fact that if a function g is analytic <strong>and</strong> bounded<br />

<strong>in</strong> the neighborhood <strong>of</strong> a po<strong>in</strong>t z0, then g has a removable s<strong>in</strong>gularity at z0.<br />

(b) Verify the Casorati-Weierstrass theorem directly for a specific analytic function <strong>of</strong> your choice, with a suitable<br />

s<strong>in</strong>gularity.<br />

Solution:<br />

Theorem 2.2 (Casorati-Weierstrass) If f is a holomorphic function <strong>in</strong> a region G ∈ C except for an essential<br />

s<strong>in</strong>gularity at the po<strong>in</strong>t z = z0, then for any w ∈ C there is a sequence {zn} ⊂ G approach<strong>in</strong>g z0 such that<br />

f(zn) → w as n → ∞.<br />

Pro<strong>of</strong>: Fix w0 ∈ C <strong>and</strong> suppose there is no sequence {zn} ⊂ G approach<strong>in</strong>g z0 such that f(zn) → w0 as n → ∞.<br />

Then there is a punctured disk ¯ D0 B(z0, ɛ) \ {z0} ⊂ G such that |f(z) − w0| > δ > 0 for all z ∈ ¯ D0. Def<strong>in</strong>e<br />

g(z) = 1/(f(z) − w0) on D0. Then<br />

lim sup<br />

z→z0<br />

z∈D0<br />

|g(z)| = lim sup<br />

z→z0<br />

z∈D0<br />

1 1<br />

≤ < ∞.<br />

|f(z) − w0| δ<br />

Thus, by lemma 2.1 (Nov. ’06, prob. 1), z0 is a removable s<strong>in</strong>gularity <strong>of</strong> g(z). Therefore, g(z) ∈ H(B(z0, ɛ)). In<br />

particular, g is cont<strong>in</strong>uous <strong>and</strong> non-zero at z = z0, so it is non-zero <strong>in</strong> a neighborhood B(z0, ɛ0) <strong>of</strong> z0. Therefore,<br />

f(z)−w0 = 1/g(z) is holomorphic <strong>in</strong> B(z0, ɛ0), which implies that the s<strong>in</strong>gularity <strong>of</strong> f(z) at z = z0 is removable.<br />

This contradiction proves the theorem. ⊓⊔<br />

(b) Consider f(z) = e z . This function has an essential s<strong>in</strong>gularity at ∞, <strong>and</strong>, for every horizontal strip,<br />

Sα = {x + iy : x ∈ R, α ≤ y < α + 2π},<br />

<strong>of</strong> width 2π, f(z) maps Sα onto C \ {0}. (In particular, f(z) comes arbitrarily close to every w ∈ C.) Now let<br />

NR = {z ∈ C : |z| > R} be any neighborhood <strong>of</strong> ∞. There is clearly a strip Sα conta<strong>in</strong>ed <strong>in</strong> NR (e.g., with<br />

α = R + 1). Therefore, f(z) = e z maps po<strong>in</strong>ts <strong>in</strong> NR to po<strong>in</strong>ts arbitrarily close (<strong>in</strong> fact equal when w = 0) to all<br />

po<strong>in</strong>ts w ∈ C. ⊓⊔<br />

4. (a) Def<strong>in</strong>e γ : [0, 2π] → C by γ(t) = s<strong>in</strong>(2t) + 2i s<strong>in</strong>(t). This is a parametrization <strong>of</strong> a “figure 8” curve, traced<br />

out <strong>in</strong> a regular fashion. F<strong>in</strong>d a meromorphic function f such that <br />

f(z) dz = 1. Be careful with m<strong>in</strong>us signs <strong>and</strong><br />

γ<br />

factors <strong>of</strong> 2πi.<br />

(b) From the theory <strong>of</strong> Laurent expansions, it is known that there are constants an such that, for 1 < |z| < 4,<br />

F<strong>in</strong>d a−10 <strong>and</strong> a10 by the method <strong>of</strong> your choice.<br />

1<br />

z 2 − 5z + 4 =<br />

∞<br />

n=−∞<br />

anz n .<br />

Solution: (a) Let G be the region whose boundary is the curve γ, <strong>and</strong> suppose f(z) ∈ H(C) except for isolated<br />

s<strong>in</strong>gularities at the po<strong>in</strong>ts {z1, . . . , zn} ⊂ G. By the residue theorem,<br />

<br />

n<br />

f(z) dz = 2πi Res(f, zj).<br />

γ<br />

55<br />

j=1


2.4 2001 November 26 2 COMPLEX ANALYSIS<br />

Therefore, if we were to f<strong>in</strong>d a function f(z) ∈ H(C) with exactly two isolated s<strong>in</strong>gularities <strong>in</strong> G (e.g., at z1 = i<br />

<strong>and</strong> z2 = −i), <strong>and</strong> such that Res(f, zj) = −i<br />

4π , then<br />

<br />

f(z) dz = 2πi <br />

<br />

−i −i<br />

Res(f, zj) = 2πi + = 1,<br />

4π 4π<br />

<strong>and</strong> the problem would be solved. Clearly,<br />

γ<br />

f(z) = −i<br />

4π<br />

j<br />

<br />

1 1<br />

− =<br />

z − i z + i<br />

1<br />

2πi<br />

z<br />

z2 + 1<br />

is such a function. ⊓⊔<br />

(b) Exp<strong>and</strong> the function <strong>in</strong> partial fractions:<br />

Then, note that<br />

converges for |z| < 4, while<br />

converges for |z| > 1. Therefore,<br />

1<br />

z2 − 5z + 4 =<br />

1 1/3 1/3<br />

= −<br />

(z − 4)(z − 1) z − 4 z − 1 .<br />

1/3<br />

z − 4<br />

1/3<br />

z − 1<br />

1<br />

z2 = −1<br />

− 5z + 4 3<br />

1 −1<br />

=<br />

3 4(1 − z/4)<br />

1<br />

= −1<br />

3 z(1 − 1/z)<br />

−1<br />

n=−∞<br />

∴ a−10 = − 1<br />

3<br />

z n − 1<br />

12<br />

= − 1<br />

12<br />

∞<br />

n=0<br />

= − 1<br />

3z<br />

∞<br />

n=0<br />

<br />

z<br />

n 4<br />

∞<br />

z −n<br />

n=0<br />

n 1<br />

z<br />

4<br />

n , for 1 < |z| < 4.<br />

<strong>and</strong> a10 = − 1<br />

12<br />

10 1<br />

.<br />

4<br />

5. (a) Suppose that f is analytic on a region G ⊂ C <strong>and</strong> {z ∈ C : |z − a| ≤ R} ⊂ G. Show that if |f(z)| ≤ M for<br />

all z with |z − a| = R, then for any w1, w2 ∈ {z ∈ C : |z − a| ≤ 1<br />

2R}, we have<br />

|f(w1) − f(w2)| ≤ 4M<br />

R |w1 − w2|<br />

(b) Expla<strong>in</strong> how Part (a) can be used with the Arzela-Ascoli theorem to prove Montel’s theorem assert<strong>in</strong>g the<br />

normality <strong>of</strong> any locally bounded family F <strong>of</strong> analytic functions on a region G.<br />

Solution: (a) By Cauchy’s formula (A.14), if w is any po<strong>in</strong>t <strong>in</strong> the disk |w − a| < R, then<br />

f(w) = 1<br />

<br />

f(ζ)<br />

2πi ζ − w dζ.<br />

|ζ−a|=R<br />

56<br />

⊓⊔


2.4 2001 November 26 2 COMPLEX ANALYSIS<br />

In particular, if w1, w2 are any two po<strong>in</strong>ts <strong>in</strong>side the “half-disk” |w − a| < R/2 (see figure 1), then<br />

f(w1) − f(w2) = 1<br />

<br />

f(ζ)<br />

−<br />

2πi |ζ−a|=R ζ − w1<br />

f(ζ)<br />

<br />

dζ<br />

ζ − w2<br />

= w1<br />

<br />

− w2<br />

f(ζ)<br />

2πi<br />

(ζ − w1)(ζ − w2) dζ.<br />

W 2<br />

W 1<br />

.<br />

.<br />

a<br />

|ζ−a|=R<br />

. R/2<br />

Figure 1: Note that, if ζ is any po<strong>in</strong>t on the outer radius, |ζ − a| = R, <strong>and</strong> if w is any po<strong>in</strong>t <strong>in</strong> the disk |w − a| < R/2,<br />

then |ζ − w| > R/2.<br />

Now, for all ζ on the outer radius <strong>in</strong> figure 1, it is clear that |ζ − w1| > R/2 <strong>and</strong> |ζ − w2| > R/2. Therefore,<br />

|f(w1) − f(w2)| ≤ |w1<br />

<br />

− w2| |f(ζ)|<br />

|dζ|<br />

2π<br />

(R/2) 2<br />

.<br />

w<br />

≤ |w1 − w2|<br />

2π<br />

≤ 4M<br />

R |w1 − w2|,<br />

|ζ−a|=R<br />

R<br />

. ζ<br />

supγ |f(ζ)|<br />

R2 ℓ(γ)<br />

/4<br />

where γ denotes the positively oriented circle {ζ : |ζ − a| = R}, <strong>and</strong> ℓ(γ) denotes its length, 2πR. ⊓⊔<br />

(b) 47 We must expla<strong>in</strong> how part (a) can be used with the Arzela-Ascoli theorem to prove Montel’s theorem assert<strong>in</strong>g<br />

the normality <strong>of</strong> any locally bounded family F ⊂ H(G).<br />

Theorem 2.3 (Arzela-Ascoli) Let F ⊂ C(G, S) be a family <strong>of</strong> cont<strong>in</strong>uous functions from an open set G ⊆ C <strong>in</strong>to<br />

a metric space (S, d). Then F is a normal family if <strong>and</strong> only if<br />

(i) F is equicont<strong>in</strong>uous on each compact subset <strong>of</strong> G, <strong>and</strong><br />

(ii) for each z ∈ G, the set {f(z) : f ∈ F} is conta<strong>in</strong>ed <strong>in</strong> a compact subset <strong>of</strong> S.<br />

47 The best treatment <strong>of</strong> normal families <strong>and</strong> the Arzela-Ascoli theorem is Ahlfors [1].<br />

57


2.4 2001 November 26 2 COMPLEX ANALYSIS<br />

Recall that a family F <strong>of</strong> functions is called locally bounded on G iff for all compact K ⊂ G there is a constant<br />

MK such that |f(z)| ≤ MK for all f ∈ F <strong>and</strong> z ∈ K.<br />

Corollary 2.2 (little Montel theorem) Assume the set-up <strong>of</strong> the Arzela-Ascoli theorem, <strong>and</strong> suppose S = C <strong>and</strong><br />

F ⊂ H(G). Then F is a normal family if <strong>and</strong> only if it is locally bounded.<br />

Because <strong>of</strong> the way the problem is stated, it is probably enough to prove just one direction <strong>of</strong> Montel’s theorem;<br />

i.e., local boundedness implies normality. For a pro<strong>of</strong> <strong>of</strong> the other direction, see Conway [3], page 153.<br />

Let S = C <strong>in</strong> the Arzela-Ascoli theorem. In that case, K ⊂ C is compact if <strong>and</strong> only if K is closed <strong>and</strong> bounded.<br />

Therefore, if F is locally bounded, condition (ii) <strong>of</strong> the theorem is clearly satisfied. To check that local boundedness<br />

also implies condition (i), we use part (a).<br />

It suffices (why?) 48 to prove that for any a ∈ G there is a neighborhood B(a, r) <strong>in</strong> which F is equicont<strong>in</strong>uous with<br />

equicont<strong>in</strong>uity constant 49 δ. So, fix a ∈ G <strong>and</strong> ɛ > 0, <strong>and</strong> let ¯ B(a, R) ⊂ G. Then, by local boundedness, there is<br />

an M > 0 such that |f(z)| ≤ M for all z ∈ ¯ B(a, R) <strong>and</strong> all f ∈ F. Therefore, by part (a),<br />

|f(w1) − f(w2)| ≤ 4M<br />

R |w1 − w2|, for all w1, w2 ∈ {|w − a| ≤ R/2}.<br />

If δ = R<br />

4M ɛ <strong>and</strong> r = R/2, then |f(w1) − f(w2)| < ɛ whenever w1, w2 ∈ B(a, r) <strong>and</strong> |w1 − w2| < δ. Therefore,<br />

F is equicont<strong>in</strong>uous <strong>in</strong> B(a, r). We have thus shown that local boundedness implies conditions (i) <strong>and</strong> (ii) <strong>of</strong> the<br />

Arzela-Ascoli theorem <strong>and</strong> thereby implies normality. ⊓⊔<br />

48 Answer: If, <strong>in</strong>stead <strong>of</strong> a s<strong>in</strong>gle po<strong>in</strong>t a ∈ G, we are given a compact set K ⊂ G, then there is a f<strong>in</strong>ite cover {B(aj, rj) : j = 1, . . . , n} by<br />

such neighborhoods with equicont<strong>in</strong>uity constants δ1, . . . , δn. Then, δ = m<strong>in</strong>j δj, is a s<strong>in</strong>gle equicont<strong>in</strong>uity constant that works for all <strong>of</strong> K.<br />

49 The careful reader might note the dist<strong>in</strong>ction between this type <strong>of</strong> “uniform” equicont<strong>in</strong>uity, which is taken for granted <strong>in</strong> complex analysis<br />

texts, e.g., Ahlfors [1] <strong>and</strong> Rud<strong>in</strong> [8], <strong>and</strong> the “po<strong>in</strong>twise” equicont<strong>in</strong>uity discussed <strong>in</strong> topology books like the one by Munkres [5]. To make peace<br />

with this apparent discrepancy, check that the two notions co<strong>in</strong>cide when the set on which a family <strong>of</strong> functions is declared equicont<strong>in</strong>uous is<br />

compact.<br />

58


2.5 2004 April 19 2 COMPLEX ANALYSIS<br />

2.5 2004 April 19<br />

Instructions. Use a separate sheet <strong>of</strong> paper for each new problem. Do as many problems as you can. Complete<br />

solutions to five problems will be considered as an excellent performance. Be advised that a few complete <strong>and</strong> well<br />

written solutions will count more than several partial solutions.<br />

Notation. D(z0, R) = {z ∈ C : |z − z0| < R} R > 0. For an open set G ⊆ C, H(G) will denote the set <strong>of</strong><br />

functions which are analytic <strong>in</strong> G.<br />

1. Let γ be a rectifiable curve <strong>and</strong> let ϕ ∈ C(γ ∗ ). (That is, ϕ is a cont<strong>in</strong>uous complex function def<strong>in</strong>ed on the trace,<br />

γ ∗ , <strong>of</strong> γ.)<br />

Let F (z) = <br />

Prove that F ′ (z) = <br />

γ<br />

ϕ(w)<br />

(w−z) dw, z ∈ C \ γ∗ .<br />

2. (a) State the Casorati-Weierstrass theorem.<br />

(b) Evaluate the <strong>in</strong>tegral<br />

γ<br />

ϕ(w)<br />

(w−z) 2 dw, z ∈ C \ γ ∗ , without us<strong>in</strong>g Leibniz’s Rule.<br />

I = 1<br />

<br />

<br />

1<br />

(z − 3) s<strong>in</strong> dz where R ≥ 4.<br />

2πi |z|=R<br />

z + 2<br />

3. Let f(z) be an entire function such that f(0) = 1, f ′ (0) = 0 <strong>and</strong><br />

Prove that f(z) = 1 for all z ∈ C.<br />

0 < |f(z)| ≤ e |z|<br />

for all z ∈ C<br />

Solution: I know <strong>of</strong> two ways to prove this. One can be found <strong>in</strong> Rud<strong>in</strong>’s Functional Analysis ([9], p. 250). The<br />

other goes as follows: 50<br />

By the Hadamard factorization theorem (see, e.g., [11]), an entire function f with zeros at {an} ⊂ C \ {0} <strong>and</strong> m<br />

zeros at z = 0 has the form<br />

f(z) = e P (z) z m<br />

∞<br />

<br />

1 − z<br />

<br />

e z/an , (46)<br />

n=0<br />

where P (z) is a polynomial <strong>of</strong> degree ρ, the “order <strong>of</strong> growth,” <strong>and</strong> k ≤ ρ < k + 1. For the function <strong>in</strong> question,<br />

we have |f(z)| > 0 so {an} = ∅ <strong>and</strong> m = 0. Also, s<strong>in</strong>ce |f(z)| ≤ e |z| , the order <strong>of</strong> growth is ρ = 1, which implies<br />

that P (z) is a polynomial <strong>of</strong> degree 1. Therefore, (46) takes the simple form,<br />

f(z) = e Bz+C ,<br />

for some constants B, C. We are given that f(0) = 1 <strong>and</strong> f ′ (0) = 0, so e C = 1, <strong>and</strong> f ′ (0) = Be C = B = 0. It<br />

follows that f(z) = 1. ⊓⊔<br />

50 This pro<strong>of</strong> came to me by sheer lucky co<strong>in</strong>cidence – I worked on this exam after hav<strong>in</strong>g just read a beautiful treatment <strong>of</strong> the Hadamard<br />

factorization theorem <strong>in</strong> Ste<strong>in</strong> <strong>and</strong> Sharkachi’s new book [11]. If you need conv<strong>in</strong>c<strong>in</strong>g that this theorem is worth study<strong>in</strong>g, take a look at how easily<br />

it disposes <strong>of</strong> this otherwise challeng<strong>in</strong>g exam problem. Also, Ste<strong>in</strong> <strong>and</strong> Sharkachi seem to have set th<strong>in</strong>gs up just right, so that the theorem is very<br />

easy to apply.<br />

59<br />

an


2.6 2006 November 13 2 COMPLEX ANALYSIS<br />

2.6 2006 November 13<br />

Notation: C is the set <strong>of</strong> complex numbers, D = {z ∈ C : |z| < 1} is the open unit disk, Π + <strong>and</strong> Π − are the upper<br />

<strong>and</strong> lower half-planes, respectively, <strong>and</strong>, given an open set G ⊂ C, H(G) is the set <strong>of</strong> holomorphic functions on G.<br />

1. (a) Suppose that f ∈ H(D \ {0}) <strong>and</strong> that |f(z)| < 1 for all 0 < |z| < 1. Prove that there is F ∈ H(D) with<br />

F (z) = f(z) for all z ∈ D \ {0}.<br />

(b) State a general theorem about isolated s<strong>in</strong>gularities for holomorphic functions.<br />

Solution:<br />

Lemma 2.1 Suppose G ⊂ C is an open set <strong>and</strong> f is holomorphic <strong>in</strong> G except for an isolated s<strong>in</strong>gularity at z0 ∈ G.<br />

If<br />

lim sup |f(z)| < ∞,<br />

z→z0<br />

z∈G<br />

then z0 is a removable s<strong>in</strong>gularity <strong>and</strong> f may be extended holomorphically to all <strong>of</strong> G.<br />

Pro<strong>of</strong>: Under the stated hypotheses, there is an ɛ > 0 <strong>and</strong> an M > 0 such that the deleted neighborhood B o <br />

{z ∈ C : 0 < |z − z0| ≤ ɛ} is conta<strong>in</strong>ed <strong>in</strong> G <strong>and</strong> such that |f(z)| ≤ M for all z ∈ B o .<br />

Let<br />

f(z) =<br />

be the Laurent expansion <strong>of</strong> f for z ∈ B o , where<br />

an = 1<br />

2πi<br />

∞<br />

n=−∞<br />

<br />

C<br />

an(z − z0) n<br />

f(ζ)<br />

dζ.<br />

(ζ − z0) n+1<br />

Here C denotes the positively oriented circle |ζ − z0| = ɛ. Chang<strong>in</strong>g variables,<br />

the coefficients are<br />

Therefore,<br />

ζ = z0 + ɛe iθ ⇒ dζ = i ɛe iθ dθ<br />

an = 1<br />

2π<br />

2π i 0<br />

f(z0 + ɛe iθ )<br />

(z0 + ɛe iθ − z0) n+1 iɛeiθ dθ.<br />

|an| ≤ 1<br />

2π<br />

M M<br />

ɛd|θ| = ,<br />

2π 0 ɛn+1 ɛn which makes it clear that, if n < 0, then |an| can be made arbitrarily small, by choos<strong>in</strong>g a sufficiently small ɛ. This<br />

proves that an = 0 for negative n, <strong>and</strong> so<br />

f(z) =<br />

∞<br />

an(z − z0) n .<br />

n=0<br />

Thus, f ∈ H(G). ⊓⊔<br />

The lemma solves part (a) <strong>and</strong> is also an example <strong>of</strong> a general theorem about isolated s<strong>in</strong>gularities <strong>of</strong> holomorphic<br />

functions, so it answers part (b). Here is another answer to part (b):<br />

60


2.6 2006 November 13 2 COMPLEX ANALYSIS<br />

Theorem 2.4 (Criterion for a pole) Let G ⊂ C be open. <strong>and</strong> suppose f(z) is holomorphic for all z ∈ G except<br />

for an isolated s<strong>in</strong>gularity at z = z0 ∈ G. Then<br />

(i) z0 is a pole <strong>of</strong> f if <strong>and</strong> only if limz→z0 |f(z)| = ∞;<br />

(ii) if m > 0 is the smallest <strong>in</strong>teger such that limz→z0 |(z − z0) mf(z)| rema<strong>in</strong>s bounded, then z0 is a pole <strong>of</strong><br />

order m.<br />

2. (a) Explicitly construct, through a sequence <strong>of</strong> mapp<strong>in</strong>gs, a one-to-one holomorphic function mapp<strong>in</strong>g the disk D<br />

onto the half disk D ∩ Π + .<br />

(b) State a general theorem concern<strong>in</strong>g one-to-one mapp<strong>in</strong>gs <strong>of</strong> D onto doma<strong>in</strong>s Ω ⊂ C.<br />

Solution: (a) 51 Let φ0(z) = 1−z<br />

1+z . Our strategy will be to show that φ0 maps the fourth quadrant onto D ∩ Π + ,<br />

<strong>and</strong> then to construct a conformal mapp<strong>in</strong>g, f, <strong>of</strong> the unit disk onto the fourth quadrant. Then the composition<br />

φ0 ◦ f will have the desired properties.<br />

Consider the boundary <strong>of</strong> the first quadrant. Note that φ0 maps the real l<strong>in</strong>e onto itself. Furthermore, φ0 takes 0 to<br />

1 <strong>and</strong> takes ∞ to -1. S<strong>in</strong>ce φ0(1) = 0, we see that the positive real axis (0, ∞) is mapped onto the segment (−1, 1).<br />

Now, s<strong>in</strong>ce φ0 maps the right half-plane P + onto the unit disk, it must map the boundary <strong>of</strong> P + (i.e., the imag<strong>in</strong>ary<br />

axis) onto the boundary <strong>of</strong> the unit disk. Thus, as 0 ↦→ 1 <strong>and</strong> ∞ ↦→ −1, the positive imag<strong>in</strong>ary axis is mapped to<br />

either the upper half-circle or the lower half-circle, <strong>and</strong> similarly for the negative imag<strong>in</strong>ary axis. Check<strong>in</strong>g that<br />

φ0(i) = −i, it is clear that the positive imag<strong>in</strong>ary axis is mapped to the lower half-circle {e iθ : −π < θ < 0}.<br />

Therefore, <strong>in</strong> mapp<strong>in</strong>g the right half-plane onto the unit disk, φ0 maps the first quadrant to the lower half-disk<br />

D ∩ Π − , <strong>and</strong> must therefore map the fourth quadrant to the upper half-disk. That is, φ0 : Q4 → D ∩ Π + , where<br />

Q4 = {z ∈ C : Rez > 0, Imz < 0}.<br />

Next construct a mapp<strong>in</strong>g <strong>of</strong> the unit disk onto the fourth quadrant as follows: If φ1(z) = iz, then φ1 ◦ φ0 : D →<br />

Π + . Let φ2(z) = z 1/2 be a branch <strong>of</strong> the square root function on Π + . Then φ2 maps Π + onto the first quadrant,<br />

Q1 = {z ∈ C : Rez > 0, Imz > 0}. F<strong>in</strong>ally, let φ3(z) = e −iπ/2 z = −iz, which takes the first quadrant to the<br />

fourth quadrant. Then, s<strong>in</strong>ce all <strong>of</strong> the mapp<strong>in</strong>gs are conformal bijections, f = φ3 ◦ φ2 ◦ φ1 ◦ φ0 is a conformal<br />

bijection <strong>of</strong> D onto Q4. Therefore, φ0 ◦ f is a conformal bijection <strong>of</strong> D onto D ∩ Π + . ⊓⊔<br />

(b) (Riemann) 52 Let Ω ⊂ C be a simply connected region such that Ω = C. Then Ω is conformally equivalent<br />

to D. That is, there is a conformal bijection, φ, <strong>of</strong> Ω onto the unit disk. Moreover, if we specify that a particular<br />

z0 ∈ Ω must be mapped to 0, <strong>and</strong> we specify the value <strong>of</strong> arg φ(z0), then the conformal mapp<strong>in</strong>g is unique.<br />

3. 53 (a) State the Schwarz lemma.<br />

(b) Suppose that f ∈ H(Π + ) <strong>and</strong> that |f(z)| < 1 for all z ∈ Π + . If f(i) = 0 how large can |f ′ (i)| be? F<strong>in</strong>d the<br />

extremal functions.<br />

Solution: (a) See theorem A.22.<br />

51 This problem also appears <strong>in</strong> April ’89 (3) <strong>and</strong> April ’95 (5).<br />

52 Look up the precise statement <strong>of</strong> the Riemann mapp<strong>in</strong>g theorem.<br />

53 A very similar problem appeared <strong>in</strong> April ’95 (6).<br />

61<br />

⊓⊔


2.6 2006 November 13 2 COMPLEX ANALYSIS<br />

(b) In order to apply the Schwarz lemma, map the disk to the upper half-plane with the Möebius map φ : D → Π +<br />

def<strong>in</strong>ed by<br />

1 − z<br />

φ(z) = i<br />

1 + z .<br />

Then, φ(0) = i. Therefore, the function g = f ◦ φ : D φ −→ Π + f −→ D satisfies |g(z)| ≤ 1 <strong>and</strong> g(0) = f(φ(0)) =<br />

f(i) = 0. By Schwarz’s lemma, then, |g ′ (0)| ≤ 1. F<strong>in</strong>ally, observe that g ′ (z) = f ′ (φ(z))φ ′ (z), <strong>and</strong> then check<br />

that φ ′ (0) = −2i. Whence, g ′ (0) = f ′ (φ(0))φ ′ (0) = f ′ (i)(−2i), which implies 1 ≥ |g ′ (0)| = 2|f ′ (i)|. Therefore<br />

|f ′ (i)| ≤ 1/2. ⊓⊔<br />

4. (a) State Cauchy’s theorem <strong>and</strong> its converse.<br />

(b) Suppose that f is a cont<strong>in</strong>uous function def<strong>in</strong>ed on the entire complex plane. Assume that<br />

(i) f ∈ H(Π + ∪ Π − )<br />

(ii) f(¯z) = f(z) all z ∈ C.<br />

Prove that f is an entire function.<br />

Solution: (a) See theorems A.16 <strong>and</strong> A.18.<br />

(b) See Marsden <strong>and</strong> H<strong>of</strong>fman.<br />

5. (a) Def<strong>in</strong>e what it means for a family F ⊂ H(Ω) to be a normal family. State the fundamental theorem for normal<br />

families.<br />

(b) Suppose f ∈ H(Π + ) <strong>and</strong> |f(z)| < 1 all z ∈ Π + . Suppose further that<br />

lim t → 0+f(it) = 0.<br />

Prove that f(zn) → 0 whenever the sequence zn → 0 <strong>and</strong> zn ∈ Γ where<br />

H<strong>in</strong>t. Consider the functions ft(z) = f(tz) where t > 0.<br />

Γ = {z ∈ Π + : |Rez| ≤ Imz}.<br />

Solution: (a) Let Ω be an open subset <strong>of</strong> the plane. A family F <strong>of</strong> functions <strong>in</strong> Ω is called a normal family if<br />

every sequence <strong>of</strong> functions <strong>in</strong> F has a subsequence which converges locally uniformly <strong>in</strong> Ω. (The same def<strong>in</strong>ition<br />

applies when the family F happens to be conta<strong>in</strong>ed <strong>in</strong> H(Ω).) 54<br />

I th<strong>in</strong>k <strong>of</strong> the Arzela-Ascoli theorem as the fundamental theorem for normal families. However, s<strong>in</strong>ce the exam<strong>in</strong>ers<br />

asked specifically about the special case when F is a family <strong>of</strong> holomorphic functions, they probably had <strong>in</strong> m<strong>in</strong>d<br />

the version <strong>of</strong> Montel’s theorem stated below, which is an easy consequence <strong>of</strong> the Arzela-Ascoli theorem. 55<br />

Theorem 2.5 (Arzela-Ascoli) Let F ⊂ C(Ω, S) be a family <strong>of</strong> cont<strong>in</strong>uous functions from an open set Ω ⊆ C <strong>in</strong>to<br />

a metric space (S, d). Then F is a normal family if <strong>and</strong> only if<br />

(i) F is equicont<strong>in</strong>uous on each compact subset <strong>of</strong> Ω, <strong>and</strong><br />

(ii) for each z ∈ Ω, the set {f(z) : f ∈ F} is conta<strong>in</strong>ed <strong>in</strong> a compact subset <strong>of</strong> S.<br />

54 Despite the word<strong>in</strong>g <strong>of</strong> the problem, the family need not satisfy F ⊂ H(Ω) <strong>in</strong> order to be normal.<br />

55 Problem 5 (b) <strong>of</strong> the November 2001 exam asks for a pro<strong>of</strong> <strong>of</strong> Montel’s theorem us<strong>in</strong>g the Arzela-Ascoli theorem.<br />

62


2.6 2006 November 13 2 COMPLEX ANALYSIS<br />

Recall that a family F <strong>of</strong> functions is called locally bounded on Ω iff for all compact K ⊂ Ω there is a constant<br />

MK such that |f(z)| ≤ MK for all f ∈ F <strong>and</strong> z ∈ K.<br />

Corollary 2.3 (little Montel theorem 56 ) Assume the set-up <strong>of</strong> the Arzela-Ascoli theorem, <strong>and</strong> suppose S = C<br />

<strong>and</strong> F ⊂ H(Ω). Then F is a normal family if <strong>and</strong> only if it is locally bounded.<br />

(b) Fix a sequence {zn} ⊂ Γ with zn → 0 as n → ∞. We must prove f(zn) → 0. Def<strong>in</strong>e fn(z) = f(|zn|z).<br />

Then, s<strong>in</strong>ce z ∈ Γ ⇒ |zn|z ∈ Γ, we have<br />

|fn(z)| = |f(|zn|z)| < 1, for all z ∈ Γ <strong>and</strong> n ∈ N.<br />

Therefore, F is a normal family <strong>in</strong> Γ. Also note that each fn is holomorphic <strong>in</strong> Γ s<strong>in</strong>ce f(tz) ∈ H(Γ) for any<br />

constant t > 0. Thus, F is a normal family <strong>of</strong> holomorphic functions <strong>in</strong> Γ.<br />

Let g be a normal limit <strong>of</strong> {fn}; i.e., there is some subsequence nk such that, as k → ∞, fnk → g locally uniformly<br />

<strong>in</strong> Γ.<br />

Consider the po<strong>in</strong>t z = i. S<strong>in</strong>ce f(it) → 0 as t ↓ 0,<br />

g(i) = lim fnk (i) = lim f(|znk |i) = 0.<br />

k→∞ k→∞<br />

In fact, for any po<strong>in</strong>t z = iy with y > 0, we have g(z) = 0. S<strong>in</strong>ce g is holomorphic <strong>in</strong> Γ, the identity theorem<br />

implies that g ≡ 0 <strong>in</strong> Γ.<br />

Next, consider<br />

fn<br />

<br />

zn<br />

= f |zn|<br />

|zn|<br />

zn<br />

<br />

= f(zn). (47)<br />

|zn|<br />

The numbers zn/|zn| lie <strong>in</strong> the compact set γ = {z ∈ Γ : |z| = 1}. S<strong>in</strong>ce fnk → g uniformly <strong>in</strong> γ, for any ɛ > 0,<br />

there is a K > 0 such that |fnk (z) − g(z)| = |fnk (z)| < ɛ, for all k ≥ K <strong>and</strong> all z ∈ γ. That is,<br />

lim sup{|fnk (z)| : z ∈ γ} lim<br />

k→∞ k→∞ fnkγ = 0,<br />

<strong>and</strong>, s<strong>in</strong>ce znk /|znk | ∈ γ, <br />

znk<br />

fnk<br />

|znk |<br />

<br />

<br />

≤ fnkγ. ∴ lim<br />

k→∞ fnk<br />

<br />

znk<br />

|znk |<br />

<br />

= 0.<br />

By (47), then, limk→∞ f(znk ) = 0.<br />

F<strong>in</strong>ally, recall that f(zn) → 0 iff every subsequence znj has a further subsequence znj k such that f(znj k ) → 0,<br />

as k → ∞. Now, if znj is any subsequence, then {f(znj )} is a normal family, <strong>and</strong>, repeat<strong>in</strong>g the argument above,<br />

there is, <strong>in</strong>deed, a further subsequence znj k such that f(znj k ) → 0. This completes the pro<strong>of</strong>. ⊓⊔<br />

Remark: In the last paragraph, we made use <strong>of</strong> the fact that a sequence converges to zero iff any subsequence has,<br />

<strong>in</strong> turn, a further subsequence that converges to zero. An alternative conclud<strong>in</strong>g argument that doesn’t rely on this<br />

result, but proceeds by way <strong>of</strong> contradiction, runs as follows: Assume we have already shown limk→∞ f(znk ) = 0,<br />

as above, <strong>and</strong> suppose f(zn) does not converge to 0 as n → ∞. Then there is a δ > 0 <strong>and</strong> a subsequence {znj }<br />

such that |f(znj )| > δ for all j ∈ N. Relabel this subsequence {zn}. Then {f(zn)} is itself a normal family <strong>and</strong><br />

we can repeat the argument above to get a further subsequence {znk } with limk→∞ f(znk ) = 0. This contradicts<br />

the assumption that |f(zn)| > δ for all n ∈ N. Therefore, f(zn) → 0, as desired.<br />

63


2.7 2007 April 16 2 COMPLEX ANALYSIS<br />

2.7 2007 April 16<br />

Notation: C is the set <strong>of</strong> complex numbers, D = {z ∈ C : |z| < 1}, <strong>and</strong>, for any open set G ⊂ C, H(G) is the set <strong>of</strong><br />

holomorphic functions on G.<br />

1<br />

1. Give the Laurent series expansion <strong>of</strong> z(z−1) <strong>in</strong> the region A ≡ {z ∈ C : 2 < |z + 2| < 3}.<br />

Solution:<br />

f(z) =<br />

1 1 − z + z 1 1<br />

= = −<br />

z(z − 1) z(z − 1) z − 1 z .<br />

Let u = z + 2. Then z = u − 2 <strong>and</strong> A = {u ∈ C : 2 < |u| < 3}. Therefore,<br />

1 1 1 1<br />

= =<br />

z u − 2 u (1 − 2/u)<br />

= 1<br />

u<br />

∞<br />

n=0<br />

n 2<br />

u<br />

converges for |u| > 2 <strong>and</strong>, substitut<strong>in</strong>g u = z + 2 <strong>in</strong> the last expression, we have<br />

1 1<br />

=<br />

z z + 2<br />

∞<br />

n=0<br />

n 2<br />

=<br />

z + 2<br />

converg<strong>in</strong>g for 2 < |z + 2|. Next, consider that<br />

∞<br />

n=0<br />

2 n (z + 2) −n−1 =<br />

1 1<br />

=<br />

z − 1 u − 3 =<br />

−1 −1<br />

=<br />

3(1 − u/3) 3<br />

−1<br />

n=−∞<br />

∞<br />

n=0<br />

n+1 1<br />

(z + 2)<br />

2<br />

n ,<br />

<br />

u<br />

n 3<br />

converges for |u| < 3 <strong>and</strong>, substitut<strong>in</strong>g u = z + 2 <strong>in</strong> the last expression, we have<br />

converg<strong>in</strong>g for |z + 2| < 3. Therefore,<br />

f(z) = 1 1<br />

−<br />

z − 1 z<br />

1<br />

z − 1<br />

= −<br />

∞<br />

n=0<br />

= −<br />

∞<br />

n=0<br />

n+1 1<br />

3<br />

n+1 1<br />

(z + 2)<br />

3<br />

n ,<br />

(z + 2) n −<br />

−1<br />

n=−∞<br />

n+1 1<br />

(z + 2)<br />

2<br />

n ,<br />

for z ∈ A. ⊓⊔<br />

2. (i) Prove: Suppose that for all z ∈ D <strong>and</strong> all n ∈ N we have that fn is holomorphic <strong>in</strong> D <strong>and</strong> |fn(z)| < 1. Also<br />

suppose that limn→∞ Imfn(x) = 0 for all x ∈ (−1, 0). Then limn→∞ Imfn(1/2) = 0.<br />

(ii) Give a complete statement <strong>of</strong> the convergence theorem that you use <strong>in</strong> part (2i).<br />

Solution: (i)<br />

(ii)<br />

3. Use the residue theorem to evaluate ∞ 1<br />

−∞ 1+x4 dx.<br />

64


2.7 2007 April 16 2 COMPLEX ANALYSIS<br />

Solution: Note that<br />

f(z) =<br />

1<br />

=<br />

1 + x4 1<br />

(z 2 + i)(z 2 − i) =<br />

1<br />

(z + e iπ/4 )(z − e iπ/4 )(z + e i3π/4 )(z − e i3π/4 ) ,<br />

which reveals that the poles <strong>of</strong> f <strong>in</strong> the upper half plane are at e iπ/4 <strong>and</strong> e i3π/4 . Let ΓR be the contour shown <strong>in</strong><br />

the figure below; i.e., ΓR = g(R) ∪ [−R, R], where R > 1. Then, by the residue theorem,<br />

<br />

ΓR<br />

<br />

f(z)dz = 2πi Res(f, e iπ/4 ) + Res(f, e i3π/4 <br />

) . (48)<br />

The other two poles <strong>of</strong> f are <strong>in</strong> the lower half-plane, so both e iπ/4 <strong>and</strong> e i3π/4 are simple poles. Therefore,<br />

Res(f, e iπ/4 ) = lim<br />

z→eiπ/4 (z − e iπ/4 1<br />

)f(z) =<br />

2eiπ/4 (eiπ/4 − ei3π/4 )(eiπ/4 + ei3π/4 = −1<br />

) 4 ie−iπ/4 ,<br />

Res(f, e i3π/4 ) = lim<br />

z→ei3π/4 (z − e i3π/4 1<br />

)f(z) =<br />

2ei3π/4 (ei3π/4 − eiπ/4 )(ei3π/4 + eiπ/4 1<br />

=<br />

) 4 ie−i3π/4 .<br />

Plugg<strong>in</strong>g these <strong>in</strong>to (48) yields<br />

<br />

<br />

1<br />

f(z)dz = 2πi<br />

4 ie−i3π/4 − 1<br />

4 ie−iπ/4<br />

<br />

= π<br />

2 (e−iπ/4 − e −i3π/4 ) = π √ .<br />

2<br />

It rema<strong>in</strong>s to show<br />

ΓR<br />

lim<br />

R→∞<br />

Chang<strong>in</strong>g variables via z = Reiθ (0 ≤ θ ≤ π),<br />

<br />

<br />

<br />

<br />

<br />

<br />

f(z)dz<br />

<br />

=<br />

<br />

π<br />

<br />

<br />

g(R)<br />

0<br />

<br />

<br />

<br />

<br />

<br />

g(R)<br />

iRe iθ<br />

1 + (Re iθ ) 4<br />

<br />

<br />

<br />

f(z)dz<br />

= 0.<br />

<br />

<br />

<br />

<br />

<br />

πR<br />

≤<br />

R4 → 0, as R → ∞.<br />

− 1<br />

4. Present a function f that has all <strong>of</strong> the follow<strong>in</strong>g properties: (i) f is one-to-one <strong>and</strong> holomorphic on D. (ii)<br />

{f(z) : z ∈ D} = {w ∈ C : Rew > 0 <strong>and</strong> Imw > 0}. (iii) f(0) = 1 + i.<br />

65<br />

⊓⊔


2.7 2007 April 16 2 COMPLEX ANALYSIS<br />

Solution: First consider 57 φ1(z) = 1−z<br />

1+z , which maps D onto the right half-plane P + = {z ∈ C : Rez > 0}.<br />

Let φ2(z) = e iπ/2 z = iz, which maps P + onto the upper half-plane Π + = {z ∈ C : Imz > 0}. Next,<br />

let φ3(z) = z 1/2 be a branch <strong>of</strong> the square root function on Π + . Then φ3 maps Π + onto the first quadrant<br />

Q1 = {z ∈ C : 0 < arg(z) < π/2}.<br />

The function φ = φ3 ◦ φ2 ◦ φ1 satisfies the first two conditions, so we check whether it satisfies condition (iii):<br />

φ1(0) = 1 ⇒ (φ2 ◦ φ1)(0) = φ2(1) = i ⇒ (φ3 ◦ φ2 ◦ φ1)(0) = φ3(i) =<br />

so apparently we’re <strong>of</strong>f by a factor <strong>of</strong> √ 2. This is easy to fix: let φ4(z) = √ 2z. Then the holomorphic function<br />

f φ4 ◦ φ maps D bijectively onto Q1 <strong>and</strong> f(0) = 1 + i, as desired. ⊓⊔<br />

5. (i) Prove: If f : D → D is holomorphic <strong>and</strong> f(1/2) = 0, then |f(0)| ≤ 1/2.<br />

(ii) Give a complete statement <strong>of</strong> the maximum modulus theorem that you use <strong>in</strong> part (i).<br />

Solution: (i) Def<strong>in</strong>e φ(z) = 1/2−z<br />

1−z/2 . This is a holomorphic bijection58 <strong>of</strong> ¯ D onto ¯ D. Therefore, g = f ◦φ ∈ H(D),<br />

|g(z)| ≤ 1 for all z ∈ D, <strong>and</strong> g(0) = f(φ(0)) = f(1/2) = 0. Thus g satisfies the hypotheses <strong>of</strong> Schwarz’s lemma<br />

(theorem A.22), which allows us to conclude the follow<strong>in</strong>g:<br />

(a) |g(z)| ≤ |z|, for all z ∈ D, <strong>and</strong><br />

(b) |g ′ (0)| ≤ 1,<br />

with equality <strong>in</strong> (a) for some z ∈ D or equality <strong>in</strong> (b) iff g(z) = e iθ z for some constant θ ∈ R. By condition (a),<br />

1/2 ≥ |g(1/2)| = |f(φ(1/2))| = |f(0)|.<br />

(ii) In part (i) I used Schwarz’s lemma, a complete statement <strong>of</strong> which appears <strong>in</strong> the appendix (theorem A.22).<br />

This is sometimes thought <strong>of</strong> as a version <strong>of</strong> the maximum modulus pr<strong>in</strong>ciple s<strong>in</strong>ce it is such an easy corollary <strong>of</strong><br />

what is usually called the maximum modulus pr<strong>in</strong>ciple. Here is a complete statement <strong>of</strong> the latter:<br />

(max modulus pr<strong>in</strong>ciple, version 1)<br />

Suppose G ⊂ C is open <strong>and</strong> f ∈ H(G) atta<strong>in</strong>s its maximum modulus at some po<strong>in</strong>t a ∈ G. Then f is constant.<br />

That is, if there is a po<strong>in</strong>t a ∈ G with |f(z)| ≤ |f(a)| for all z ∈ G, then f is constant. 59<br />

6. Prove: If G is a connected open subset <strong>of</strong> C, any two po<strong>in</strong>ts <strong>of</strong> G can be connected by a parametric curve <strong>in</strong> G.<br />

57 This is my favorite Möebius map. Not only does it map the unit disk onto the right half-plane, but also it maps the right half-plane onto the<br />

unit disk. This feature makes φ1 an extremely useful tool for conformal mapp<strong>in</strong>g problems, where you’re frequently required to map half-planes to<br />

the unit disk <strong>and</strong> vice-versa. Another nice feature <strong>of</strong> this map is that φ −1<br />

1 = φ1. (Of course this must be the case if φ1 is to have the first feature.)<br />

Also note that, like all l<strong>in</strong>ear fractional transformations, φ1 is a holomorphic bijection <strong>of</strong> C. Therefore, if φ1 is to map the <strong>in</strong>terior <strong>of</strong> the unit disk<br />

to the right half-plane, it must also map the exterior <strong>of</strong> the unit disk to the left half-plane.<br />

58 See Rud<strong>in</strong> [8] page 254-5 (<strong>in</strong> particular, theorem 12.4) for a nice discussion <strong>of</strong> functions <strong>of</strong> the form φα(z) = z−α<br />

. In addition to 12.4,<br />

1− ¯αz<br />

sec. 12.5 <strong>and</strong> theorem 12.6 are popular exam questions.<br />

59There are a couple <strong>of</strong> other versions <strong>of</strong> the maximum modulus pr<strong>in</strong>ciple you should know, though for most problems on the comprehensive<br />

exams, the version above usually suffices. The other two versions are stated <strong>and</strong> proved clearly <strong>and</strong> concisely <strong>in</strong> Conway [3], but they also appear<br />

as theorems A.20 <strong>and</strong> A.21 <strong>of</strong> the appendix.<br />

66<br />

1 + i<br />

√ 2<br />

⊓⊔


2.7 2007 April 16 2 COMPLEX ANALYSIS<br />

Solution: First, recall that if A ⊂ G ⊂ C, then A is said to be open relative to G, or simply open <strong>in</strong> G, if for any<br />

a ∈ A there is a neighborhood B(a, ɛ) = {z ∈ C : |z − a| < ɛ} such that B(a, ɛ) ∩ G ⊂ A. 60<br />

Next, recall that a subset G ⊂ C is connected iff the only subsets <strong>of</strong> G that are both open <strong>and</strong> closed relative to G<br />

are the empty set <strong>and</strong> G itself. Equivalently, if there exist non-empty disjo<strong>in</strong>t subsets A, B ⊂ G that are open <strong>in</strong> G<br />

<strong>and</strong> have the property G = A ∪ B, then G is not connected, or disconnected. 61<br />

Now, suppose G is a connected open subset <strong>of</strong> C. Fix z0 ∈ G <strong>and</strong> let Ω ⊂ G be the subset <strong>of</strong> po<strong>in</strong>ts that can<br />

be connected to z0 by a parametric curve <strong>in</strong> G. S<strong>in</strong>ce G is open, ∃B(z0, ɛ) ⊂ G for some ɛ > 0, <strong>and</strong> clearly<br />

B(z0, ɛ) ⊂ Ω. In particular, Ω = ∅ . If we can show Ω is both open <strong>and</strong> closed <strong>in</strong> G, then it will follow by<br />

connectedness that Ω = G, <strong>and</strong> the problem will be solved.<br />

(Ω is open) Let w ∈ Ω be connected to z0 by a parametric curve γ ⊂ G. S<strong>in</strong>ce G is open, ∃ɛ > 0 such that<br />

B(w, ɛ) ⊂ G. Clearly any w1 ∈ B(w, ɛ) can be connected to z0 by a parametric curve (from w1 to w, then from w<br />

to z0 via γ) that rema<strong>in</strong>s <strong>in</strong> G. This proves that B(w, ɛ) ⊂ Ω, so Ω is open.<br />

(Ω is closed) We show G \ Ω is open (<strong>and</strong> thus, <strong>in</strong> fact, empty). If z ∈ G \ Ω, then, s<strong>in</strong>ce G is open, ∃δ > 0<br />

such that B(z, δ) ⊂ G. We want B(z, δ) ⊂ G \ Ω. This must be true s<strong>in</strong>ce, otherwise, there would be a po<strong>in</strong>t<br />

z1 ∈ B(z, δ) ∩ Ω which could be connected to both z <strong>and</strong> z0 by parametric curves <strong>in</strong> G. But then a parametric<br />

curve <strong>in</strong> G connect<strong>in</strong>g z to z0 could be constructed, which would put z <strong>in</strong> Ω – a contradiction.<br />

We have thus shown that Ω is both open <strong>and</strong> closed <strong>in</strong> G, as well as non-empty. S<strong>in</strong>ce G is connected, Ω = G. ⊓⊔<br />

60 For example, the set A = [0, 1], although closed <strong>in</strong> C, is open <strong>in</strong> G = [0, 1] ∪ {2}.<br />

61 To see the equivalence note that, <strong>in</strong> this case, A is open <strong>in</strong> G, as is A c = G \ A = B, so A is both open <strong>and</strong> closed <strong>in</strong> G. Also it is <strong>in</strong>structive<br />

to check, us<strong>in</strong>g either def<strong>in</strong>ition, that G = [0, 1] ∪ {2} is disconnected.<br />

67


2.8 2007 November 16 2 COMPLEX ANALYSIS<br />

2.8 2007 November 16<br />

Do as many problems as you can. Complete solutions (except for m<strong>in</strong>or flaws) to 5 problems would be considered an<br />

excellent performance. Fewer than 5 complete solutions may still be pass<strong>in</strong>g, depend<strong>in</strong>g on the quality.<br />

1. Let G be a bounded open subset <strong>of</strong> the complex plane. Suppose f is cont<strong>in</strong>uous on the closure <strong>of</strong> G <strong>and</strong> analytic<br />

on G. Suppose further that there is a constant c ≥ 0 such that |f| = c for all z on the boundary <strong>of</strong> G. Show that<br />

either f is constant on G or f has a zero <strong>in</strong> G.<br />

2. (a) State the residue theorem.<br />

(b) Use contour <strong>in</strong>tegration to evaluate<br />

∞<br />

0<br />

x2 (x2 dx.<br />

+ 1) 2<br />

Important: You must carefully: specify your contours, prove the <strong>in</strong>equalities that provide your limit<strong>in</strong>g arguments,<br />

<strong>and</strong> show how to evaluate all relevant residues.<br />

3. (a) State the Schwarz lemma.<br />

(b) Suppose f is holomorphic <strong>in</strong> D = {z : |z| < 1} with f(D) ⊂ D. Let fn denote the composition <strong>of</strong> f with<br />

itself n times (n = 2, 3, . . . ). Show that if f(0) = 0 <strong>and</strong> |f ′ (0)| < 1, then {fn} converges to 0 locally uniformly<br />

on D.<br />

4. Exhibit a conformal mapp<strong>in</strong>g <strong>of</strong> the region common to the two disks |z| < 1 <strong>and</strong> |z − 1| < 1 onto the region <strong>in</strong>side<br />

the unit circle |z| = 1.<br />

5. Let {fn} be a sequence <strong>of</strong> functions analytic <strong>in</strong> the complex plane C, converg<strong>in</strong>g uniformly on compact subsets <strong>of</strong><br />

Cto a polynomial p <strong>of</strong> positive degree m. Prove that, if n is sufficiently large, then fn has at least m zeros (count<strong>in</strong>g<br />

multiplicities).<br />

Do not simply refer to Hurwitz’s theorem; prove this version <strong>of</strong> it.<br />

6. Let (X, d) be a metric space.<br />

(a) Def<strong>in</strong>e what it means for a subset K ⊂ X to be compact.<br />

(b) Prove (us<strong>in</strong>g your def<strong>in</strong>ition <strong>in</strong> (a)) that K ⊂ X is compact implies that K is both closed <strong>and</strong> bounded <strong>in</strong> X.<br />

(c) Give an example that shows the converse <strong>of</strong> the statement <strong>in</strong> (b) is false.<br />

Please email comments, suggestions, <strong>and</strong> corrections to williamdemeo@gmail.com.<br />

68


2.9 Some problems <strong>of</strong> a certa<strong>in</strong> type 2 COMPLEX ANALYSIS<br />

2.9 Some problems <strong>of</strong> a certa<strong>in</strong> type<br />

Collected <strong>in</strong> this section are miscellaneous problems about such th<strong>in</strong>gs as what can be said <strong>of</strong> a holomorphic (or<br />

harmonic) function when given <strong>in</strong>formation about how it behaves near a boundary or near <strong>in</strong>f<strong>in</strong>ity.<br />

Behavior near <strong>in</strong>f<strong>in</strong>ity<br />

1. If f(z) is an entire function which tends to <strong>in</strong>f<strong>in</strong>ity as z tends to <strong>in</strong>f<strong>in</strong>ity, then f(z) is a polynomial.<br />

2. If f(z) is an <strong>in</strong>jective entire function, then f(z) = az + b for some constants a <strong>and</strong> b.<br />

3. If u(z) is a nonconstant real valued harmonic function <strong>of</strong> C, then there is a sequence {zn} ⊂ C with zn → ∞ <strong>and</strong><br />

u(zn) → 0 as n → ∞.<br />

Behavior on or near the unit circle<br />

4. If f(z) is holomorphic <strong>in</strong> an open set conta<strong>in</strong><strong>in</strong>g the closed unit disk, <strong>and</strong> if f(e iθ ) is real for all θ ∈ R, then f(z)<br />

is constant.<br />

5. Prove or disprove: There exists a function f(z) holomorphic on the unit disk D such that |f(zn)| → ∞ whenever<br />

{zn} ⊂ D <strong>and</strong> |zn| → 1.<br />

6. Prove or disprove: There exists a function u(z) harmonic on the unit disk D such that |u(zn)| → ∞ whenever<br />

{zn} ⊂ D <strong>and</strong> |zn| → 1.<br />

Other <strong>Problems</strong><br />

7. If f is holomorphic <strong>in</strong> the punctured disk {0 < |z| < R} <strong>and</strong> if Ref ≤ M for some constant M, then 0 is a<br />

removable s<strong>in</strong>gularity.<br />

8. If f is holomorphic <strong>in</strong> the unit disk, with |f(z)| ≤ 1, f(0) = 0, <strong>and</strong> f(r) = f(−r) for some r ∈ (0, 1),<br />

then<br />

<br />

<br />

|f(z)| ≤ |z| <br />

z<br />

<br />

2 − r2 <br />

<br />

<br />

.<br />

1. See (1b) <strong>of</strong> April ’95.<br />

<strong>Solutions</strong><br />

1 − r 2 z 2<br />

2. Suppose f ∈ H(C) is <strong>in</strong>jective. Then f −1 is a cont<strong>in</strong>uous function <strong>in</strong> C which maps compact sets to compact<br />

sets. Therefore, if {zn} ⊂ C is any sequence tend<strong>in</strong>g to <strong>in</strong>f<strong>in</strong>ity, then the image f({zn}) cannot rema<strong>in</strong> <strong>in</strong>side any<br />

closed disk (s<strong>in</strong>ce f −1 maps all such disks to closed bounded sets <strong>in</strong> C). Thus f(z) → ∞ whenever z → ∞. By the<br />

previous problem, f is a polynomial. F<strong>in</strong>ally, if f has degree greater than one, or if f is constant, then f would not be<br />

<strong>in</strong>jective. Therefore, f is a polynomial <strong>of</strong> degree one. ⊓⊔<br />

3. See (4) <strong>of</strong> April ’95.<br />

4. See (1a) <strong>of</strong> April ’89.<br />

69


2.9 Some problems <strong>of</strong> a certa<strong>in</strong> type 2 COMPLEX ANALYSIS<br />

5. & 6. That both <strong>of</strong> these statements are false is a corollary <strong>of</strong> the next two lemmas.<br />

Lemma 1: If f ∈ H(D), then there is a sequence {zn} ⊂ D with |zn| → 1 such that the sequence {|f(zn)|} is<br />

bounded.<br />

Lemma 2: If u is harmonic <strong>in</strong> D, then there is a sequence {zn} ⊂ D with |zn| → 1 such that the sequence {u(zn)}<br />

is bounded.<br />

Pro<strong>of</strong> <strong>of</strong> Lemma 1: First suppose that f has <strong>in</strong>f<strong>in</strong>itely many zeros <strong>in</strong> D. Then, <strong>in</strong> any closed disk {|z| ≤ 1 − ɛ} ⊂ D,<br />

the zeros <strong>of</strong> f must be isolated (otherwise f ≡ 0). S<strong>in</strong>ce such a disk is compact, it conta<strong>in</strong>s only f<strong>in</strong>itely many zeros<br />

<strong>of</strong> f. We conclude that there must be a sequence <strong>of</strong> zeros <strong>of</strong> f tend<strong>in</strong>g to the boundary <strong>of</strong> D.<br />

Now suppose f has f<strong>in</strong>itely many zeros <strong>in</strong> D. Let {α1, . . . , αN} be the set <strong>of</strong> all zeros <strong>of</strong> f (count<strong>in</strong>g multiplicities).<br />

Then<br />

f(z) = (z − α1) · · · (z − αN)g(z),<br />

where g is holomorphic <strong>and</strong> non-zero <strong>in</strong> D. Therefore, the function 1/g is also holomorphic <strong>in</strong> D. By the maximum<br />

modulus pr<strong>in</strong>cipal, <strong>in</strong> each compact disk Dn = {|z| ≤ 1 − 1/n} (n ≥ 2), the function |1/g(z)| atta<strong>in</strong>s its maximum<br />

<strong>in</strong> Dn on the boundary at, say, the po<strong>in</strong>t zn, where |zn| = 1 − 1/n. The reciprocals <strong>of</strong> these maxima must satisfy<br />

|g(x2)| ≥ |g(x3)| ≥ · · · . Of course, the product (z − α1) · · · (z − αN) is bounded <strong>in</strong> D, so the sequence {|f(zn)|} is<br />

bounded. ⊓⊔<br />

Please email comments, suggestions, <strong>and</strong> corrections to williamdemeo@gmail.com.<br />

70


A Miscellaneous Def<strong>in</strong>itions <strong>and</strong> Theorems<br />

A.1 Real Analysis<br />

A.1.1 Metric Spaces<br />

The follow<strong>in</strong>g theorem is found <strong>in</strong> Conway [3].<br />

A MISCELLANEOUS DEFINITIONS AND THEOREMS<br />

Theorem A.1 Let (X, d) be a metric space; then the follow<strong>in</strong>g are equivalent statements:<br />

(a) X is compact;<br />

(b) Every <strong>in</strong>f<strong>in</strong>ite set <strong>in</strong> X has a limit po<strong>in</strong>t (<strong>in</strong> X);<br />

(c) X is sequentially compact<br />

(d) X is complete <strong>and</strong> for all ɛ > 0 there exist {x1, . . . , xn} ⊂ X such that<br />

n<br />

X = B(xk, ɛ)<br />

(The last property is called total boundedness.)<br />

A.1.2 Measurable Functions<br />

k=1<br />

Def<strong>in</strong>ition A.1 A complex function s on a measurable space X whose range consists <strong>of</strong> only f<strong>in</strong>itely many po<strong>in</strong>ts is<br />

called a simple function.<br />

If α1, . . . , αn are the dist<strong>in</strong>ct values <strong>of</strong> a simple function, <strong>and</strong> if we set Ai = {x ∈ X : s(x) = αi}, then clearly<br />

n<br />

s = αiχAi,<br />

i=1<br />

where χAi is the characteristic function <strong>of</strong> the set Ai. Note that the def<strong>in</strong>ition assumes noth<strong>in</strong>g about the sets Ai (<strong>in</strong><br />

particular, they may or may not be measurable). Thus, a simple function, as def<strong>in</strong>ed here, is not necessarily measurable.<br />

Cont<strong>in</strong>uous functions <strong>of</strong> cont<strong>in</strong>uous functions are cont<strong>in</strong>uous, <strong>and</strong> cont<strong>in</strong>uous functions <strong>of</strong> measurable functions<br />

are measurable. We state this as<br />

Theorem A.2 62 Let Y <strong>and</strong> Z be topological spaces, <strong>and</strong> let g : Y → Z be cont<strong>in</strong>uous.<br />

(a) If X is a topological space, if f : X → Y is cont<strong>in</strong>uous, <strong>and</strong> if h = g ◦ f, then h : X → Z is cont<strong>in</strong>uous.<br />

(b) If X is a measurable space, if f : X → Y is measurable, <strong>and</strong> if h = g ◦ f, then h : X → Z is measurable.<br />

Pro<strong>of</strong>: If V is open <strong>in</strong> Z, then g −1 (V ) is open <strong>in</strong> Y , <strong>and</strong> h −1 (V ) = (g ◦ f) −1 (V ) = f −1 (g −1 (V )). If f is cont<strong>in</strong>uous, then<br />

h −1 (V ) is open, prov<strong>in</strong>g (a). If f is measurable, then h −1 (V ) is measurable, prov<strong>in</strong>g (b). ⊓⊔<br />

Note, however, that measurable functions <strong>of</strong> cont<strong>in</strong>uous functions need not be measurable.<br />

Theorem A.3 63 Let u <strong>and</strong> v be real measurable functions on a measurable space X, let Φ be a cont<strong>in</strong>uous mapp<strong>in</strong>g <strong>of</strong><br />

the plane <strong>in</strong>to a topological space Y , <strong>and</strong> def<strong>in</strong>e<br />

Then h : X → Y is measurable.<br />

62 Theorem 1.7 <strong>of</strong> Rud<strong>in</strong> [8].<br />

63 Theorem 1.8 <strong>of</strong> Rud<strong>in</strong> [8].<br />

h(x) = Φ(u(x), v(x)) (x ∈ X).<br />

71


A.1 Real Analysis A MISCELLANEOUS DEFINITIONS AND THEOREMS<br />

A.1.3 Integration<br />

There are a h<strong>and</strong>ful <strong>of</strong> results that are the most essential, <strong>and</strong> lay the foundation on which everyth<strong>in</strong>g else is built.<br />

Rud<strong>in</strong> [8] gives a beautifully succ<strong>in</strong>ct <strong>and</strong> clear presentation <strong>of</strong> these <strong>in</strong> just seven pages (pp. 21–27). 64 Some <strong>of</strong> these<br />

results are presented below, but do yourself a favor <strong>and</strong> read the master [8].<br />

The follow<strong>in</strong>g theorem 65 is an essential <strong>in</strong>gredient <strong>of</strong> many pro<strong>of</strong>s (e.g. the pro<strong>of</strong> that simple functions are dense<br />

<strong>in</strong> L p , presented below).<br />

Theorem A.4 (Rud<strong>in</strong> Theorem 1.17) Let f : X → [0, ∞] be measurable. There exist simple measurable functions<br />

sn on X such that<br />

(a) 0 ≤ s1 ≤ s2 ≤ · · · ≤ f,<br />

(b) sn(x) → f(x) as n → ∞, for every x ∈ X.<br />

Theorem A.5 (Lebesgue’s Monotone Convergence Theorem) Let {fn} be a sequence <strong>of</strong> measurable functions on<br />

X, <strong>and</strong> suppose that, for every x ∈ X,<br />

(a) 0 ≤ f1(x) ≤ f2(x) ≤ · · · ≤ ∞,<br />

(b) fn(x) → f(x) as n → ∞.<br />

Then f is measurable, <strong>and</strong> <br />

X<br />

<br />

fn dµ →<br />

X<br />

f dµ as n → ∞.<br />

Theorem A.6 (Fatou’s lemma) If fn : X → [0, ∞] (n = 1, 2, . . . ) is a sequence <strong>of</strong> positive measurable functions,<br />

then <br />

<br />

lim <strong>in</strong>f fn ≤ lim <strong>in</strong>f<br />

Theorem A.7 (Lebesgue’s dom<strong>in</strong>ated convergence theorem) Let {fn} be a sequence <strong>of</strong> measurable functions on<br />

(X, M, µ) such that fn → f a.e. If there is another sequence <strong>of</strong> measurable functions {gn} satisfy<strong>in</strong>g<br />

(i) gn → g a.e.,<br />

(ii) gn → g < ∞, <strong>and</strong><br />

(iii) |fn(x)| ≤ gn(x) (x ∈ X; n = 1, 2, . . .),<br />

then f ∈ L 1 (X, M, µ), fn → f, <strong>and</strong> fn − f1 → 0.<br />

Theorem A.8 (Egor<strong>of</strong>f) If (X, M, µ) is a measure space, E ∈ M a set <strong>of</strong> f<strong>in</strong>ite measure, <strong>and</strong> {fn} a sequence <strong>of</strong><br />

measurable functions such that fn(x) → f(x) for almost every x ∈ E, then for all ɛ > 0 there is a measurable subset<br />

A ⊆ E such that fn → f uniformly on A <strong>and</strong> µ(E \ A) < ɛ.<br />

64 Study these seven pages until you can recite all seven theorems <strong>and</strong> their pro<strong>of</strong>s <strong>in</strong> your sleep. Also, pay attention to the details. Rud<strong>in</strong> is<br />

careful to choose def<strong>in</strong>itions <strong>and</strong> hypotheses that lend themselves to a succ<strong>in</strong>ct exposition, usually without too much loss <strong>of</strong> generality. For example,<br />

he sometimes takes the range <strong>of</strong> a “real-valued” function to be [−∞, ∞], rather than R. It is <strong>in</strong>structive to pause occasionally to consider how his<br />

arguments depend on such choices.<br />

65 I label it “Rud<strong>in</strong> Theorem 1.17” because I cited it as such so <strong>of</strong>ten when practic<strong>in</strong>g to take my comprehensive exams that the number stuck with<br />

me.<br />

72<br />

fn


A.1 Real Analysis A MISCELLANEOUS DEFINITIONS AND THEOREMS<br />

A.1.4 Approximat<strong>in</strong>g Integrable Functions 66<br />

Let X be a locally compact Hausdorff space. (See Rud<strong>in</strong> [8], section 2.3 for the def<strong>in</strong>ition.) Let Cc(X) denote<br />

the vector space <strong>of</strong> complex valued cont<strong>in</strong>uous functions on X with compact support (i.e. the closure <strong>of</strong> the set<br />

{x ∈ X : f(x) = 0} is compact). First, we present an important application <strong>of</strong> Rud<strong>in</strong> Theorem 1.17 (theorem A.4<br />

above) <strong>and</strong> the dom<strong>in</strong>ated convergence theorem (DCT).<br />

Theorem A.9 Let S be the set <strong>of</strong> all complex, measurable, simple functions on X such that<br />

If 1 ≤ p < ∞, then S is dense <strong>in</strong> L p (µ).<br />

Pro<strong>of</strong>: Let 1 ≤ p < ∞, <strong>and</strong> f ∈ L p (µ).<br />

µ({x ∈ X : s(x) = 0}) < ∞.<br />

Case 1. f ≥ 0.<br />

Let sn be as <strong>in</strong> Theorem 1.17. Then sn ≤ f implies sn ∈ L p (µ) for all n = 1, 2, . . . . Def<strong>in</strong>e gn = |f − sn| p . Then gn ∈ L p (µ),<br />

n = 1, 2, . . . , gn → 0 as n → ∞, <strong>and</strong> gn ≤ |f| p ∈ L 1 (µ). Therefore, by the DCT, gn → 0 as n → ∞. That is,<br />

f − snp → 0 as n → ∞.<br />

Case 2. f : X → [−∞, ∞].<br />

Let f = f + − f − where f + = max{0, f} <strong>and</strong> f − = m<strong>in</strong>{0, −f}. Let sn <strong>and</strong> tn be simple functions such that<br />

Then, by Case 1, we have<br />

as n → ∞. F<strong>in</strong>ally, note that<br />

0 ≤ s1(x) ≤ s2(x) ≤ · · · ≤ f + (x)<br />

0 ≤ t1(x) ≤ t2(x) ≤ · · · ≤ f − (x).<br />

f + − snp → 0 <strong>and</strong> f − − tnp → 0<br />

f − (sn − tn) p p = f + − f − − (sn − tn) p p ≤ f + − sn p p + f − − tn p p.<br />

Case 3. f : X → C.<br />

This case follows from Case 2 once we split f up <strong>in</strong>to real <strong>and</strong> complex parts. ⊓⊔<br />

The follow<strong>in</strong>g is a deep result whose pro<strong>of</strong> depends on Urysohn’s lemma. (For the pro<strong>of</strong>, see Rud<strong>in</strong> [8], section<br />

2.4.)<br />

Theorem A.10 (Lus<strong>in</strong>’s Theorem) Fix ɛ > 0. Let f be a complex measurable function which vanishes <strong>of</strong>f a set <strong>of</strong><br />

f<strong>in</strong>ite measure. Then there exists g ∈ Cc(X) such that<br />

Furthermore, we may arrange it so that<br />

µ({x ∈ X : f(x) = g(x)}) < ɛ.<br />

sup<br />

x<br />

∈ X|g(x)| ≤ sup |f(x)|<br />

x∈X<br />

Lus<strong>in</strong>’s theorem is a key <strong>in</strong>gredient <strong>in</strong> the follow<strong>in</strong>g. The short, elegant pro<strong>of</strong> is lifted directly from Rud<strong>in</strong>. (I won’t<br />

pretend I can improve on his masterpiece.)<br />

Theorem A.11 For 1 ≤ p < ∞, Cc(X) is dense <strong>in</strong> L p (µ).<br />

66 This topic is very important <strong>and</strong> appears on the comprehensive exam syllabus.<br />

73


A.1 Real Analysis A MISCELLANEOUS DEFINITIONS AND THEOREMS<br />

Pro<strong>of</strong>: Def<strong>in</strong>e S as <strong>in</strong> theorem A.9. If s ∈ S <strong>and</strong> ɛ > 0, then by Lus<strong>in</strong>’s Theorem there exists g ∈ Cc(X) such that g(x) = s(x)<br />

except on a set <strong>of</strong> measure < ɛ, <strong>and</strong> |g| ≤ s∞. Hence,<br />

g − sp ≤ 2ɛ 1/p s∞.<br />

S<strong>in</strong>ce S is dense <strong>in</strong> L p (µ), this completes the pro<strong>of</strong>. ⊓⊔<br />

We close out this subsection by giv<strong>in</strong>g a careful solution to a basic but important exercise from Rud<strong>in</strong> [8], Chapter<br />

2. First, def<strong>in</strong>e a step function to be a f<strong>in</strong>ite l<strong>in</strong>ear comb<strong>in</strong>ation <strong>of</strong> characteristic functions <strong>of</strong> bounded <strong>in</strong>tervals <strong>in</strong> R 1 .<br />

(Notice how much more special these functions are than the simple functions, def<strong>in</strong>ed above.)<br />

Lemma A.1 (Rud<strong>in</strong> Exercise 2.24) If f ∈ L 1 (R) then there exists a sequence {gn} <strong>of</strong> step functions on R such that<br />

lim<br />

n→∞ |f − gn| = 0.<br />

Pro<strong>of</strong>: We proceed <strong>in</strong> steps <strong>of</strong> successively greater generality. (I’ll fill <strong>in</strong> the details soon!)<br />

Case 1. f = χA for some measurable set A with µ(A) < ∞.<br />

Case 2. f = n i=1 αiχAi ∈ L1 (R).<br />

N.B. the assumption that f is <strong>in</strong>tegrable implies, <strong>in</strong> particular, that each Ai is measurable with µ(Ai) < ∞.<br />

Case 3. f ∈ L 1 (R), with f ≥ 0.<br />

Case 4. f ∈ L 1 (R).<br />

(Details com<strong>in</strong>g soon!)<br />

A.1.5 Absolute Cont<strong>in</strong>uity <strong>of</strong> Measures<br />

Two excellent sources for the material appear<strong>in</strong>g <strong>in</strong> this section are Rud<strong>in</strong> [8] (§ 6.7, 6.10) <strong>and</strong> Foll<strong>and</strong> [4] (§ 3.2).<br />

Let µ be a positive measure on a σ-algebra M, <strong>and</strong> let λ be an arbitrary complex measure on M. (Recall that<br />

the range <strong>of</strong> a complex measure is a subset <strong>of</strong> C, while a positive measure takes values <strong>in</strong> [0, ∞]. Thus the positive<br />

measures do not form a subclass <strong>of</strong> the complex measures.)<br />

Suppose, for any E ∈ M, that µ(E) = 0 ⇒ λ(E) = 0. In this case, we say that λ is absolutely cont<strong>in</strong>uous with<br />

respect to µ, <strong>and</strong> write λ ≪ µ. If there is a set A ∈ M such that, for all E ∈ M, λ(E) = λ(A ∩ E), then we say that<br />

λ is concentrated on A. Suppose λ1 <strong>and</strong> λ2 are measures on M <strong>and</strong> suppose there exists a pair <strong>of</strong> disjo<strong>in</strong>t sets A <strong>and</strong><br />

B such that λ1 is concentrated on A <strong>and</strong> λ2 is concentrated on B. Then we say that λ1 <strong>and</strong> λ2 are mutually s<strong>in</strong>gular,<br />

<strong>and</strong> write λ1 ⊥ λ2.<br />

Theorem A.12 (Lebesgue-Radon-Nikodym) 67 Let µ be a positive σ-f<strong>in</strong>ite measure on a σ-algebra M <strong>in</strong> a set X, <strong>and</strong><br />

let λ be a complex measure on M.<br />

(a) There is then a unique pair <strong>of</strong> complex measures λa <strong>and</strong> λs on M such that<br />

If λ is positive <strong>and</strong> f<strong>in</strong>ite, then so are λa <strong>and</strong> λs.<br />

(b) There is a unique h ∈ L 1 (µ) such that<br />

67 Rud<strong>in</strong>[8], 6.10.<br />

λ = λa + λs, λa ≪ µ, λs ⊥ µ.<br />

<br />

λa(E) =<br />

E<br />

74<br />

h dµ ∀E ∈ M.


A.1 Real Analysis A MISCELLANEOUS DEFINITIONS AND THEOREMS<br />

The pair (λa, λs) is called the Lebesgue decomposition <strong>of</strong> λ relative to µ. We call h the Radon-Nikodym derivative <strong>of</strong><br />

λa with respect to µ, <strong>and</strong> write h = dλa/dµ <strong>and</strong><br />

dλa = dλa<br />

dµ dµ.<br />

Strictly speak<strong>in</strong>g, dλa/dµ should be viewed as the equivalence class <strong>of</strong> functions that are equal to h µ-a.e.<br />

Corollary A.1 68 Suppose ν is a σ-f<strong>in</strong>ite complex measure <strong>and</strong> µ, λ are σ-f<strong>in</strong>ite measures on (X, M) such that ν ≪<br />

µ ≪ λ. Then<br />

(a) If g ∈ L 1 (ν), then g dν<br />

dµ ∈ L1 (µ) <strong>and</strong><br />

(b) ν ≪ λ, <strong>and</strong><br />

A.1.6 Absolute Cont<strong>in</strong>uity <strong>of</strong> Functions<br />

<br />

<br />

g dν =<br />

g dν<br />

dµ dµ.<br />

dν dν dµ<br />

=<br />

dλ dµ dλ λ-a.e.<br />

Lemma 1.2 Let f : R → R be a function. If f is differentiable on [a, b], f ′ ∈ L 1 ([a, b]), <strong>and</strong> x<br />

a f ′ (t)dt =<br />

f(x) − f(a) for a ≤ x ≤ b, then f ∈ AC[a, b].<br />

Pro<strong>of</strong> Assum<strong>in</strong>g the stated hypotheses, by a st<strong>and</strong>ard theorem, 69 f ′ ∈ L1 implies that for all ɛ > 0 there is a δ > 0<br />

such that, if E ⊂ R is measurable mE < δ, then<br />

<br />

|f ′ |dm < ɛ. (49)<br />

E<br />

Let A = ∪n i=1 (ai, bi) be a f<strong>in</strong>ite union <strong>of</strong> disjo<strong>in</strong>t open <strong>in</strong>tervals <strong>in</strong> [a, b] such that n i=1 (bi − ai) < δ. Then<br />

mA ≤ n i=1 (bi − ai) < δ, so<br />

n<br />

n<br />

<br />

<br />

bi<br />

<br />

|f(bi) − f(ai)| = f<br />

<br />

′ <br />

<br />

<br />

dm<br />

≤<br />

n<br />

bi<br />

|f ′ <br />

|dm = |f ′ |dm < ɛ (50)<br />

i=1<br />

i=1<br />

ai<br />

by (49). Thus, f ∈ AC[a, b]. ⊓⊔<br />

A.1.7 Product Measures <strong>and</strong> the Fub<strong>in</strong>i-Tonelli Theorem<br />

Let (X, S, µ) <strong>and</strong> (Y, T , ν) be measure spaces. If we want to construct a measurable space out <strong>of</strong> X × Y , it is natural<br />

to start by consider<strong>in</strong>g the collection <strong>of</strong> subsets S × T = {A × B ⊆ X × Y : A ∈ S, B ∈ T }. Note, however, that<br />

this collection is not, <strong>in</strong> general, an algebra <strong>of</strong> sets. To get an adequate collection on which to def<strong>in</strong>e product measure,<br />

then, def<strong>in</strong>e 70 S ⊗ T = σ(S × T ); that is, S ⊗ T is the σ-algebra generated by S × T .<br />

In my op<strong>in</strong>ion, the most useful version <strong>of</strong> the Fub<strong>in</strong>i <strong>and</strong> Tonelli theorems is the one <strong>in</strong> Rud<strong>in</strong> [8]. It beg<strong>in</strong>s<br />

by assum<strong>in</strong>g only that the function f(x, y) is measurable with respect to the product σ-algebra S ⊗ T . Then, <strong>in</strong> a<br />

s<strong>in</strong>gle, comb<strong>in</strong>ed Fub<strong>in</strong>i-Tonelli theorem, you get everyth<strong>in</strong>g you need to answer any <strong>of</strong> the st<strong>and</strong>ard questions about<br />

<strong>in</strong>tegration with respect to product measure. Here it is:<br />

68 Foll<strong>and</strong> [4], Prop. 3.9.<br />

69 The “st<strong>and</strong>ard theorem” cited here appears <strong>of</strong>ten on the comprehensive exams (cf. Nov. ’91 #6), but <strong>in</strong> a slightly weaker form <strong>in</strong> which the<br />

conclusion is that | <br />

E f ′ dm| < ɛ. In the present case we need <br />

E |f ′ |dm < ɛ to get the sum <strong>in</strong> (50) to come out right.<br />

70 This notation is not completely st<strong>and</strong>ard. In Aliprantis <strong>and</strong> Burk<strong>in</strong>shaw [2] (p. 154), for example, S ⊗ T denotes what we call S × T , while<br />

σ(S ⊗ T ) denotes what we have labeled S ⊗ T . At the opposite extreme, I believe Rud<strong>in</strong>[8] simply takes S × T to be the σ-algebra generated by<br />

the sets {A × B : A ∈ S, B ∈ T }.<br />

75<br />

i=1<br />

ai<br />

A


A.1 Real Analysis A MISCELLANEOUS DEFINITIONS AND THEOREMS<br />

Theorem A.13 (Fub<strong>in</strong>i-Tonelli)<br />

Assume (X, S, µ) <strong>and</strong> (Y, T , ν) are σ-f<strong>in</strong>ite measure spaces, <strong>and</strong> f(x, y) is a (S ⊗ T )-measurable function on X × Y .<br />

(a) If f(x, y) ≥ 0, <strong>and</strong> if φ(x) = <br />

<br />

f(x, y) dν(y) <strong>and</strong> ψ(y) = f(x, y) dµ(x), then φ is S-measurable, ψ is<br />

Y X<br />

T -measurable, <strong>and</strong> <br />

<br />

φ dµ = f(x, y) d(µ × ν) = ψ dν. (51)<br />

X<br />

X×Y<br />

(b) If f : X × Y → C <strong>and</strong> if one <strong>of</strong><br />

<br />

|f(x, y)| dµ(x) dν(y) < ∞ or<br />

Y<br />

X<br />

holds, then so does the other, <strong>and</strong> f ∈ L 1 (µ × ν).<br />

(c) If f ∈ L 1 (µ × ν), then,<br />

(i) for almost every x ∈ X, f(x, y) ∈ L 1 (ν),<br />

<br />

X<br />

<br />

Y<br />

Y<br />

|f(x, y)| dν(y) dµ(X) < ∞<br />

(ii) for almost every y ∈ Y, f(x, y) ∈ L 1 (µ),<br />

(iii) φ(x) = <br />

Y f dν is def<strong>in</strong>ed almost everywhere (by (i)), moreover φ ∈ L1 (µ),<br />

(iv) ψ(y) = <br />

X f dµ is def<strong>in</strong>ed almost everywhere (by (ii)), moreover ψ ∈ L1 (ν), <strong>and</strong><br />

(v) equation (51) holds.<br />

76


A.2 Complex Analysis A MISCELLANEOUS DEFINITIONS AND THEOREMS<br />

A.2 Complex Analysis<br />

A.2.1 Cauchy’s Theorem 71<br />

A cont<strong>in</strong>uous function γ : [a, b] → C, where [a, b] ⊂ R, is called a path <strong>in</strong> C, <strong>and</strong> such a path is called rectifiable if it<br />

is <strong>of</strong> bounded variation, i.e., if there is a constant M > 0 such that, for any partition a = t1 < t2 < · · · < tn = b <strong>of</strong><br />

[a, b], <br />

i |γ(ti) − γ(ti−1)| ≤ M. In particular, if γ is a piecewise smooth path, it is rectifiable.<br />

If γ : [a, b] → C is a path <strong>in</strong> C, the set <strong>of</strong> po<strong>in</strong>ts {γ(t) : a ≤ t ≤ b} is called the trace <strong>of</strong> γ. Some authors denote<br />

this set by γ∗ , <strong>and</strong> others by {γ}. We will write γ∗ if clarity dem<strong>and</strong>s it. Otherwise, if we simply write z ∈ γ, it should<br />

be obvious that we mean γ(t) = z for some a ≤ t ≤ b. F<strong>in</strong>ally, if γ is a closed rectifiable path <strong>in</strong> C, we call the region<br />

which has γ as its boundary the <strong>in</strong>terior <strong>of</strong> γ.<br />

A curve is an equivalence class <strong>of</strong> paths that are equal modulo a change <strong>of</strong> parameter. If a path γ has some (nonparametric)<br />

property that <strong>in</strong>terests us (e.g., it is closed or smooth or rectifiable), then <strong>in</strong>variably that property is shared<br />

by every path <strong>in</strong> the equivalence class <strong>of</strong> parametrization <strong>of</strong> γ. Therefore, when parametrization has no relevance to<br />

the discussion, we <strong>of</strong>ten speak <strong>of</strong> the “curve” γ, by which we mean any one <strong>of</strong> the paths that represent the curve.<br />

Def<strong>in</strong>ition A.2 If γ is a closed rectifiable curve <strong>in</strong> C then, for w /∈ γ∗ , the number<br />

n(γ; w) = 1<br />

<br />

dz<br />

2πi z − w<br />

is called the <strong>in</strong>dex <strong>of</strong> γ with respect to the po<strong>in</strong>t w. It is also sometimes called the w<strong>in</strong>d<strong>in</strong>g number <strong>of</strong> γ around w.<br />

Theorem A.14 (Cauchy’s formula, ver. 1) Let G ⊆ C be an open subset <strong>of</strong> the plane <strong>and</strong> suppose f ∈ H(G). If γ<br />

is a closed rectifiable curve <strong>in</strong> G such that n(γ; w) = 0 for all w ∈ C \ G, then for all z ∈ G \ γ∗ ,<br />

f(z)n(γ; z) = 1<br />

<br />

f(ζ)<br />

2πi γ ζ − z dζ.<br />

A number <strong>of</strong> important theorems <strong>in</strong>clude a hypothesis like the one above concern<strong>in</strong>g γ – i.e., a closed rectifiable<br />

curve with n(γ; w) = 0 for all w ∈ C \ G (where G is some open subset <strong>of</strong> the plane). This simply means that γ<br />

is conta<strong>in</strong>ed with its <strong>in</strong>terior <strong>in</strong> G. In other words, γ does not w<strong>in</strong>d around any po<strong>in</strong>ts <strong>in</strong> the complement <strong>of</strong> G (e.g.,<br />

“holes” <strong>in</strong> G, or po<strong>in</strong>ts exterior to G). Such a curve γ is called homologous to zero <strong>in</strong> G, denoted γ ≈ 0, <strong>and</strong> a version<br />

<strong>of</strong> a theorem with this as one <strong>of</strong> its hypotheses may be called the “homology version” <strong>of</strong> the theorem.<br />

More generally, if G ⊆ C is open <strong>and</strong> γ1, . . . , γm are closed rectifiable curves <strong>in</strong> G, then the curve γ = γ1 + · · · +<br />

γm is homologous to zero <strong>in</strong> G provided n(γ1, w) + · · · + n(γm, w) = 0 for all w ∈ C \ G. Thus, either theorem A.14,<br />

or the follow<strong>in</strong>g generalization, might be called “the homology version <strong>of</strong> Cauchy’s formula:”<br />

Theorem A.15 (Cauchy’s formula, ver. 2) Let G ⊆ C be an open subset <strong>of</strong> the plane <strong>and</strong> suppose f ∈ H(G). If<br />

γ1, . . . , γm are closed rectifiable curves <strong>in</strong> G with γ = γ1 + · · · + γm ≈ 0, then for all z ∈ G \ γ ∗ ,<br />

f(z)<br />

m<br />

j=1<br />

n(γj, z) = 1<br />

2πi<br />

γ<br />

m<br />

<br />

j=1<br />

γj<br />

f(ζ)<br />

ζ − z dζ.<br />

The next theorem (or its generalization below) might be called “the homology version <strong>of</strong> Cauchy’s theorem:”<br />

Theorem A.16 (Cauchy’s theorem, ver. 1) Let G ⊆ C be an open set <strong>and</strong> suppose f ∈ H(G). If γ is a closed<br />

rectifiable curve that is homologous to zero <strong>in</strong> G, then <br />

f(z)dz = 0.<br />

71 Most <strong>of</strong> the material <strong>in</strong> this section can be found <strong>in</strong> Conway [3].<br />

γ<br />

77


A.2 Complex Analysis A MISCELLANEOUS DEFINITIONS AND THEOREMS<br />

Theorem A.17 (Cauchy’s theorem, ver. 2) Let G ⊆ C be an open set <strong>and</strong> suppose f ∈ H(G). If γ1, . . . , γm are<br />

closed rectifiable curves <strong>in</strong> G such that γ = γ1 + · · · + γm ≈ 0, then<br />

m<br />

<br />

j=1<br />

A partial converse <strong>of</strong> Cauchy’s theorem is the follow<strong>in</strong>g:<br />

γj<br />

f(z) dz = 0.<br />

Theorem A.18 Let G be an open set <strong>in</strong> the plane <strong>and</strong> f ∈ C(G, C). Suppose, for any triangular contour T ⊂ G with<br />

T ≈ 0 <strong>in</strong> G, that <br />

f(z)dz = 0. Then f ∈ H(G).<br />

T<br />

This theorem is still valid (<strong>and</strong> occasionally easier to apply) if we replace “any triangular contour” with “any rectangular<br />

contour with sides parallel to the real <strong>and</strong> imag<strong>in</strong>ary axes.” This stronger version is sometimes called Morera’s<br />

theorem, <strong>and</strong> the exercise on page 81 <strong>of</strong> Sarason [10] asks you to prove it us<strong>in</strong>g theorem A.18.<br />

A.2.2 Maximum Modulus Theorems<br />

Theorem A.19 (max mod pr<strong>in</strong>ciple, ver. 1) Suppose G ⊂ C is open <strong>and</strong> f ∈ H(G) atta<strong>in</strong>s its maximum modulus<br />

at some po<strong>in</strong>t a ∈ G. Then f is constant.<br />

That is, if there is a po<strong>in</strong>t a ∈ G with |f(z)| ≤ |f(a)| for all z ∈ G, then f is constant. 72<br />

Theorem A.20 (max mod pr<strong>in</strong>ciple, ver. 2) If G ∈ C is open <strong>and</strong> bounded, <strong>and</strong> if f ∈ C( ¯ G) ∩ H(G), then<br />

max{|f(z)| : z ∈ ¯ G} = max{|f(z)| : z ∈ ∂G}.<br />

That is, <strong>in</strong> an open <strong>and</strong> bounded region, if a holomorphic function is cont<strong>in</strong>uous on the boundary, then it atta<strong>in</strong>s its<br />

maximum modulus there.<br />

Theorem A.21 (max mod pr<strong>in</strong>ciple, ver. 3) Let G ⊂ Ĉ = C ∪ {∞} be open, let f ∈ H(G), <strong>and</strong> suppose there is an<br />

M > 0 such that<br />

lim |f(z)| ≤ M, for every a ∈ ∂∞G.<br />

z→a<br />

Then |f(z)| ≤ M for all z ∈ G.<br />

Theorem A.22 (Schwarz’s lemma) Let f ∈ H(D), |f(z)| ≤ 1 for all z ∈ D, <strong>and</strong> f(0) = 0. Then<br />

(a) |f(z)| ≤ |z|, for all z ∈ D,<br />

(b) |f ′ (0)| ≤ 1,<br />

with equality <strong>in</strong> (a) for some z ∈ D \ {0} or equality <strong>in</strong> (b) iff f(z) = e iθ z for some constant θ ∈ R.<br />

Please email comments, suggestions, <strong>and</strong> corrections to williamdemeo@gmail.com.<br />

72 This version <strong>of</strong> the maximum modulus pr<strong>in</strong>ciple is an easy consequence <strong>of</strong> the open mapp<strong>in</strong>g theorem, which itself can be proved via the local<br />

mapp<strong>in</strong>g theorem, which <strong>in</strong> turn can be proved us<strong>in</strong>g Rouché’s theorem. Of course, you should know the statements <strong>of</strong> all <strong>of</strong> these theorems <strong>and</strong>,<br />

s<strong>in</strong>ce prov<strong>in</strong>g them <strong>in</strong> this sequence is not hard, you might as well know the pro<strong>of</strong>s too! Two excellent references giv<strong>in</strong>g clear <strong>and</strong> concise pro<strong>of</strong>s<br />

are Conway [3] <strong>and</strong> Sarason [10].<br />

78


B List <strong>of</strong> Symbols<br />

F an arbitrary field<br />

Q the rational numbers<br />

Z the <strong>in</strong>tegers<br />

N the natural numbers, {1, 2, . . .}<br />

C the complex numbers (a.k.a. the complex plane)<br />

Ĉ the extended complex plane, C ∪ {∞}<br />

R the real numbers (a.k.a. the real l<strong>in</strong>e)<br />

T the unit circle, {z ∈ C : |z| = 1}<br />

ˆR the extended real l<strong>in</strong>e, [−∞, ∞]<br />

Rez the real part <strong>of</strong> a complex number z ∈ C<br />

Imz the imag<strong>in</strong>ary part <strong>of</strong> a complex number z ∈ C<br />

D or U the open unit disk, {z ∈ C : |z| < 1}<br />

H(G) the holomorphic functions on an open set G ⊂ C<br />

Π + the upper half-plane, {z ∈ C : Imz > 0}<br />

Π − the lower half-plane, {z ∈ C : Imz < 0}<br />

P + the right half-plane, {z ∈ C : Rez > 0}<br />

P − the left half-plane, {z ∈ C : Rez < 0}<br />

∂∞G the extended boundary <strong>of</strong> a set G ⊂ Ĉ<br />

C[0, 1] the space <strong>of</strong> cont<strong>in</strong>uous real valued functions on [0, 1].<br />

L 1 the space <strong>of</strong> <strong>in</strong>tegrable functions; i.e., measurable f such that |f| < ∞.<br />

L p for 0 < p < ∞, the space <strong>of</strong> measurable functions f such that |f| p < ∞.<br />

L ∞ the space <strong>of</strong> essentially bounded measurable functions;<br />

i.e., measurable f such that {x : |f(x)| > M} has measure zero for some M < ∞.<br />

S ⊗ T the product σ-algebra generated by S <strong>and</strong> T .<br />

References<br />

[1] Lars Ahlfors. Complex Analysis. McGraw-Hill, New York, 3rd edition, 1968.<br />

REFERENCES<br />

[2] Charalambos D. Aliprantis <strong>and</strong> Owen Burk<strong>in</strong>shaw. Pr<strong>in</strong>ciples <strong>of</strong> Real Analysis. Academic Press, New York, 3rd<br />

edition, 1998.<br />

[3] John B. Conway. Functions <strong>of</strong> One Complex Variable I. Spr<strong>in</strong>ger-Verlag, New York, 2nd edition, 1978.<br />

[4] Gerald B. Foll<strong>and</strong>. Real Analysis: Modern Techniques <strong>and</strong> Their Applications. John Wiley & Sons Ltd, New<br />

York, 1999.<br />

[5] James R. Munkres. Topology: A First Course. Prentice Hall International, Englewood Cliffs, NJ, 1975.<br />

[6] H. L. Royden. Real Analysis. Macmillan, New York, 3rd edition, 1988.<br />

[7] Walter Rud<strong>in</strong>. Pr<strong>in</strong>ciples <strong>of</strong> Mathematical Analysis. McGraw-Hill, New York, 3rd edition, 1976.<br />

[8] Walter Rud<strong>in</strong>. Real <strong>and</strong> Complex Analysis. McGraw-Hill, New York, 3rd edition, 1987.<br />

[9] Walter Rud<strong>in</strong>. Functional Analysis. McGraw-Hill, New York, second edition, 1991.<br />

[10] Donald J. Sarason. Notes on Complex Function Theory. Henry Helson, 1994.<br />

[11] Elias Ste<strong>in</strong> <strong>and</strong> Rami Shakarchi. Complex Analysis. Pr<strong>in</strong>ceton <strong>University</strong> Press, 2003.<br />

79


Index<br />

absolute cont<strong>in</strong>uity<br />

<strong>of</strong> functions, 15–16, 73<br />

<strong>of</strong> measures, 6, 14, 72<br />

approximat<strong>in</strong>g <strong>in</strong>tegrable functions, 7<br />

area theorem, 48<br />

Arzela-Ascoli theorem, 57, 62<br />

Baire category theorem, 22<br />

Banach space<br />

ℓp, 35<br />

<strong>of</strong> bounded l<strong>in</strong>ear operators, 26<br />

Banach-Ste<strong>in</strong>hauss theorem, 22, 35<br />

Borel σ-algebra, 34<br />

Borel set, 34<br />

Casorati-Weierstrass theorem, 54<br />

applied, 47, 58<br />

Cauchy’s formula, 75<br />

applied, 39, 47, 56<br />

Cauchy’s theorem, 75–76<br />

converse <strong>of</strong>, see Morera’s theorem<br />

problems, 44, 45, 48, 52, 61<br />

pro<strong>of</strong> by Green’s theorem, 51–52<br />

Cauchy-Riemann equations, 42, 51<br />

closed graph theorem, 30<br />

conformal mapp<strong>in</strong>g<br />

problems, 39, 44, 50, 60, 66, 68<br />

connected, 67<br />

criterion for a pole, 59<br />

curve, 75<br />

disconnected, 67<br />

dom<strong>in</strong>ated convergence theorem<br />

applied, 11, 23, 28, 33<br />

general version, 72<br />

st<strong>and</strong>ard version, 21<br />

Egor<strong>of</strong>f’s theorem, 72<br />

problems, 20, 28–29, 32–33<br />

equicont<strong>in</strong>uity<br />

po<strong>in</strong>twise vs. uniform, 57<br />

equicont<strong>in</strong>uous, 62<br />

even functions, 16<br />

Fatou’s lemma, 72<br />

Fub<strong>in</strong>i-Tonelli theorem, 73–74<br />

80<br />

applied, 8, 13<br />

fundamental theorem <strong>of</strong> algebra, 52<br />

fundamental theorem <strong>of</strong> calculus, 16<br />

Green’s theorem, 51<br />

Hölder’s <strong>in</strong>equality, 22<br />

Hadamard factorization theorem<br />

applied, 58<br />

Hahn-Banach theorem, 22<br />

homologous to zero, 75<br />

implicit function theorem, 18<br />

<strong>in</strong>dex, 75<br />

<strong>in</strong>verse function theorem<br />

<strong>of</strong> calculus, 18<br />

Laurent expansion, 55, 59, 64<br />

Lebesgue decomposition, 73<br />

Liouville’s theorem<br />

applied, 38, 48, 53<br />

maximum modulus theorem, 76<br />

applied, 66<br />

monotone convergence theorem<br />

applied, 4, 5, 12<br />

Montel’s theorem, 57, 62<br />

applied, 45<br />

Morera’s theorem, 76<br />

problems, 48, 61<br />

mutually s<strong>in</strong>gular, 72<br />

normal family, 40, 45, 57, 62–63<br />

odd functions, 16<br />

path, 75<br />

Picard’s theorem, 38<br />

product measures, 8, 73<br />

Radon-Nikodym<br />

derivative, 14, 73<br />

problems, 7–8, 14, 19, 30, 34<br />

theorem, 72–73<br />

removable s<strong>in</strong>gularity theorem, 59<br />

residue theorem, 68<br />

applied, 40, 43, 55, 65, 68


INDEX INDEX<br />

Riemann mapp<strong>in</strong>g theorem, 60<br />

Riesz representation theorem, 35–36<br />

applied, 16–17<br />

for L p , 22<br />

Rouché’s theorem, 53<br />

Schwarz’s lemma, 76<br />

applied, 50, 61, 66, 68<br />

Stone-Weierstrass theorem, 21<br />

applied, 7<br />

Tonelli’s theorem, see Fub<strong>in</strong>i-Tonelli theorem<br />

uniform boundedness pr<strong>in</strong>ciple, 22, 35<br />

w<strong>in</strong>d<strong>in</strong>g number, 75<br />

81

Hooray! Your file is uploaded and ready to be published.

Saved successfully!

Ooh no, something went wrong!